全部 1- 101- 201- 301- 401- 501- 601- 701- 801- 901- 1001- 最新50


面白い問題おしえて〜な 11問目
1 名前:132人目の素数さん:2005/12/12(月) 11:01:19
面白い問題、教えてください

過去ログ
http://www3.tokai.or.jp/meta/gokudo-/omoshi-log/
まとめwiki
http://www6.atwiki.jp/omoshiro2ch/

2 名前:132人目の素数さん:2005/12/12(月) 11:02:58
前スレ

面白い問題おしえて〜な 十問目
http://science4.2ch.net/test/read.cgi/math/1117474512/

3 名前:132人目の素数さん:2005/12/12(月) 13:21:38
おつ

4 名前:132人目の素数さん:2005/12/12(月) 18:03:52
オメガ乙!

(1)ある自然数mが存在して、任意の自然数nをΣ[i=2〜m]ai^i (各aiは非負整数)の形に表すことが出来るか。
↑分かると思うけど、mはnに依らない定数。あと、iが2から始まってるのはタイプミスではない。
(2)非負整数列{fk(n)}(n,k∈Z)が、どんなkについてもfk(0)=0,inf[n∈Z−{0}]fk(n)/n^(k+2)>0
満たすとする。このとき、ある自然数mが存在して、任意の自然数nをΣ[i=0〜m]fi(ai) (各aiは非負整数)の
形に表すことが出来るか。

5 名前:132人目の素数さん:2005/12/12(月) 20:39:09
>>4
(2)fk(n)を偶数のみからなる列にしたらもうだめなんじゃないの?

6 名前:132人目の素数さん:2005/12/12(月) 20:49:57
>>5
その場合は確かに駄目。でも、他の場合は?もしかしたら、駄目じゃないfk(n)が存在するかもしれない。
でも、もしかしたら どんなfk(n)でもアウトかもしれない。それを聞いているのが(2)。

7 名前:132人目の素数さん:2005/12/12(月) 21:02:45
>>6
できる例もってんの?

8 名前:132人目の素数さん:2005/12/12(月) 21:08:40
>>7
それ言ったらヒントになっちゃうじゃないか。

9 名前:132人目の素数さん:2005/12/12(月) 21:09:45
>>4
両方ない気がする。1/nは発散するけど1/fk(n)って収束するから
凸不等式とかつかったらいいんじゃね?

10 名前:953:2005/12/12(月) 21:16:02
サイコロを5回投げるとき、小さい順に出る確率をエレガントに求めよ。

(いろんな解き方があるだろうから、ワクワクテカテカ…)

11 名前:132人目の素数さん:2005/12/12(月) 21:20:22
>>10 全事象は6^5
小さい順ってのは1,1,2,3,4とかはダメなのかな?
ダメだとしたら小さい順になるのは6通りだから
求める確率は6/6^5=1/6^4

12 名前:9:2005/12/12(月) 21:23:09
>>9は撤回。

13 名前:132人目の素数さん:2005/12/12(月) 21:24:36
こんにちわ


14 名前:132人目の素数さん:2005/12/12(月) 21:25:58
>>11
前スレでも書いたが、小さい順ってのは1,1,2,3,4も考慮します。
君たちのエレガントな解法を楽しみにしている年末の寒い夜。

15 名前:9:2005/12/12(月) 21:41:43
むかしフィボナッチ数列にでてくる異なる数の有限個の和でかけない自然数が無限にある
ことの証明で>>9みたいな論法のやつあった記憶があるんだけどあれどうやるんだっけ?
>>9の方法だと任意の自然数が4つの平方数でかけるって定理も否定できてしまう。
思い出そうとしても思い出せん。だれか過去ログ残してない?

16 名前:132人目の素数さん:2005/12/12(月) 21:47:48
>>15
つ「にくちゃんねる」

17 名前:132人目の素数さん:2005/12/12(月) 21:52:57
>>10
とりあえず教科書にでも出てそうな解法

目の出方は全部で 6^5 通り
すなわち 7776 通り

そのうち、小さい順になってる出方は

 ○○|○|○|○||

このような図におきかえて考えると
(図の説明は略)
10!/5!5! 通り すなわち 252 通り

求める確率は 252 / 7776 = 7 / 116

18 名前:132人目の素数さん:2005/12/12(月) 21:59:16
>>17
7/216 ではなからうか?

19 名前:132人目の素数さん:2005/12/12(月) 22:04:52
そうそれ

20 名前:132人目の素数さん:2005/12/13(火) 15:42:32
正方形を作図するとき、コンパスは最低何回必要になるか。

※1 コンパスの軸をずらす、又は描く円の半径の大きさを変える等するまでを1回とする。
※2 点を打って「1辺0cmの正方形」というのは面白くない。

21 名前:132人目の素数さん:2005/12/13(火) 15:44:56
3回はできた
2回は可能だろうか

22 名前:132人目の素数さん:2005/12/13(火) 23:09:06
目盛り付きの三角定規と鉛筆があれば
コンパスなんていらない。

23 名前:132人目の素数さん:2005/12/13(火) 23:17:18
>>9の問題かんがえてるんだけど
∫∫[xi≧1]1/(x1^2+x2^3+・・・+xn^(n+1))dx1dx2・・・dxnってnが大きいと発散するよね?

24 名前:132人目の素数さん:2005/12/13(火) 23:18:13
>>21
2回で可能かどうかはおいといて3回の答えおしえてください。

25 名前:132人目の素数さん:2005/12/13(火) 23:34:54
長方形があれば、そこからコンパスを2回用いて
正方形を切り出せることは容易。

そこでまずOを中心とする円を描き(1回目)、その直径をABとする。
円周上にCを取り、COの延長と円の交点をDとする。
これでADCBは長方形になるから、あと2回で正方形ができる。

26 名前:132人目の素数さん:2005/12/13(火) 23:39:57
直線引いて垂線をひく(コンパス2回)
交点を中心に円を描く

27 名前:132人目の素数さん:2005/12/13(火) 23:41:53
途中で書き込んじゃった。
直線引いて垂直な直線をかく(コンパス2回)
交点を中心に円を描く(3回目)
円がそれぞえの直線と交わったところを結ぶ

28 名前:132人目の素数さん:2005/12/14(水) 00:03:38
2回で可能だね

29 名前:132人目の素数さん:2005/12/14(水) 00:06:47
>>27をヒントに2回でできた。
点Oを中心に円Pを描き、直径ABを描く。
AB以外の円周上の点Cをとると、
直線ACとBCは垂直。
よって、以下>>27と同じ。



30 名前:132人目の素数さん:2005/12/14(水) 00:07:29
>>28
おお!さすがにそれが最小値だろうな。やりかたおしえてくらはい。

31 名前:132人目の素数さん:2005/12/14(水) 00:08:29
>>29
なーるほど。GJ!

32 名前:132人目の素数さん:2005/12/14(水) 00:32:24
1回でいけそうな予感

33 名前:32:2005/12/14(水) 00:43:37
1回でいけた.

(1)中心をOとする円を描く
(2)2本の異なる直径AB,CDをひく.
(3)AC上に適当な点Eをとり,ABとDEとの交点をFとする.
(4)OEとCFとの交点をGとし,AGとBCとの交点をHとすればOHはBCの
中点.
(5)同様の手順でACの中点Iを求める.
(6)OH,OIを延長し円との交点を四つ結べば正方形.

(4)はメネラウスの定理から
(AE/EC)(CD/DO)(OF/FA)=1
AHとOCの交点をPとすればチェバの定理から
(AE/EC)(CP/PG)(GF/FA)=1
が成り立つことからCP/PG=CD/DO=2となり,Pが△ABCの重心となる
ことからHがBCの中点であることが言える.

34 名前:132人目の素数さん:2005/12/14(水) 01:30:44
こういうことか
ttp://www.vipper.org/vip157815.jpg

すごいな

35 名前:132人目の素数さん:2005/12/14(水) 01:33:02
0回は無理ですか?
てかコンパスなしで直角を作るのがそもそも無理?

36 名前:132人目の素数さん:2005/12/14(水) 02:14:24
一回でできるなんて想像すらしなかったな。

37 名前:32:2005/12/14(水) 02:18:51
>>29で直径→直角のアイデアがあったからそれを使ってみた.
中点とるのはだいぶ前に正方形7等分をやった時のと同じ手法.

38 名前:132人目の素数さん:2005/12/14(水) 17:22:24
赤玉3個、白玉2個の入った袋がある。この中から無作為に玉を一つ取り出し、
赤玉が出れば元に戻し、白玉が出れば元に戻さないという試行を繰り返すとき、
n回後に袋の中の白玉がなくなる確率および、白玉がなくなるまでに行う試行回数の期待値を求めよ。

39 名前:132人目の素数さん:2005/12/14(水) 17:54:09
p_n : n 回目までに一個目の白球が出ている確率
q_n : n 回目までに二個めの白球が出ている確率
とすると、
p_n = p_{n-1} + (1 - p_{n-1}) * (2/5)
q_n = q_{n-1} + (1 - q_{n-1}) * (2/4) * p_{n-1}
なので、これを解く。p, q ともに二項間漸か式なのでα^n+β^n みたいな形で出てくる。
n回後に袋の中の白玉がなくなる確率は、q_n - q_{n-1} なのでそれを計算。

白球がなくなるまでの試行回数の計算は
2*(q_2 - 0) + 3*(q_3-q_2) + .... を計算して欲しいというのが出題者の
意図なんだろうけど下のように計算しても問題ない。

一個目の白球が出るまでの試行回数の期待値は5/2
一個目が出た後で二個めの白球が出るまでの試行回数は4/1
ということで、5/2 + 4/1 = 13/2 回

40 名前:39:2005/12/14(水) 17:57:12
ごめん、スレ間違えた……。分からない問題の質問スレかと思った。

41 名前:20:2005/12/14(水) 18:37:33
思いのほか盛り上がっていただけた様で光栄です。
こちらの用意していた解答は出題時点で3回、今日2回を思いついたのですが、
まさか1回があるとは…。>>32氏thx。

ということで追加問題。正規解答は期待しないで下さいまし。

1.任意の長さの1辺を持つ正方形を作図するとき、コンパス(ry
2.正五角形を作図する時、コンパ(ry。また、任意の長さの1辺を持つ正五角形(ry。

42 名前:132人目の素数さん:2005/12/14(水) 18:40:38
前スレ使い切ろーぜ

43 名前:132人目の素数さん:2005/12/15(木) 11:11:47
円積問題が解けたと聞いて飛んできました。

44 名前:132人目の素数さん:2005/12/15(木) 13:25:35
飛べるのですか?うらやましい限りです。

45 名前:132人目の素数さん:2005/12/15(木) 14:32:31
I=[0,1]とする。
(1)有限個の有界な開区間(a1,b1),…,(an,bn)でIが被覆されるとき、Σ[i=1〜n](bi−ai)>1
なることを数学的帰納法で示せ。
(2)NからIへの全単射が存在しないことを、(1)及びIのコンパクト性を用いて示せ。

46 名前:32:2005/12/16(金) 02:47:58
>>41
1.は前の考えを使いまわして1回で可能.
与えられた長さを半径に持つ円を描けば,できる正方形の一辺はその
√2倍.
もう一度同じ方法で各辺に直交する直径がかけるから,それを使って
でき上がり.

2.は考え中.正三角形なら1回で作れた.

47 名前:132人目の素数さん:2005/12/16(金) 04:47:07
>>33
間違いを訂正しろ。
話はそれからだ!

48 名前:132人目の素数さん:2005/12/16(金) 08:33:44
>>47
>>33はおかしいの?

49 名前:132人目の素数さん:2005/12/16(金) 14:51:25
チェバの定理から〜の後

(AE/EC)(CP/PG)(GF/FA)=1
が成り立つことからCP/PG=CD/DO=2となり,

(AE/EC)(CP/PO)(OF/FA)=1
が成り立つことからCP/PO=CD/DO=2となり,

ってことじゃない?

50 名前:132人目の素数さん:2005/12/16(金) 18:48:47
△ABCにおいて∠A=180/7度、BC=1である。
また頂点Cから辺ABにAD=1となるような点Dをとる。
このときCDの長さを求めよ。



51 名前:132人目の素数さん:2005/12/17(土) 01:11:10
>50
問題の条件が足りないと思う。

52 名前:132人目の素数さん:2005/12/18(日) 03:50:57
実数列{an}に対して、Σ[i=1〜∞]aiが収束しているとする。また、実数列{bn}がβ∈Rに収束しているとする。
このとき、Σ[i=1〜∞]aibi は収束すると言えるか?

53 名前:132人目の素数さん:2005/12/18(日) 04:25:39
普通に微積か実数論の練習問題じゃね?

54 名前:132人目の素数さん:2005/12/18(日) 05:25:59
>>52
宿題は質問スレに書け、厨房め!

55 名前:52:2005/12/18(日) 08:09:12
は?宿題じゃねぇし。どうせアレだろ、ε−δ論法を少し弄れば「収束する」ことが証明できるとでも思ったんだろ?
息抜き程度の難易度で、それなりに楽しめる問題を見つけたと思ったんだけどなぁ…馬鹿に出題したのが間違いだったよ。

解答:収束するとは言えない。
略証)ある実数列{an}が存在して、どんなβ∈Rに対しても、βに収束する適当なbnを選べばΣaibiが発散することを示す。
an={(-1)^(n−1)}/nで定義される実数列{an}を考える。Σ[i=1〜∞]ai=log2 となって収束する。
bn=1/logn (nは偶数),0 (nは奇数) とするとbn→0であるが、Σ[i=1〜∞]aibi=−Σ[i=1〜∞]1/(2ilog2i)
となり、2ilog2i はi≧1において単調増加だからΣ[i=1〜∞]1/(2ilog2i)=Σ[j=0〜∞]Σ[i=2^j〜2^(j+1)−1]1/(2ilog2i)
≧Σ[j=0〜∞]Σ[i=2^j〜2^(j+1)−1]1/(2*2^(j+1)log2*2^(j+1))=Σ[j=0〜∞]1/{4(j+2)log2}=∞だから
Σ[i=1〜∞]aibi=−∞となって発散する。一般のβ∈Rに対しては、Bn=β+bn とすればBn→βであるが
ΣaiBi=βlog2+Σaibi=βlog2−∞=−∞となって 発散する。

56 名前:52:2005/12/18(日) 08:09:41
(連投失礼)
考察1:一般に、絶対収束はしないが条件収束する{an}を取ってくれば、an=(an)^(+)−(an)^(-) (ただしx^(+)=max{0,x},
x^(-)=−min{0,x})と分解するとΣ(ai)^(+)=Σ(ai)^(-)=+∞,Σ{(ai)^(+)−(ai)^(-)}=α∈Rとなっていることが
分かるから、σn→0,Σσi(ai)^(+)=+∞を満たすσnを選んで bn=σn (an≧0),0 (an<0)とbnを構成することによって、
Σaibiが発散するようにできる。bn→β≠0の場合は、上の解答と同じように作ればよい。で、σnの構成の一例は次。
Sm=Σ[i=1〜m](ai)^(+)とおくと 仮定よりSmは単調増加しながら発散するから、任意のn∈Nに対してあるM∈Nが存在して
m≧Mのとき常に2^n<Smが成り立つようにできる。このようなMのうち最小のものをMnとおくと、Mnは(広義)単調増加する
自然数列で2^n<SMn が常に成り立つ。便宜上、Mnは狭義単調増加にしておきたいので、Xn=Mn+nとでもおけばXnは狭義
単調増加で2^n<(SMn≦)SXnが常に成り立つ。そこで、σnをσn=1 (1≦n≦X1),1/(k+1) (Xk<n≦X(k+1)) と定義すれば
Σ[i=1〜Xn]σi(ai)^(+)≧(1/n)Σ[i=1〜Xn](ai)^(+)=(SXn)/n>(2^n)/n→∞ (n→∞)及びσn→0 (n→∞)が成り立つから、
このσnが求める数列の一例。
考察2:{an}が絶対収束する場合は、Σaibi は必ず収束する。

グチャグチャの計算だから計算ミスがあるかも(^^;

57 名前:132人目の素数さん:2005/12/18(日) 08:45:00
a(n)=b(n)=(-1)^n/n^(1/2).


58 名前:132人目の素数さん:2005/12/18(日) 09:57:38
コンパスと定木だけで作図できない正多角形はあるか?
あるなら何か?

59 名前:132人目の素数さん:2005/12/18(日) 10:59:59
>>58
それは超有名問題だな。ガウスが解いたやつだ。

60 名前:132人目の素数さん:2005/12/18(日) 13:49:38
>>41
これコンパス一回つかえば任意の作図可能な点が作図できるみたい。・・・たぶん

61 名前:132人目の素数さん:2005/12/22(木) 04:32:01
以下の問いに答えよ。

(1)
△ABCの垂心をH、外心をO、OからABに下ろした垂線の足をO'とする。CH=2OO'を示せ。

(2)
(1)の△ABCについて、線分OHの中点Uを中心として半径が△ABCの外接円の半径の1/2の円を描く。また、CからABに下ろした垂線の足をE、線分CHの中点をVとする。
E、V、O'はこの円上に存在することを示せ。

(3)
(1)、(2)を利用して『9点円定理』を示せ。

62 名前:132人目の素数さん:2005/12/22(木) 14:04:59
>61
誰か解いてよー(´Д`)

63 名前:132人目の素数さん:2005/12/22(木) 17:05:51
実数論とか解析学ちっくな問題を出すと宿題扱いされるスレはここですか?

64 名前:132人目の素数さん:2005/12/22(木) 19:10:40
ふつーに演習問題として載ってそうな問題だと宿題扱いだね。

65 名前:132人目の素数さん:2005/12/22(木) 19:39:22
少なくとも>>61は宿題だね。

66 名前:132人目の素数さん:2005/12/22(木) 21:26:35
ずいぶん主観的だな。

67 名前:132人目の素数さん:2005/12/22(木) 22:09:14
>>61の学友です。
これは塾の宿題です。
キモイので話しかけたことはないのですが、ハッキリ言ってクズです。
本当にありがとうございました。

68 名前:132人目の素数さん:2005/12/23(金) 01:58:43
その問題が宿題だろうがなんだろうが、出されたもん解くのが筋だろーが!

俺は解けないが

69 名前:132人目の素数さん:2005/12/23(金) 02:35:36
補助線

70 名前:132人目の素数さん:2005/12/23(金) 10:44:44
>>68
「香織はバスローブを脱ぎ捨てると、その豊満な」まで読んだ。

71 名前:132人目の素数さん:2005/12/23(金) 10:46:12
眼科いった方がいいよ。

72 名前:132人目の素数さん:2005/12/23(金) 10:57:53
さみしいなあ

73 名前:GiantLeaves ◆6fN.Sojv5w :2005/12/23(金) 17:52:09
talk:>>70 香織のスリーサイズを求めよ。

74 名前:132人目の素数さん:2005/12/23(金) 21:20:42


∀⌒0000000=(    )

75 名前:132人目の素数さん:2005/12/23(金) 22:12:56

数という概念を否定せよ。

さあ、道徳、いや どう解く ?


76 名前:132人目の素数さん:2005/12/24(土) 00:15:42
>>74


哲学厨か

77 名前:132人目の素数さん:2005/12/24(土) 01:34:53
>>74
AAサロン
http://aa5.2ch.net/aasaloon/

ここで存分に

78 名前:数セミから拝借してチョット変えてみました。:2005/12/24(土) 02:19:28
次の条件を満たす平面上の閉曲線S考える。

・Sは原点を通る任意の直線と2点で交わる
・Sは原点を中心とする単位円周の任意の接線と2点で交わる
・|a|<1とする。Sは直線y=aと2点で交わる
・|a|<1とする。Sは直線x=aと2点で交わる

このとき、Sは原点を中心とする単位円周と言えるか。

79 名前:132人目の素数さん:2005/12/24(土) 02:22:47
>>78
>・Sは原点を中心とする単位円周の任意の接線と2点で交わる
 
こんな条件があんのになんでSが単位円周になりえるわけ?

80 名前:132人目の素数さん:2005/12/24(土) 02:25:56
楕円 x^2/9 + y^2/4 = 1 とかでもいいような

81 名前:132人目の素数さん:2005/12/24(土) 02:29:28
>79
すみません。書き間違えました。2点ではなく1点です。

82 名前:132人目の素数さん:2005/12/24(土) 02:30:21
>>81
一点のみを共有する?交わるじゃなくて?

83 名前:132人目の素数さん:2005/12/24(土) 02:34:19
>82
交わるです。

84 名前:132人目の素数さん:2005/12/24(土) 02:35:17
>>83
じゃあ単位円周でもその条件みたしてないじゃん。

85 名前:132人目の素数さん:2005/12/24(土) 02:38:29
>84
何故ですか?馬鹿ですみません。1点を共有する=1点で交わるとの解釈です。

86 名前:132人目の素数さん:2005/12/24(土) 02:42:02
>>85
普通わざわざ“交わる”という単語をつかうときは“接する”状態はふくめないだろ?
てか“共有点をもつ”ってかけばその手のまぎらわしさなしに伝わるんだから。
なんでわざわざそんな誤解をうむ言葉づかいするんだよ?

87 名前:132人目の素数さん:2005/12/24(土) 02:44:44
>>79
“閉曲線”は“単純閉曲線”?

88 名前:132人目の素数さん:2005/12/24(土) 02:52:06
>86
接することも交わることとの解釈からです。
あと、(単位円周でないようなSがあった場合に)交わるとの表記の方が一般性を持つだろうとの判断です。
誤解を与えたならば申し訳ありません

89 名前:132人目の素数さん:2005/12/24(土) 02:54:34
>87
そういう方向で。単純でない場合はこちらも全く考えていなかったので…。

90 名前:132人目の素数さん:2005/12/24(土) 03:32:23
>>89
じゃあこんな感じで。
S1={(x,y)|x^2+y^2=1}とおく。同型写像f:S1→Sを固定しておく。
以下閉半直線L(θ)をL(θ)={(rcosθ,rsinθ) | r≧1}
とする。まずL(θ)∩S=φを示す。あるp∈L(θ)∩Sがとれたとする。
M:{(x,y)| xcosθ+ysinθ=1}、N:{(x,y)| xcosθ+ysinθ=-1}とおく。
仮定よりS∩Nは空でないのでq∈S∩Nをとる。同型写像g:S1→Sを
g((1,0))=p、g(-1,0)=qととる。
gの連続性と直線Mによって分割される2つの開半平面のことなる成分にp,qがあるから
からα=(cosu,sinu)、β=(cosv,sinv)をg(α)、g(β)∈S∩M、0<u<π、-π<v<0ととれる。
しかしこれはS∩Mが1元集合であることに反する。
よってS∩L(θ)は任意のθに対し空である。
これをもちいてSが単位円であることをしめす。仮定により任意のφにたいして
S∩{(x,y)| xcosφ+ysinφ=1}は1元集合{p}とかけるがもしp≠(cosφ、sinφ)であれば
pはあるθをとってL(θ)にふくまれる。これは前段の帰結に反する。
よってS∩{(x,y)| xcosφ+ysinφ=1}=(cosφ,sinφ)である。これからSが単位円周となることは容易。□

91 名前:132人目の素数さん:2005/12/24(土) 04:37:25
・Sは単純閉曲線
・Sは原点を中心とする単位円周の任意の接線と1点で交わる

↑この2条件だけでよくね?

92 名前:91:2005/12/24(土) 05:50:45
デキタ。でも説明するのマンドクセ。泥臭い方法だし。

93 名前:132人目の素数さん:2005/12/24(土) 07:45:51
問題文が長いのは勘弁してください

94 名前:132人目の素数さん:2005/12/25(日) 21:08:01
実数列{an}は、Σaiが条件収束するとする。このとき、任意の実数xに対して、数列{an}を適当に並べ替えた
数列{aφn}を選べば、Σaφiがxに収束するように出来る。
↑このスレで出された問題では無いが、やっと解けた(´∀`)

95 名前:132人目の素数さん:2005/12/25(日) 21:34:00
嫌いな料理はどんどん先送りにして、好きなもんだけ食ってくやつだろ。
まあ、どれもいつかは食わなきゃならんわけだが。

96 名前:132人目の素数さん:2005/12/25(日) 23:08:11
>>94
せっかくだから解答かいてちょ。

97 名前:94:2005/12/26(月) 06:17:59
>>96
いや、すんごく泥臭いやり方なので勘弁して(^^;

98 名前:132人目の素数さん:2005/12/26(月) 06:59:53
とりあえずxを超えて大きくなるまで正の項を足して、次に
xより小さくなるまで負の項を足して、またxを超えて大きくなるまで正の項を足して、
またxより小さく(りゃ

でしょ

99 名前:132人目の素数さん:2005/12/26(月) 07:47:59
わかるまで考えたのは立派だが
それもよくある演習問題でしょうにw

100 名前:132人目の素数さん:2005/12/26(月) 14:54:40
>それもよくある演習問題でしょうにw

でつ ………俺、このスレの’演習問題を馬鹿にする’ような雰囲気大嫌いだな。

101 名前:132人目の素数さん:2005/12/26(月) 20:12:39
>>100
るぱん賛成だぜ、とっつぁん。

102 名前:132人目の素数さん:2005/12/26(月) 20:25:36
演習問題がバカにされてるっていうよりは、
このスレの趣旨からして「よくある」問題は趣旨に合ってないってことだろ
その意味で「よくある演習問題」はバカにされても当然だと思うよ

103 名前:132人目の素数さん:2005/12/26(月) 20:38:18
馬鹿にするために1レス消費するくらいなら、スルーするか、もしくは’面白い問題’とやらを書き込んで欲しいな。
厨房め〜 とか荒れるだけだし。

104 名前:132人目の素数さん:2005/12/26(月) 20:41:25
バカにされた本人なら気分が悪いのも当然だとは思うけど、
でもスレ違いに「スレ違いだよ」って言ってあげるのは
スレの自治としては必要なことなんじゃね

105 名前:132人目の素数さん:2005/12/26(月) 20:47:02
ああ、こんな だから最近つまらんのか。

106 名前:132人目の素数さん:2005/12/27(火) 00:27:33
>>86名大の試験では「接する」も「一点で交わる」って表現するぞ。


107 名前:132人目の素数さん:2005/12/27(火) 00:31:29
名大は馬鹿だからな

108 名前:132人目の素数さん:2005/12/27(火) 02:17:59
>>103
スルーしない同レベルのバカの方が多いだろが

109 名前:132人目の素数さん:2005/12/27(火) 02:20:53
>>103
> 1レス消費するくらいなら、スルーするか、もしくは’面白い問題’とやらを書き込んで欲しいな。
オマエモナー

110 名前:132人目の素数さん:2005/12/27(火) 02:45:02
そもそもスレの趣旨が、面白い問題教えろ、だからな。曖昧すぎ。

111 名前:132人目の素数さん:2005/12/27(火) 02:55:44
ざっと見ただけでこれだけあるのに
よくある演習問題をわざわざこのスレに書いて
文句を言われると駄々をこねる奴の気が知れん。


分からない問題はここに書いてね225
http://science4.2ch.net/test/read.cgi/math/1135406867/

くだらねぇ問題はここへ書け ver.3.14(40桁略)3993
http://science4.2ch.net/test/read.cgi/math/1132560000/

◆ わからない問題はここに書いてね 182 ◆
http://science4.2ch.net/test/read.cgi/math/1134810000/

おいらには解けない4
http://science4.2ch.net/test/read.cgi/math/1097325180/

おまいらこの問題解ける?
http://science4.2ch.net/test/read.cgi/math/1135016339/

112 名前:132人目の素数さん:2005/12/27(火) 07:01:04
当人が面白いと感じているのなら、そんなスレには書き込まんわな。

113 名前:132人目の素数さん:2005/12/27(火) 19:57:31
n を1より大きな自然数とする。
(x^x)^n = y^y が非自明な自然数解 (x, y) を持つための n の条件を求めよ。
ただし、自明な解とは (x, y) = (1, 1) のことを指している。

114 名前:132人目の素数さん:2005/12/27(火) 20:38:03
>>113
n=lx^(l-1)となる2以上の整数xと自然数lが存在すること。
は少なくとも一つの解答になるかな?用意されてる解答とちがうかもしれないけど。
この手の問題は答えが一意にさだまらないからな。

115 名前:132人目の素数さん:2005/12/27(火) 21:10:23
>>114
もっと多くの n でいけますよ〜。

116 名前:132人目の素数さん:2005/12/27(火) 21:14:01
>>115
まじっすか?n=lx^(l-1)とあらわせないnの例をあげてみてください。

117 名前:132人目の素数さん:2005/12/27(火) 21:23:19
n=(b/a)x^((b/a)-1)とあらわせるとき?

118 名前:132人目の素数さん:2005/12/27(火) 21:28:05
たとえば、n=9なら上の条件に当てはまらないですけど、
(x, y) = (2^24, 2^27) が解になってます。

>>117
その形だとどんな数が当てはまらないんだろう…
もしかしたらあってるかもしれない。答えはもっと単純ですよ

119 名前:132人目の素数さん:2005/12/27(火) 21:38:42
>>118
あてはまらないものがない以上正解なんじゃないの?

120 名前:132人目の素数さん:2005/12/27(火) 21:46:43
十分条件にはなってても必要十分じゃなくね?

121 名前:132人目の素数さん:2005/12/27(火) 21:52:36
でも反例ないよ。

122 名前:132人目の素数さん:2005/12/27(火) 21:55:47
非自明解が存在しない n もちゃんとあるんですけど……。

123 名前:132人目の素数さん:2005/12/27(火) 21:59:24
素数じゃないときはいつでもそんな表示をもつんじゃね?
n=uv u,v≠1のときx=((u-1)v)^(u-1)、b=u、a=u-1とおけば
(b/a)x^((b/a)-1)=(u/(u-1))((u-1)v)^(b-a)=uv=nになる気がする。
素数はそのような表示ないし。素数じゃないときはOKなんじゃね?

124 名前:132人目の素数さん:2005/12/27(火) 22:04:15
>>118
それ、解になってないと思うが。もしかして(x^x)^nとx^(x^n)を勘違いしてないか?

125 名前:132人目の素数さん:2005/12/27(火) 22:08:23
なってるんじゃね?
(x^x)^n=(2^(24・2^24))^9=2^(2^27・3^3)
y^y=(2^27)^(2^27))=2^(2^27・3^3)

126 名前:124:2005/12/27(火) 22:08:32
しまったw n=3だと思ってた。ゴメン。あとsage。

127 名前:132人目の素数さん:2005/12/27(火) 22:15:47
ふううううん

128 名前:124:2005/12/27(火) 22:16:21
んん!?n≧3のとき、x=(n−1)^(n−1),y=(n−1)^nは非自明な解にならないか?

129 名前:132人目の素数さん:2005/12/27(火) 22:21:27
まさにそう。もう大方解けたね。
あとは、n=2でどうなのか、と。

130 名前:124:2005/12/27(火) 22:24:49
いや、それも簡単。n=d(c+1) (d,cは自然数)と表したときに、x=(dc)^c,y=(dc)^(c+1) が解になる。

131 名前:124:2005/12/27(火) 22:26:56
ORZ
orz
λ...
(お察し下さい)

n=2のときは解無しですた。

132 名前:113:2005/12/27(火) 22:39:26
とりあえず、お疲れ〜。おれが用意していた解答は
STEP1
t = y/x を使って問題文の式を整理すると
x = t^(t/(n-t)), y = t^(n/(n-t))
とパラメータ表示できる。
t=n-1を代入すればx, yが自然数になるので
n>2の時の非自明解を生成できる。

STEP2.
n=2のとき背理法により解がないことを証明していく。
まず、x^(2x) = y^y から、x < y < 2x ---(1) を示す。
次に、任意の x の素因数 p をとってきて、右辺と左辺を
pの指数に関してのみ比較し、yがxの倍数であることを示す。
でも、yがxの倍数であることが(1)に矛盾。

よって答えは、n>2 という感じでした。

133 名前:124:2005/12/27(火) 23:00:38
補題)自然数a,b,x,yが、a^x=b^yを満たすとき、x≧yならばa|b,x≦yならばb|aが成り立つ。
証:x=yのとき…明らか。x>yのとき…a=dα,b=dβ (α,βは互いに素)と表示すると、与式⇔{d^(x−y)}α^x=β^yと
変形できる。左辺はαで割り切れるから、右辺のβ^yもαで割り切れる。ところがα,βは互いに素だからα=1を得る。
このときa=d,b=dβ=aβとなって、a|bが成り立つ。x<yのとき…同様にしてb|aとなる。

これを用いて、(x^x)^2=y^yに非自明解が無いことを示す。
x≧2,y≧2としてよい。f(z)=z^zはz≧1において単調増加なので、もしy≧2xとするとx^(2x)=y^y≧(2x)^(2x)
⇔1≧2^(2x)が成り立つことになって矛盾。よってy<2xである。これに補題を用いると、y<2xよりx|yを得る。
つまりy=cx とおける。これを与式に入れて整理するとx^2=(cx)^cとなるので、c=1,c≧2で場合分けすると
非自明な解が存在しないことが容易に分かる。

134 名前:132人目の素数さん:2005/12/28(水) 00:33:08
超有名問題なんだが・・・知らない人ドゾ
 
n個のコインがあり一個だけニセモノがまじっておりニセモノは重さがちがう。
r≧log[3](2n+1)をみたす自然数rをとるとき天秤をr回もちいればニセモノをさがしだせる事を示せ。

135 名前:134:2005/12/28(水) 00:39:03
あ、書き忘れてた。重さがちがうことだけはわかってる。重いか軽いかはわかんないという設定で。

136 名前:132人目の素数さん:2005/12/28(水) 10:12:41
(1)a^(1+b^2)=b^aの自然数解(a,b)を全て求めよ。
(2)f(x),g(x)をxの非負整数係数多項式でf(x)の定数項は1とする。また、g≠0とする。
このとき、a^f(b)=b^{a*g(a)}の自然数解(a,b)を全て求めよ。

137 名前:132人目の素数さん:2005/12/30(金) 17:39:54
>>136
(1,1) しかないね。

p を a の素因数とし、p^s|a , p^{s+1}†a, p^r|b, p^{r+1}†b とする(s>0, r>0)。
p^{s f(b)} = p^{ar g(a)} <=> s f(b) = ar g(a)
右辺、a は p^s の倍数。
一方、左辺、f(b) は (p の倍数 + 1) なので、s は p^s の倍数でないといけない。
s > p^s を満たすような素数 p と自然数 s は存在しないので変。
よって、a = 1 でないといけない。そうすると b = 1 も自動的に決定。

138 名前:GiantLeaves ◆6fN.Sojv5w :2005/12/30(金) 18:43:08
みかんの皮をむかずに中身だけを取り出すのは不可能であることを証明せよ。

139 名前:132人目の素数さん:2005/12/30(金) 18:44:14
ストローさしてぐちゅぐちゅにして飲む

140 名前:132人目の素数さん:2005/12/30(金) 18:44:55
あぁ、ごめん。不可能なことの証明ねw

141 名前:132人目の素数さん:2005/12/30(金) 18:45:39
今年も30時間をきりました。

142 名前:132人目の素数さん:2005/12/30(金) 23:04:35
みかんの皮の内側と外側ってどっちが内側でどっちが外側だろうね

143 名前:132人目の素数さん:2005/12/30(金) 23:29:16
凍らして割ればいいんじゃね?

144 名前:132人目の素数さん:2005/12/30(金) 23:57:39
縦横高さ以外の軸のほうから掻き出せばいいんじゃね?

145 名前:132人目の素数さん:2005/12/31(土) 00:02:07
え〜〜〜〜〜とっ・・・・・・・・どこだよそれっ!!!

146 名前:132人目の素数さん:2005/12/31(土) 00:13:14
>138
みかんを閉曲面って考えたらいいのかい?

147 名前:132人目の素数さん:2005/12/31(土) 00:36:24
面積が7で辺の長さがすべて有理数となるような直角三角形の3辺の長さを決定せよ。

(一つでも見つかればぉk)

148 名前:小牧愛佳 ◆u/3FmANAKA :2005/12/31(土) 01:18:41
a,b,c,dを自然数として

(bd/ac)*(b^2/a^2-d^2/c^2)=7を満たすa,b,c,dを見つけてください・・・

149 名前:132人目の素数さん:2005/12/31(土) 01:25:15
>147
35/12、24/5、337/60が見付かった。

150 名前:132人目の素数さん:2005/12/31(土) 13:24:14
類題:大福の皮を破らずに中のアンコを食べることは可能であるか。

151 名前:GiantLeaves ◆6fN.Sojv5w :2005/12/31(土) 13:27:03
talk:>>146 みかんの皮は球面と位相同型となる曲面と見る。
talk:>>150 雪■大福の場合は不可能。

152 名前:GiantLeaves ◆6fN.Sojv5w :2005/12/31(土) 13:27:49
talk:>>146 当たり前だが、三次元空間の中の曲面だぞ。

153 名前:132人目の素数さん:2005/12/31(土) 13:28:42
皮に穴の開いている大福を選ぶ

154 名前:132人目の素数さん:2005/12/31(土) 15:40:16
>151
内部の任意の点と外部の任意の点を結んだ直線と曲面が少なくとも1つの共有点を持つことを示せばいいのかな?

155 名前:小牧愛佳 ◆u/3FmANAKA :2005/12/31(土) 16:54:45
>>147の類似問題です

面積が41で辺の長さがすべて有理数となるような直角三角形の3辺の長さを決定してください

一つでも見つかればおっけーです


156 名前:132人目の素数さん:2005/12/31(土) 17:42:52
合同数ですか。

157 名前:132人目の素数さん:2005/12/31(土) 19:46:48
楕円曲線の有理点みつける問題だな。テートの教科書に有理点みつけるための
アルゴリズムの存在は証明されてるけど実際の実行はめちゃめちゃ大変ってかいてあった。
加藤先生の数論1か2あたりに似たような例が載ってた記憶があるんだけど。

158 名前:GiantLeaves ◆6fN.Sojv5w :2005/12/31(土) 20:31:53
talk:>>154 それでいいが、できるかな?

159 名前:小牧愛佳 ◆u/3FmANAKA :2005/12/31(土) 21:07:47
面積が素数で辺の長さがすべて有理数となるような直角三角形は

100以下の素数では5と7と41以外では存在しないと思うのですが・・・

そしてその組み合わせはそれぞれ一つしかないと思います・・

反例があったらご指摘よろしくお願いしますです。

160 名前:132人目の素数さん:2006/01/01(日) 07:19:03
駄目だ、解けない…

互いに異なる3つの自然数a1,a2,a3で、
ai|(aj+ak+aj*ak) ((i,j,k)=(1,2,3),(2,3,1),(3,1,2))
を満たすものは存在するか。

161 名前:GiantLeaves ◆6fN.Sojv5w :2006/01/01(日) 07:37:45
talk:>>160 a1=1,a2=3,a3=7 など。

162 名前:160:2006/01/01(日) 07:40:34
しまった…何やってんだ俺。訂正します。

互いに異なる3つの素数a1,a2,a3で、
ai|(aj+ak+aj*ak) ((i,j,k)=(1,2,3),(2,3,1),(3,1,2))
を満たすものは存在するか。

163 名前:132人目の素数さん:2006/01/01(日) 09:50:10
>>159
こつこつやるのは良いことだが、ここくらいは参考にした方がよい。

http://www.asahi-net.or.jp/~KC2H-MSM/mathland/math10/

164 名前:132人目の素数さん:2006/01/01(日) 10:19:21
>>162
素数・・・

165 名前:小牧愛佳 ◆u/3FmANAKA :2006/01/01(日) 10:55:45
>>163
ご指摘ありがとうございます

定義1までは作れたんですが・・・こんなのあったんですね・・・

166 名前:132人目の素数さん:2006/01/01(日) 13:26:23
>158
当たり前な気がするけど、チャント示すとなると…。

167 名前:132人目の素数さん:2006/01/01(日) 14:16:20
>>150
大福を皮ごと喰う

168 名前:132人目の素数さん:2006/01/01(日) 14:21:45
とりあえず、大福の皮を食べずにアンコだけ食べるような変なやつとは絶交だ。

169 名前:132人目の素数さん:2006/01/01(日) 21:09:19
>>158
曲座標で考えれば、中間値の定理とかですぐ証明できそうな気がしないでもないけど
これだと不完全ですか?

170 名前:132人目の素数さん:2006/01/01(日) 22:39:15
まずは2次元で考えたらどうかな。単位円と位相同型な曲線で。

171 名前:GiantLeaves ◆6fN.Sojv5w :2006/01/02(月) 13:55:46
とりあえず、三次元空間内で球面の内側の点と外側の点を曲線で結ぶと、曲線のどこかが球面上にあることは認めよう。
(証明しようと思えばできるだろう。)
先に定めた二点が、内側、外側になる曲面で、球面と位相同型であるものと、球面の間に、適当なイソトピー写像を作れば[>>138]ができる。
初めに、イソトピー写像ができることを示せばあとは簡単かな?

問題改。
三次元空間内で、球面と位相同型である曲面と球面との間にイソトピー写像を構成できることを示せ。

できるのかもしれないが、どうやろう?あとは幾何学の専門家にパス。

172 名前:GiantLeaves ◆6fN.Sojv5w :2006/01/02(月) 14:02:01
こんな難しい問題は私にもできないので、簡単な問題にしよう。
三次元空間内で、球面の内部の点と同じ球面の外部の点を曲線で結んだとき、
曲線と球面が少なくとも一点で交わることを証明せよ。
(ヒント:曲線とは何か?)

173 名前:132人目の素数さん:2006/01/02(月) 14:16:07
>>172
ヲイヲイ

174 名前:GiantLeaves ◆6fN.Sojv5w :2006/01/02(月) 14:19:02
talk:>>173 簡単すぎる?

175 名前:132人目の素数さん:2006/01/02(月) 14:53:10
47 :132人目の素数さん :2006/01/02(月) 12:40:28
2次の正方行列 A,B が A^2-2AB+B^2=O を満たせば AB=BA と言えるか?

48 :132人目の素数さん :2006/01/02(月) 12:55:42
言える。と思う。

49 :ぴかぽん :2006/01/02(月) 13:14:47
>47
言えない。

176 名前:132人目の素数さん:2006/01/02(月) 15:14:30
↓の問題お願いします。
ttp://www10.plala.or.jp/mathcontest/2005r.htm

177 名前:1:2006/01/02(月) 19:22:40
謹賀新年。

地上に置くと、秒速1cmで歩く虫がいる。
長さが10cmの、いくらでも延びるゴムひもがある。

(1) 虫をゴムひもの一端に乗せ、ひもの上を他端に向けて歩かせる。
同時に、ゴムひもが毎秒10cmの割合で延びていくとする。
このとき虫は他端に到着できるだろうか?

(2) ゴムひもの長さを表す函数をr(t)とするとき、虫が他端に
到着できるためにr(t)が満たすべき条件はどんなものか?
ちなみに(1)では、r(t)=10+10tである。

なお、ゴムひもは一様に延びるものとする。たとえば、ひもの中点に
印を付けると、ひもがどんなに延びても印は中点のままである。

178 名前:132人目の素数さん:2006/01/02(月) 21:49:06
>>177
ゴムひもがのびることによって虫のスピードはあがると考えていいすか?
t秒放置したのち一瞬でt・10cmのばすという操作を繰り返すとかんがえて
t→0とするとかんがえると虫は毎秒1cmよりはやくなるよね?この方針で桶?

179 名前:1:2006/01/03(火) 02:12:44
>>178
ひもの一端を地面に固定した上で、虫の対地速度を考えれば、それでおk。
この場合、たとえば中点の対地移動速度は5cm/sなので、
仮に虫が中点にいたとすると、その瞬間には虫は対地速度6cm/sで
移動していることになる。

なお(2)については、およその見当はついているものの、
厳密な解答は持っていませんのであしからず。

180 名前:132人目の素数さん:2006/01/03(火) 02:19:04
hg

181 名前:132人目の素数さん:2006/01/03(火) 02:59:19
>>179
だとするとこう?
ゴムひもをx=0に固定しもう片方の端がx>0をx=r(t)にしたがってうごくとする。
このゴムひも上をx軸正の方向にゴム上を等速度vでうごくとする。
t秒後の虫の位置をs(t)としてs’=v+(s/r)r’。変形して(s/r)’=v/r。
s(0)=0からs/r=∫[0,t](v/r)dt。“虫がもう一端に達する⇔s/r=1となる時刻がある”である。
(1)r=10t+10、v=1のときs/r=∫[0,t](1/(10t+10))dt=(1/10)(log(10t+10)-log10)であるからt=e^10-1で到達する。
(2)もう一端に達しない⇔∫[0,t](1/r)dt<1/v (∀t)
たとえばr(t)=ve^tのときならもう一端には到達しない。

182 名前:1:2006/01/03(火) 03:33:58
>>181
ご名答。俺も今さっき、(2)も同じ結論に達したところ。
「虫がもう一端に達する⇔∫[0,T](v/r)dt=1となる正の時刻Tがある」ですな。

v=1、r(t)=10+t^2 とすると、もう無理。
v=1、r(t)=10+at^2 (a>0)の場合、aが小さければ届きそうなんだけど、
計算が面倒なので放棄。

蛇足ながら、(1)の、到達できることの簡単な説明を。
まず、ひもの1/10の地点が1cm/sで動いているのに対し、
虫は1cm/sより若干は速く進んでいることを考えれば、
1/10の地点には到達できる。この瞬間の虫の対地速度は2cm/s。
ひもの2/10の地点が2cm/sで動いているので、2/10地点にも到達できる。
この瞬間の虫の対地速度は3cm/s。以下同様にして、3/10、4/10、‥と
次々に到達できることがわかる。

183 名前:132人目の素数さん:2006/01/03(火) 15:55:34
ま、着くまでに虫の寿命がくr

184 名前:132人目の素数さん:2006/01/03(火) 16:31:33
>v=1、r(t)=10+at^2 (a>0)の場合

a<π^2/40 なら到達可能。

185 名前:132人目の素数さん:2006/01/05(木) 17:30:23
1の原始n乗根をa_1、a_2、…、a_tとする。
Π(1-a_t)がとりうる値をすべて決定せよ。

186 名前:132人目の素数さん:2006/01/05(木) 23:21:27
円周上に頂点を持つn多角形を考える(n≧4)。
この時、そのn多角形の内角が最低でも1つ、鋭角となる確率を求めよ。

187 名前:132人目の素数さん:2006/01/06(金) 00:30:00
0,1,素数。


188 名前:132人目の素数さん:2006/01/06(金) 19:13:05
>>186
半円上に n-1 個以上の頂点が来ればいいのかな?
でも計算の仕方が分からんw

189 名前:132人目の素数さん:2006/01/06(金) 22:59:56
186解いてみたけど、計算が合わんし、どこ間違ってるのか分からん。

円周上に並んだ頂点を左回りにP1, P2, P3, ... , Pn と命名。
命題 Ai : 「頂点 Pi が鋭角である」 と定義。
このとき、求める確率は
P(A1∪A2∪A3∪...∪An ) = ΣP(Ai) - ΣP(Ai∩Aj) + ΣP(Ai∩Aj∩Ak) -+...
ここで、隣り合わない2頂点が共に鋭角になることはない(証明略)ことから
P(A1∪A2∪A3∪...∪An ) = ΣP(Ai) - ΣP(Ai∩A{i+1})
= nP(A1) - nP(A1∩A2)

P(A1) は Pn(位置を固定して考える) を端点にもつ半円の片側に
残りn-1点のうちn-2点が含まれる確率なので P(A1) = (n-1)/2^{n-2}
P(A1∩A2) は、arg(Pn)=0 、θ < arg(P1) < θ+dθ の位置にあるとすると
(1) 残りの n-2点が π+θ < arg(Pi) < 2π に存在する確率と
(2) 一点が π < arg(Pi) < π+θ、n-3点が π+θ < arg(Pi) < 2π に存在する確率
の和である。

190 名前:132人目の素数さん:2006/01/06(金) 23:09:15
計算すると
(1) は (n-1)!/(n-3)! (dθ/(2π)) (θ/(2π)) ((π-θ)/(2π))^{n-3}
(2) は (n-1)!/(n-2)! (dθ/(2π)) ((π-θ)/(2π))^{n-2}
P1の位置が 0 < arg(P1) < π の範囲内での(1)と(2)の確率を足し上げて
P(A1∩A2) = (n-1)/(2π)^{n-1}∫{0->π} (n-2) θ(π-θ)^{n-3} + (π-θ)^{n-2}
= 1/2^{n-2}

nP(A1) - nP(A1∩A2) に以上の結果を代入して
P(A1∪A2∪A3∪...∪An ) = n(n-1)/2^{n-2} - n/2^{n-2} = (n^2-2n)/2^{n-2}

となったんだけど、n=4 を代入すると 確率が 2 になるのでありえない。
どっかで間違ってることは確かなんだけど、それがどこなのかわからない(´・ω・`)

191 名前:132人目の素数さん:2006/01/06(金) 23:12:30
袋の中に、1から6までの数字が書いてある球が2個ずつ合計12個ある。
この中から3個の球を同時に取り出す。
取り出した球の数字について考える。
(1) 3つの数の和が5である確立
(2) 3つの数字のうち、最も大きい数が4である確立
(3) 3つの数の積が偶数である確立

192 名前:132人目の素数さん:2006/01/06(金) 23:30:00
1324という並びを考えたり考えなかったりしているところ。


193 名前:189:2006/01/06(金) 23:58:17
>>192
確率の計算をするときは、Pn の位置を固定しているだけで
他の頂点は後付けで番号がつくように計算できてるとおもう。

194 名前:132人目の素数さん:2006/01/07(土) 00:14:12
4つの数字を+−×÷使って10にする、、、てやつで難しいの教えてください!

195 名前:189:2006/01/07(土) 02:25:57
間違ってたところが分かった。
Pnを固定して残りn-1点を円周上に配するのと、円周上にn点配するのは
全く別のことだ。
前者だとPnとP1の間の角が後者の場合よりも小さくなりがちだ。

計算やり直しだな。

196 名前:132人目の素数さん:2006/01/07(土) 05:41:09
>>189
頑張ってくれ。解答もちゃんと用意してあるから。

197 名前:132人目の素数さん:2006/01/07(土) 14:11:08
>>196
同様に確からしい、をどう定義するかで確率が変わるから、そこはキチンと明示してくれよ。

198 名前:132人目の素数さん:2006/01/07(土) 14:20:23
普通に円周上の一様分布でいいだろ

199 名前:132人目の素数さん:2006/01/07(土) 14:26:00
(1,0)-(-1,0)-(0,1)-(0,-1)-(1,0).


200 名前:196:2006/01/07(土) 14:30:50
>>198で。

201 名前:188:2006/01/07(土) 15:33:17
計算の仕方を思い付いた。

n=5の場合。
1点目はどこでもよい。
2〜5点目は、1点目から見て+πラジアンまでの位置 か -πラジアンまでの位置かのいずれかにある(いずれも確率1/2)。
前者を'+' 後者を'-' と表記すると、題意を満たす2〜5点目の配置は、

++++
-+++
+-++
++-+
+++-

----
+---
-+--
--+-
---+

の 10(=5*2) 通り。
よって、求める確率は、
10*((1/2)^4)
= 5*((1/2)^3)
= 5/8

同様の議論で、頂点数がnの場合、求める確率は、
n*((1/2)^(n-2))

一応、n=4 で確率1になるので、正しげなんだけど、どう?

202 名前:188:2006/01/07(土) 15:46:37
あ、でもよく考えるとおかしいな。
+1/100π, -1/100π みたいなのばかり、1点目の近くに密集している場合も題意を満たしますね。

203 名前:189:2006/01/07(土) 16:16:13
モンテカルロで間違い探ししながら解いてみた。
スマートじゃなくてすまない(´・ω・`)

>>189-190の計算で間違ってたのは P(A1) の計算だったみたい。
点Pn を arg(Pn) = 0 の位置に固定したときに、P1 が鋭角になるのは
Pn 以外の n-1個の点が
(1) 全て π < arg(P) < 2π の位置にある
(2) 1個は 0 < arg(P) < π で、n-2個が π < arg(P) < 2π の位置にある
の二通りで
(1) の確率は (1/2)^{n-1}
(2) の確率は (n-1) * (1/2)*(1/2)^{n-2}
∴ P(A1) = 1/2^{n-1} + (n-1)/(2^{n-1}) = n/2^{n-1}

他は、189-190の通りに計算すれば、求める確率は
n (n/2^{n-1} + 1/2^{n-2}) = n(n-2)/2^{n-1}

204 名前:132人目の素数さん:2006/01/07(土) 16:44:16
俺の今の状態は一生に一度しかあり得ないのか?

205 名前:132人目の素数さん:2006/01/07(土) 18:30:52
>>188
正方形の場合は鋭角が1つもない。
だから、n=4のときは1未満になるよ。

>>189
これは、1問でも解けたら凄いと言われる、あるテスト(大学生用)からとってきた問題なんだけどね・・・。
凄いね、大正解!おめでとう。

206 名前:189:2006/01/07(土) 22:44:37
>>205
ありがとう。

もっと簡単なときかたないのかな。
出てくる数値が簡単だからなんとなくいい方法がありそうな気がするんだけど。

207 名前:132人目の素数さん:2006/01/07(土) 22:45:57
鳩ノ巣原理と絡むのか?

208 名前:132人目の素数さん:2006/01/08(日) 08:20:31
シンプルなのをひとつ。

1
∫((ln(x+1)) / (x^2+1))dx
0

これを計算せよ。

209 名前:132人目の素数さん:2006/01/08(日) 08:33:31
>>208
なつかし〜な、それ。

210 名前:132人目の素数さん:2006/01/08(日) 09:02:42
宿題乙。

211 名前:132人目の素数さん:2006/01/08(日) 10:24:39
>>210
宿題じゃありませんが何か?
てか普通に考えて、宿題だったら別の所に書き込むだろ。
頭を使うから面白いと思っただけ。文句があるなら放置すれば良いじゃん。

212 名前:132人目の素数さん:2006/01/08(日) 11:02:14
>>211
もまいの書き込むタイミングが悪いから変なちゃちゃが入った
とは解せんのか?

213 名前:132人目の素数さん:2006/01/08(日) 11:34:38
日米数学研究所の存続を廻って
JH内で意見が分かれていること。

214 名前:132人目の素数さん:2006/01/08(日) 13:13:18
>>212
そか、円の問題の出題者が帰って来てないか。
もうその問題は解決してると思ってたんだが、スマソ。

215 名前:132人目の素数さん:2006/01/08(日) 14:19:30
>>211
>>102>>104
このスレはこういうスレなのさ。

216 名前:132人目の素数さん:2006/01/08(日) 14:22:46
てか普通に宿題だと思う。

217 名前:132人目の素数さん:2006/01/09(月) 07:03:02
なんかつまらないことになってんな。
確かに出す方も出す方だが、騒いでる方も5分で解いてやって、
「これは簡単すぎるから、もっと面白い問題を持って出直してこい」とか言えばいいものを。
そうしてやれば、出してる方も「こいつらは実は解けないからこういう風にしか言えないんだ」とか変に誤解することもないだろうし。
っていうのも>>215の言うようにスレ違いか。
それから、>>210 = >>216はそんなに煽らなくていいだろ。
また>>211が帰って来てややこしくなるじゃんよ。粘着しすぎ。もうちょっと考えろ。
あと考えたが、俺的には>>211は的を得てるね。これは多分宿題じゃない。
ただ、>>211は問題がこのスレの住人が求めているものではないと分かっていないだけじゃね?そんな>>211は半年ROMって来い。
・・・それとも、>>208は、本当はそんなに難しいのか?基本に見えるが。

218 名前:132人目の素数さん:2006/01/09(月) 08:12:17
>あと考えたが、俺的には>>211は的を得てるね。これは多分宿題じゃない。
>>63-64

俺のイメージだと、うまい変形を考えて解く定積分の問題と、うまい数え方を考えて解く確率の問題を
このスレに出した場合、前者は宿題、後者は面白い(?)問題 という扱いになると思う。要するに>>63

219 名前:132人目の素数さん:2006/01/09(月) 08:58:57
簡単すぎる、とか文句をたれるのは
実際に解いてからしろ、というのには酸性だけど、
問題を出す側にも、
問題文だけ読んである程度面白さが伝わるようにする
というのは必要かもね。

解いてみるまで面白さがわからないのだと、
解く側のモチベーションが上がりにくいし、
不必要に煽る奴も出てくるんじゃないかな。

220 名前:132人目の素数さん:2006/01/09(月) 09:04:37
            議  論  終  了
_____________________________________



_____________________________________

            ス  レ  再  開


以降は、>>218-219を参考にしてマターリ行きましょう。

221 名前:132人目の素数さん:2006/01/09(月) 10:35:14
数列f(n)はf(1)=0,f(2)=2,f(3)=3,f(n)=f(n−2)+f(n−3) (n≧4)を満たしている。
「f(n)/nが自然数になる⇔nが素数である」は正しいか。

222 名前:132人目の素数さん:2006/01/09(月) 12:20:18
ttp://web2.incl.ne.jp/yaoki/chahi3.htm

みたことある

223 名前:132人目の素数さん:2006/01/09(月) 14:01:50
よくある問題の焼き直しだけど
http://tv8.2ch.net/test/read.cgi/zurui/1096206531/80

224 名前:132人目の素数さん:2006/01/09(月) 14:43:27
>>218
問題集に載ってるよ。
DQN向けの大学教養の微積の問題集に載ってるよ。
去年、演習問題のレポートにもなったし。
変数変換して解くだけ…
5分もかからない問題、面白いと言うほどの問題ではない。

225 名前:132人目の素数さん:2006/01/10(火) 02:16:52
>>223の関連問題
3枚の金貨があるが、その中に本物よりも1グラム軽い偽物が含まれる可能性がある。
偽物は1枚かもしれないし2枚かもしれないし、もしくは3枚とも偽物、あるいは全部本物かもしれない。
天秤と、本物と分かっている金貨を何枚でも使うことができるが、
どれが偽物か完全に判別するのは、天秤2回では不可能なことを示せ。

(もちろん、金貨を通貨として使うなどのとんち系の解法は禁止である)

226 名前:132人目の素数さん:2006/01/10(火) 10:28:48
>>224
>>220
親切心のつもりかもしれんが、蒸し返すのはやめれ

227 名前:132人目の素数さん:2006/01/10(火) 21:21:57

_____________________________________

            議  論  再  開


228 名前:132人目の素数さん:2006/01/10(火) 22:21:27
俺の最も嫌いなことの一つは、仕切りたがるやつに 「NO!」 と言ってやることだ!

229 名前:132人目の素数さん:2006/01/10(火) 22:37:50
昨日帰りの満員電車で小便漏らしました。
その日は頻尿だったのでヤバイかなぁ〜と思いつつ電車に乗ったんだよね。
自分が降りる駅の一つ手前で、なんと!人身事故で停車・・・
乗ってる間も小便を我慢してたんで俺の顔は真っ青でやばい汗ダラダラだったと思う。
案の定、我慢できず脱尿、脱尿、脱尿、だっぷん・・・・・
まず、しゃがんでカバンに入っていたコンビニ袋を取り出しました。
俺の行動に周りはいい迷惑だったと思う。たぶんゲロを吐くと思われたと・・・
でも俺はなんのタメライもなくズボンとパンツを下げ、チンコ丸出しで
そのビニール袋に発射。。。
「 ブシュブシュ ブシュブシュ ブシュ〜」と俺のチンコから
「ポタッ ポタッ ポタッ」と袋へ落ちる
悪臭と周りの人達の驚きの眼差し・・・そして罵声・・・。
俺は顔をあげられない・・・チンコをポケットティッシュで拭き、俺はしゃがんだまま・・・。
約1時間そのままだった・・・
やっと電車が動いた時、いままで罵声を言ってた周りの人が
「よかったな、はやくうち帰って風呂入れよ^^」
ってやさしく声をかけてくれた。
俺はあまりの惨めさ、恥ずかしさで泣いた。
でも本当は感謝の言葉で大泣きしていた俺がいた。

230 名前:132人目の素数さん:2006/01/11(水) 11:27:43
A君が駅から毎時4キロの速さで、3・5キロ離れた自宅まで歩き出した。
しかし途中で雨が降ってきたので、毎時9キロの速さで走ったところ30分で家に着いた
A君が走り出ししたのは駅を出発してから何分後?

231 名前:132人目の素数さん:2006/01/11(水) 12:56:47
東大スレよりコピペ
---
関数x(t)、y(t)はx:[0,1]→R、y:[0,1]→Rの連続関数であり、
x(0)=y(0)=0、x(1)=y(1)=1を満たす。

この時、次の二つの問いに答えよ。
(1)
任意の実数t∈(0,1)に対し、x(t)、y(t)のうち少なくとも一つが無理数である。
という条件を満たすx(t)、y(t)の例を一つ求めよ。

(2)
任意の実数t∈(0,1)に対し、x(t)、y(t)のうち少なくとも一つが有理数である。
という条件を満たすx(t)、y(t)は存在するか。

232 名前:132人目の素数さん:2006/01/11(水) 16:46:27
「任意の凸多面体は三角形、四角形または五角形のいずれかを
少なくとも一つ面として持つことを示せ。」

ヒント:面あたりの辺の本数の平均を評価すればよい。

233 名前:132人目の素数さん:2006/01/11(水) 17:36:35
>>231
(1) たとえばx(t)=t, y(t)=log2(t+1)
(2) すべての自然数と,0<a<1を満たすすべての有理数は、オイラーの対角線論法によって
1対1に対応することが示されている。したがって(x(t),y(t))の組と対応するtも、自然数と1対1に対応しなければならない。
ところが実数tの濃度は自然数より濃いから、これは矛盾である。

234 名前:132人目の素数さん:2006/01/11(水) 17:58:14
>>233
(2)はおれもそれ考えたけど、x(t)やy(t)がある値を
無限回とる場合とかはまずいよね。なんか引っ掛けっぽい。

答えは存在するよね。
x(t)
= 0 on [0, 1/2)
= 2t - 1 on [1/2, 1]

y(t)
= 2t on [0, 1/2)
= 1 [1/2, 1]

235 名前:132人目の素数さん:2006/01/11(水) 18:40:55
>>234
(2)に何か条件を付けてみたらどうなる?
如何なる小さな(空でない)区間でも定数でないとか。

236 名前:132人目の素数さん:2006/01/11(水) 19:23:08
>如何なる小さな(空でない)区間でも定数でないとか。
単調増加ってこと?
だったら>233 の議論が成立してるよね。

237 名前:132人目の素数さん:2006/01/11(水) 19:49:37
>>236
単調とは全然限らないよ。
例えば折れ線グラフ。
これでは(反)例は出来ないだろうが。

238 名前:132人目の素数さん:2006/01/11(水) 21:16:30
230は算数で言う速さとつるかめ算の問題かな?引っ掛けあるなら知りたい

それより問題って自分で考えたのでもいいの?

239 名前:132人目の素数さん:2006/01/11(水) 21:18:11
反例が存在するとすると、x(t) か y(t) がある値を
不加算回とることになると思うんだけど、仮に
x(t) = c が不加算個の零点を持つとすると
どこかに連続濃度で零点の集積点が存在してて
そっから定数区間の存在とか言えそうな気がする。

直感で書いてるので変なこと言ってるかもしれないが。


240 名前:132人目の素数さん:2006/01/11(水) 21:22:17
とりあえず普通は非加算って言うと思う

241 名前:132人目の素数さん:2006/01/11(水) 21:29:33
非可算だろう

242 名前:238:2006/01/11(水) 21:30:06
やっぱ書きます
簡単だと思うんで、もしあれならスルーしてくらさいΣは文字表現上わかりにくいから使わないですけど了承くださいネ。

数列の一般項を
An=sin(n×75゚)と定義します
この時
A1からA100までの和を求めなさい


243 名前:132人目の素数さん:2006/01/11(水) 21:34:43
あーこれは12が周期だけどあちこち打ち消してとかそんな感じかな

>>241
ワロスw

244 名前:132人目の素数さん:2006/01/11(水) 22:07:35
せーかーい
97〜100までの和は常識と和積で解けます
糸も簡単に…
良問と自負してたのに

245 名前:132人目の素数さん:2006/01/11(水) 22:19:20
いや面白いとは思うよ

246 名前:132人目の素数さん:2006/01/11(水) 22:23:17
>>245
おととい、センター模試の質問でそんなのがあって
解いたばかりなので、どこが面白いんって感じだった。すまん

247 名前:132人目の素数さん:2006/01/12(木) 00:00:45
センター模試?マジですか…
その問題見たいです。お暇なら貼ってください

次、たまにある問題ですけど…もしかしたら非常によくある問題かも
@〜Gまでの番号の付いた玉がそれぞれ1個ずつあります。
同じく@〜Gまでの番号の付いた箱が1個ずつあります。
ここで@〜Gまでのボールを@〜Gまでの箱に一つずつ入れる時、同じ番号同士セットにならないように入れる入れ方は何通りあるでしょうか

248 名前:132人目の素数さん:2006/01/12(木) 00:12:30
かくらん順列って言われるやつだと思うけど
答えはきれいにはならなかった気が

249 名前:132人目の素数さん:2006/01/12(木) 00:16:47
>>231
x(0)=y(0)=0、x(1)=y(1)=√2
に変えるとどうなる?

250 名前:132人目の素数さん:2006/01/12(木) 00:33:22
(0,0)→(1,0)→(1,1)→(1.4, 1)→(1.4, 1.4)→(1.41, 1.4)→(1.41, 1.41)→(1.414, 1.41)→...
な折れ線で(√2, √2) まで結んでいけばok

あー、お茶漬けがたべたい。

251 名前:132人目の素数さん:2006/01/12(木) 00:38:34
(1,0)→(1,1)はまずかろう

252 名前:132人目の素数さん:2006/01/12(木) 00:41:12
>>250
なるほど

253 名前:132人目の素数さん:2006/01/12(木) 01:47:03
>>247
7!

254 名前:253:2006/01/12(木) 01:48:49
違った。7*7!

255 名前:253:2006/01/12(木) 01:54:03
やっぱ違うわ。分らん。

256 名前:132人目の素数さん:2006/01/13(金) 20:47:36
関数x(t)、y(t)はx:[0,1]→R、y:[0,1]→Rの連続関数であり、
x(0)=y(0)=0、x(1)=y(1)=1
任意の実数t∈(0,1)に対し、x(t)、y(t)のうち少なくとも一つが有理数である。
という条件を満たすとする。

このとき、x-y平面上の曲線( x(t),y(t) ) t∈[0,1]の長さの最小値を求めよ。


257 名前:132人目の素数さん:2006/01/14(土) 00:39:33
面白そうなので甜菜。

[0,1]×[0,1]の部分集合Eに対し
Ex={y|(x,y)∈E}
Ey={x|(x,y)∈E}
と定める。このとき、次を同時に満たすEが存在することを示せ。

・各 x∈[0,1] に対して、Exは[0,1]から高々可算個の点を除いた集合である。
・各 y∈[0,1] に対して、Eyは[0,1]の高々可算個の点から成る集合である。

http://science4.2ch.net/test/read.cgi/math/1136598062/800

258 名前:132人目の素数さん:2006/01/14(土) 17:48:37
むずかしそーだ。

259 名前:132人目の素数さん:2006/01/16(月) 20:01:24
2006/mを10進表記したときに循環小数となるようなmをすべて決定せよ。

260 名前:132人目の素数さん:2006/01/16(月) 20:08:49
>>259
m=2006の時、明らかに
与式 = 1.0000……
となり、循環小数である。

これは、他のどのような有理数でも成立するため、
答え m≠0を満たす全ての有理数。

261 名前:132人目の素数さん:2006/01/16(月) 23:15:36
>>257
連続体仮説を仮定すれば楽勝。
Fubini の定理に関連して、測度論の本に例として載っていると思う。

仮定しない場合は、Turing degree とかを使うのではないかと。

262 名前:132人目の素数さん:2006/01/18(水) 12:14:04
数オリの問題解いてたら思いついた問題。

nを奇数、a1,…,anを相異なる整数とする。多項式 1+Π[i=1〜n](x−ai) が
Z[x]上 既約にならないことはあるか。

263 名前:132人目の素数さん:2006/01/18(水) 19:05:14
>>262
もとの数オリの問題はどんな問題?
それが分かれば分かるかも

264 名前:132人目の素数さん:2006/01/20(金) 09:00:51
>>263
                         /⌒/   へ    \|\
            /           /  /   /( ∧  ) ヘ ヘ
           く           // ( /| | V )ノ( ( (  ヘ\   お  て
    ┘/^|    \         (  | |ヘ| レ   _ヘ|ヘ ) _ヘ    し  め
    /|   .|              |  )) )/⌒""〜⌒""   iii\    え |
     .|  α  _          ヘ レレ  "⌒""ヘ〜⌒"  ||||>    て
          _∠_       イ |  |  /⌒ソi   |/⌒ヘ  <    や に
     _     (_        ) ヘ  | ‖ () ||  || () ||  _\   ん は
     /               (  ) ヘ |i,ヘゝ=彳  入ゝ=彳,i|\    ね
    /ー               ( /  """/   ー""""   >   |
      _)   |          ヘ(||ii    ii|||iiii_/iii)ノヘ|||iiiii<   |||||
          |          ( ヘ|||||iiii∠;;;;;;;;;;;;;;;;;;;;;;;;;;;;;;;;;;     フ    ""'
    /////   ヘ_/       ) ヘ|||""ヘ===二二二===7フ / ム/∧ ∧ ∧
    /////              (  | ii  | |LL|_|_LLL// |    )( ∨| ∨)
   ・・・・・                ) )| || | |||||||||||||||||||||||| | |   ( ヘ | ヘ ) (
          ___        | | /| .| |||/⌒/⌒ヘ | | |  iiiiヘ ( | ( | /
            /         / (|.| | |       | | |  iii  ) | ヘ )( )
            (          ( /..|  | |_____/ | |  iii  ( )( // /
            \         ) )..|  |ヘL|_|_L/ / /  ,,,,--(/Vヘ)(/
                       / ( .|ヘ \_ヘ |_/ / /

265 名前:132人目の素数さん:2006/01/20(金) 15:30:54
>260
ちょwwww確かにあってるけどw

266 名前:132人目の素数さん:2006/01/20(金) 18:20:14
>>264
馬鹿は引っ込め
>>262
http://www.iis.it-hiroshima.ac.jp/~ohkawa/math/math_pro_a017.htm

267 名前:132人目の素数さん:2006/01/20(金) 20:36:49
>>266
      ( ゜д゜ ) 
  ゴト /| y |)  ターンしちまぇよ…
 / ̄;y=ー ̄ ̄/||
/_____/. ||
||       || ||
||       ||

268 名前:132人目の素数さん:2006/01/22(日) 15:53:46
0≦x≦1、pを0≦p≦1なる実数として、関数F_p(x)を以下で定義する。
F_p(x)=pF_p(2x) (0≦p≦1/2)
F_p(x)=(1ーp)F_p(2xー1)+p (1/2≦p≦1)

(1)F_p(x)は連続関数であることを示せ。

(2)xが二進法展開でx=1/2^a_0+1/2^a_1+1/2^a_2…1/2^a_n(a_0<a_1<a_2<…<a_n)なる表示を持つとき、
F_p(x)=p^a_0+tp^a_1+(t^2)p^a_2+…(t^n)p^a_n
を示せ。

269 名前:132人目の素数さん:2006/01/22(日) 23:24:08
>268これは難問

270 名前:132人目の素数さん:2006/01/23(月) 13:52:05
>>268
> F_p(x)=p^a_0+tp^a_1+(t^2)p^a_2+…(t^n)p^a_n
t の定義は?

271 名前:268:2006/01/23(月) 15:47:16
すまん、t=(1-p)/pだ

272 名前:132人目の素数さん:2006/01/24(火) 17:44:31
平方数であるフィボナッチ数は1と144に限ることを示せ。

273 名前:132人目の素数さん:2006/01/24(火) 22:41:53
>>272
そんな既出問題より、どうせなら立方数で行こうぜ

274 名前:132人目の素数さん:2006/01/26(木) 08:56:12
平面上に面積 1 の円がある。
この円の面積を4本の異なる直線で6等分せよ。
ただし、4本の直線はどれも、円と二点で交わるものとする。

275 名前:132人目の素数さん:2006/01/26(木) 15:34:41
>>274
√3/4 < π/6 だから、存在は中間値の定理より明らか。

276 名前:132人目の素数さん:2006/01/26(木) 21:50:15
>>275
解説キボン

277 名前:132人目の素数さん:2006/01/27(金) 14:16:00
>>275
何か勘違いしてないか?直線4本だぞ?5本なら(左から右に並べて縦に引けば)明らかだが。

278 名前:132人目の素数さん:2006/01/27(金) 15:33:19
>>277
中心角が 2π/3 の扇形 2 個を半分にすればいいんでしょ。
頂角が 2π/3 で斜辺の長さが 1 の二等辺三角形の面積は √3/4 じゃないの。

279 名前:132人目の素数さん:2006/01/27(金) 18:20:10
やっと、言いたいことが分かった。

んじゃ、同じ問題で9分割してみてくれ。

280 名前:277:2006/01/27(金) 21:58:01
>>278
それ、「線分」だったら出来るけどさ、「直線」だと無理でしょ?そういう二等辺三角形を
書いた時点で、(引いた直線によって)既に書いてある他の扇形がさらに分割されてしまう。

281 名前:132人目の素数さん:2006/01/27(金) 22:09:19
>>280
そうでもない、画像作ってウプするから、しばらく待て

282 名前:132人目の素数さん:2006/01/27(金) 22:14:46
http://v.isp.2ch.net/up/401ff9ef599b.png

超下手な画像だが、とりあえず上げて見た。


解説をすると、まず、二本の直線を円の直径になるように引く。
この際、二本の直線は60°で交わるように引く。

その後、中心角120°の扇型が二つ出来上がるが、
この二つを二等分すれば、それで六等分割が完成する。


>>279
9等分割の間違いだろ。 できなさそうだな。

283 名前:132人目の素数さん:2006/01/28(土) 00:59:42
下手すぎてワロスw
最低限円の中心を通るようにかけよw

284 名前:277:2006/01/28(土) 08:58:47
>>282
ホントだ…ちゃんと面積計算したら、扇形の弧の部分で交わるように直線引いて やっと2等分に
なるんだな。もっと中心の近くに直線引かないと2等分されないと思ってた。サンクス。

285 名前:132人目の素数さん:2006/01/28(土) 15:22:19
計算しなくても、菱形を描くとわかるのだが。

286 名前:132人目の素数さん:2006/01/28(土) 16:14:51
んで、誰か九等分割できる猛者はいないのか?

287 名前:132人目の素数さん:2006/01/29(日) 21:03:47
a,bを自然数とする。
aをbで割った商をQ(a,b)、余りをR(a,b)と書く。

このとき、連立方程式
Q( (a^2+b^2), (a+b) ) = Q( (a^3+b^3), (a^2+b^2) )
R( (a^2+b^2), (a+b) ) = R( (a^3+b^3), (a^2+b^2) )
を解け。

288 名前:132人目の素数さん:2006/01/29(日) 21:10:10
>>287
a≠bを忘れた。

289 名前:132人目の素数さん:2006/01/31(火) 04:04:21
age

290 名前:132人目の素数さん:2006/01/31(火) 06:16:54
おせーて

291 名前:132人目の素数さん:2006/01/31(火) 08:30:00
ab(a−b)=0。


292 名前:132人目の素数さん:2006/02/05(日) 08:17:02
713

293 名前:132人目の素数さん:2006/02/11(土) 18:17:51
(tan(3pi/11)+4sin(2pi/11))^2
が整数である事を証明できますか?

294 名前:132人目の素数さん:2006/02/11(土) 22:17:54
もう実際の値を求めちゃったほうが早くないかな

295 名前:132人目の素数さん:2006/02/12(日) 00:21:48
>>294
そりゃ、電卓で計算すりゃ、その整数は11だってのが分かるけど、
それじゃ数学じゃ無いじゃん。

296 名前:132人目の素数さん:2006/02/12(日) 00:23:13
nを自然数、P(x)をn次の多項式とする。
P(0),P(1),・・・P(n)が整数ならば、すべての整数kに対し、P(k)は整数であることを証明せよ。


297 名前:132人目の素数さん:2006/02/12(日) 00:59:50
面白くない+マルチ=>>296

298 名前:132人目の素数さん:2006/02/12(日) 01:01:06
>>294
いや、11であることを直接示さずに、
整数であることだけ示すのは難しいだろう、ということ

299 名前:132人目の素数さん:2006/02/12(日) 01:45:24
>>298
難しくない。 5 + 6 である事を示せばよい。

300 名前:132人目の素数さん:2006/02/12(日) 19:03:54
転載。
http://science4.2ch.net/test/read.cgi/math/1136270347/597
任意の自然数nに対して、xy平面上の正方形で、その周上および内部に
合計でちょうどn個の格子点を含むものが存在することを示してください。

円ならどうか?

301 名前:132人目の素数さん:2006/02/12(日) 20:39:44
>>300
点(√2,√3)から各格子点までの距離は全て異なる。なぜなら、2つの格子点(x,y),(a,b)に対して
(x−√2)^2+(y−√3)^2=(a−√2)^2+(b−√3)^2とすると、式変形して(x−a)√2+(y−b)√3
=(x^2+y^2−a^2−b^2)/2∈Qとなるが、一般に有理数p,q,rに対してp√2+q√3=rが成り立つのは
p=q=r=0に限られるから、x=a,y=bとなり(x,y)=(a,b)となるから。
そこで、点(√2,√3)から各格子点までの距離を小さい順にr1,r2,r3,…とすると、点(√2,√3)を
中心として半径r (rn<r<r(n+1))の円を書けば、この円の周上には格子点がなく、内部にはちょうどn個の
格子点が存在するので、これが求める円となる。

302 名前:132人目の素数さん:2006/02/12(日) 21:18:03
平面上にn個の凸図形(S1,S2,…Sn)がある。
任意の整数i,jに対してSi∩Sjが空集合でないとき、
S1∩S2∩…∩Snが空集合でないことを示せ。

303 名前:132人目の素数さん:2006/02/12(日) 22:10:23
問題間違ってない?

n本の直線を平面上にどの3本も同じ点で交わらないように配置して、
これを十分長い、細長い長方形に置き換えると、
Si∩Sjが空集合でないは満たすけど、
どの i,j,k に対しても Si∩Sj∩Sk=φになるよね。


304 名前:132人目の素数さん:2006/02/12(日) 22:14:58
>>303
ぬお、本当だ……あれ・・・三次元だと似たような話があったような気がしたんだけど
もう少し考えてくるわ

305 名前:132人目の素数さん:2006/02/14(火) 00:44:11
123
456
789

と9マスに分かれたタイルがあり、1と3に黒いナイト(チェスの駒)
7と9に白いナイト(チェスの駒)がおいてある。同じ番号のマスに
2つの駒は置けないとして、白の駒を1と3、黒の駒を7と9に
入れ替えるのは最低何手か? 最低の手数であることも証明してね。
(ナイトの駒の動きはわかるよね。左右上下に将棋の桂馬みたいに
 動かせます)

◆ わからない問題はここに書いてね 186 ◆より
http://science4.2ch.net/test/read.cgi/math/1139675746/148


306 名前:132人目の素数さん:2006/02/14(火) 02:10:12
>>293
p を 4n-1 型の素数として、
x を 1 の原始 p 乗根、g を p の原始根とすると
(x - x^g + x^(g^2) - x^(g^3) + … - x^(g^(p-1)))^2 = -p
が成り立つ。

p = 11, g = 2, x を 1 の原始 11 乗根とすれば
x - x^2 + x^4 - x^8 + … - x^512
= x - x^2 + x^4 - x^8 + x^5 - x^10 + x^9 - x^7 + x^3 - x^6  …(*)
で、(*)^2 = -11

(*) は2項だけを除いて、偶数乗の項の符合が -、
奇数乗の項の符合が + という特殊事情があって簡単にできる。

(*) = (x - x^2 + x^3 - x^4 + x^5 - x^6 + x^7 - x^8 + x^9 - x^10) + 2(x^4 - x^7)
= (x - x^11)/(x + 1) + 2(x^4 - x^7)
= (x^6 - x^(-6))/(x^6 + x^(-6)) + 2(x^4 - x^(-4))

x = e^(6π/11) とすれば
(*) = i{tan(36π/11) + 4sin(24π/11)}
= i{tan(3π/11) + 4sin(2π/11)}

(*)^2 = -11 だったから
{tan(3π/11) + 4sin(2π/11)}^2 = 11

307 名前:132人目の素数さん:2006/02/14(火) 02:15:05
× x = e^(6π/11) とすれば
○ x = e^(6πi/11) とすれば

308 名前:132人目の素数さん:2006/02/14(火) 20:27:02
また間違えてた

× (x - x^g + x^(g^2) - x^(g^3) + … - x^(g^(p-1)))^2 = -p
○ (x - x^g + x^(g^2) - x^(g^3) + … - x^(g^(p-2)))^2 = -p

おまけ
(tan(π/7) - 4sin(2π/7))^2 = ?
(tan(π/19) - 4sin(6π/19) + 4sin(7π/19) + 4sin(8π/19))^2 = ?
(tan(π/23) - 4sin(2π/23) + 4sin(3π/23) - 4sin(5π/23) - 4sin(6π/23))^2 = ?

309 名前:132人目の素数さん:2006/02/24(金) 20:16:10
0が100個連続する円周率の並びはあるだろうか。

310 名前:132人目の素数さん:2006/02/25(土) 11:25:11



311 名前:132人目の素数さん:2006/02/25(土) 13:18:29
>>309
ないとは言えない。

312 名前:132人目の素数さん:2006/02/25(土) 17:49:03
>>309
円周率の並び……を定義しないとねぇ。

313 名前:132人目の素数さん:2006/02/25(土) 19:48:16
古い雑誌をあさっていたら昔の"C-Magazine"が出てきたので、そこにあった問題を転載w
----
1729は、二つの3乗数の和で2通りに表せる最小のものである。
1729 = 1^3 + 12^3 = 9^3 + 10^3
では、「二つの3乗数の和で3通りに表せる数の最小のもの」を見つけよ。
----
この問題のオリジナルは、ラマヌジャンの逸話が有名だけど、こういう問題を一般化してみると面白いかも。

314 名前:132人目の素数さん:2006/02/25(土) 21:03:11
>>313
Cマガジンって・・・・読んだ事無いけど、いかにもプログラミングって感じの本じゃないか?
虱潰し以外に、考える方法あるんかい?

315 名前:132人目の素数さん:2006/02/25(土) 22:15:21
うん。その通りかも。
一般化は無理なのかな…

316 名前:132人目の素数さん:2006/02/25(土) 22:58:59
オメーには教えてやらねーよ!

317 名前:132人目の素数さん:2006/02/25(土) 23:21:07
>>316
AA付けろよ

318 名前:132人目の素数さん:2006/02/25(土) 23:52:48
今日の京大経済の入試に面白いのがあった。

わかりやすく書きなおしてみると、こんなの。

「それぞれ偶数個の白玉と黒玉からなる数珠があるとき、白玉黒玉が
 どう並んでいようとも、どこか適当な2点で数珠をきれば白、黒
 ともに半々にできることを示せ。」

どう切り込むか悩むけど、わかってみれば簡単。中学生にも理解できる。

319 名前:132人目の素数さん:2006/02/26(日) 04:43:56
>>318
事実とすれば東大のパクーリだがそれはいかがなものか

320 名前:132人目の素数さん:2006/02/26(日) 05:06:24
R上で定義された、f(f(x))=-xとなる実関数f(x)を一例挙げよ

とかどうかな?

321 名前:132人目の素数さん:2006/02/26(日) 06:08:53
面白いかどうかは分かりませんけど、一つ考えて下さい

「直角三角形の斜辺の長さの2乗は他の二辺の2乗の和に等しい」

なぜ等しいのか証明せよ

322 名前:132人目の素数さん:2006/02/26(日) 08:25:07
>>321
正方形ABCDの辺ABの長さをa,bに分割する点をP、辺BCの長さをa,bに分割する点をQ
辺CDの長さををa,bに分割する点をR、辺DAの長さををa,bに分割する点をSとする。
ここで、四角形PQRSは正方形であるから、一辺の長さをcとする。

正方形ABCDの面積は(a+b)^2、一方正方形ABCDの面積は、
4つの直角三角形と、正方形PQRSの和でも書けるので、2ab+c^2
したがって、
(a+b)^2=2ab+c^2
展開して整理すると、
a^2+b^2=c^2

323 名前:132人目の素数さん:2006/02/26(日) 10:48:14
>>319
事実だけど、東大のパクリってどういうこと。
過去に同じ問題出たの?

324 名前:132人目の素数さん:2006/02/26(日) 11:00:42
>>318
サンドイッチが食べたくなったじゃないか。

325 名前:132人目の素数さん:2006/02/26(日) 11:26:26
>>323
同じではないがこんなのがある
http://hiw.oo.kawai-juku.ac.jp/nyushi/honshi/02/t01-21p/4.html

326 名前:132人目の素数さん:2006/02/26(日) 11:27:49
ふつーに
余弦定理より
a^2=b^2+c^2-2bcCos90じゃダメなん?

327 名前:132人目の素数さん:2006/02/26(日) 11:33:16
>326
△OAB∧O=90度なら
|OA↑-OB↑|^2=|OA↑|^+|OB↑|^2-2OA↑・OB↑
cos90゜=0より(ry

328 名前:328:2006/02/26(日) 11:36:45
3=log_{2}(8)


329 名前:329:2006/02/26(日) 11:37:15
3^2=9


330 名前:330:2006/02/26(日) 11:39:48
3-3=0

331 名前:331:2006/02/26(日) 11:42:02
3/31















3月31日

332 名前:132人目の素数さん:2006/02/26(日) 11:49:30
>>320
存在するとすれば、奇関数で微分不能であることはすぐにわかるね。
実数を二つのカテゴリーA,Bにわけて(ただし、a∈Aならば-a∈A
かつ、b∈Bならば-b∈B)、f(a)=b、f(b)= -a となるような関数を
定義すればいいんだな。たとえば、x=1を定義域から外していい
のなら、
x=0でf(x)=0
0<|x|<1でf(x)=1/x
|x|>1でf(x)=-1/x
ってのがある。




333 名前:132人目の素数さん:2006/02/26(日) 13:04:46
>>325
似て非なる問題だと思われ。
以下のようにして解いてみたけど、京大の問題と全然違うん
じゃないの?

両端が赤の弧は赤、青の弧は青、両端の色が異なる弧は黒に
塗る。すると、当然ながら、赤と青の弧が隣り合うことはな
く、必ず間に黒い弧がいくつか挟まるはず。
ところで、黒い弧が二つ並ぶ場合、端の玉は赤ー青ー赤か
青ー赤ー青なので、この二つの弧と真ん中の玉をとりはずして、
両端の同色の玉を融合させて輪を短くしても、残りの弧の色は
元と変わりはないはず。こうやって、隣り合う黒い弧を二本
セットではずして輪を小さくしていって、黒い弧が連続しない
ように変形できる。
さて、ここで、たとえば、赤い弧の連結部分をひとつ取り上
げると、その両端には黒い弧が必ず存在し、そのさらに隣に
は青い弧がつながるはず。これらをとりはずして、切れ目に
現れた青い弧同士を繋げてやっても、残りの弧の色に変わり
はない。こうやって、赤い弧の連結部分を同様にして外して
いけば、いずれすべての赤い弧は取り外せる。残りはすべて
青か黒となるはずだが、青と青の間に黒い弧が一つだけはい
ることはできないはずなので、すべて青の弧になる。つまり、
黒い弧はすべて2個ずつセットではずしていくことができた
ことになる。よって、両端の色が異なる弧は偶数個あったと
いうことが示せた。

334 名前:132人目の素数さん:2006/02/26(日) 13:56:51
>>313
C-Magazineは今4月号でめでたく休刊です。

335 名前:132人目の素数さん:2006/02/26(日) 14:10:36
>>334
な、なぬ〜〜〜〜!
結構前に愛読してる先輩から借りて読んだんが、よさげな本だと思った。
休刊(≒廃刊)なのか…。

336 名前:320:2006/02/26(日) 15:11:09
>>332
それ友人が「これは惜しいな」って言ってきたやつと同じだw

結局一日後にその友人が発見しました。

337 名前:132人目の素数さん:2006/02/26(日) 15:23:32
>>320
x の小数部分 {x} を
{x} = x - [x]  ([ ] はガウス記号)
と定義して

f(x) = 0  (if x=0)
f(x) = x-(1/2)  (if x>0 ∧ {-x}<1/2)
f(x) = -x-(1/2)  (if x>0 ∧ {-x}≧1/2)
f(x) = x+(1/2)  (if x<0 ∧ {x}<1/2)
f(x) = -x+(1/2)  (if x<0 ∧ {x}≧1/2)

338 名前:132人目の素数さん:2006/02/26(日) 21:40:47
>>318
ゴリ押し!

黒,白の数珠がそれぞれ2n個ずつあったとする。白の数珠を1つ選び、その数珠の番号を[0]とする。
この数珠から、時計周りに順番に数珠に番号[1],[2],…,[4n−1]をつけていく。ただし、[m]=m+4nZ
である。数珠の色が黒のとき、その数珠に重さ1を与える。白のときは重さ0を与える。[i]番目の数珠の
重さをf([i])で表すことにする。Si=Σ[k=i〜i+2n−1]f([i]) (0≦i≦4n−1)とおくとき、あるiが存在
してSi=nとなっていることを示せばよい。まず、S(i+1)−Si=f([2n+i])−f([i])=0 or 1 or −1である。…*
次に、M=max[0≦i≦4n−1]Si,m=min[0≦i≦4n−1]Siとおくと、Σ[i=0〜4n−1]Si=2nΣ[i=0〜4n−1]f([i])=4n^2
であるからM≧n,m≦nである。M=n or m=nのときは成立するから、M>n,m<nとしてよい。M=Sx,m=Syとして、
(x<yのとき)*より、x<i<yなるiでSi=nを満たすものが存在する。よって成り立つ。(x>yのとき)同様にして成り立つ。

339 名前:132人目の素数さん:2006/02/26(日) 22:04:39
>>338
そそ、そんな感じ。
重さ1とかいわんでも、起点となる玉から右回りにn個分のうち、
白玉か黒玉かどっちかの個数をf(i)として考えればよろし。どちらか
が半数なら、反対の色も半数だからね。f(i)-f(i+1)=0または±1と、
f(i)の最大最小ということね。


340 名前:132人目の素数さん:2006/02/26(日) 22:58:58
>>338
さっきはちょっと時間がなかったんで、補足します。
まず、白玉と黒玉は個数が異なってもいいんすよ。
白を2kとすると、黒は2n-2kですわ。合わせて2n個ね。

で、どれかの玉に起点を定めて、i番目の玉から右周りに
n個分のうち白玉の数をf(i)とするです。すると、f(i)とf(i+n)
はちょうど数珠を二つに割った片割れ同士に対応するので、
f(i)+f(i+n)=2kとなります。あと、f(i+1)は端の玉の出入り
だけ考えれば、f(i)からたかだか1増減するだけ。ちゅまり、
自然数としては連続的に変化するわけですね。ここが味噌か。
さらに、f(i)は周期2nの周期関数ですので、f(i+2n)=f(i)ね
(iは任意の自然数でよし)。
で、f(i)の値の最大値をM、最小値をmとしやす。んで、
そのときのiの値をそれぞれ、imax,iminとします。
すると、f(imax)+f(imax+n)=2k より、f(imax+n)=2k-M
ですが、この値はMを超えないので、2k-M≦Mとなり、
k≦Mが成り立ちます。同様にして、m≦kも成立。
f(i)の自然数としての連続性と最大、最小がkを挟むこと
から、f(i)=kとなるiが存在することは明らか。




341 名前:132人目の素数さん:2006/02/26(日) 23:12:02
>>337
うへ。なかなか、ややこしいすね。
確認する気にならんがな。

342 名前:132人目の素数さん:2006/02/26(日) 23:19:13
東大文系の過去問もほぼ同じやり方で解けるね

343 名前:340:2006/02/27(月) 00:32:17
すまん、最大値だの、周期関数だの、過剰な一般化してたわ。

さっき予備校の模範解答みたら、
f(1)=kならそれが求める分割。
f(1)>kなら、f(1)+f(1+n)=2kより、f(1+n)<kで、その間にf(i)=kとなるiが存在する。
f(1)<kなら、f(1+n)>kで、同様。
と、簡単にやってあった。

344 名前:132人目の素数さん:2006/02/27(月) 03:49:59
別スレからサルベージ

分からない問題はここに書いてね232
http://science4.2ch.net/test/read.cgi/math/1139679770/

206 名前:132人目の素数さん[sage] 投稿日:2006/02/13(月) 00:12:11
知人に聞かれたがマジで分からん・・・。

・1時に1秒、2時に2秒…12時に12秒時報が鳴る時計がある
・時計は1秒単位で任意に時刻あわせできる
・24時間時報が鳴り続けるには、最低何個の時計が必要か?


1個の時計だと、24 時間のうち、1+2+ ... + 12 秒時報が鳴っているが、
鳴っていない時間に、時刻をずらした時計を追加していくと
鳴りっぱなしを実現できるわけだが・・・。

だれか上手い考え方プリーズ。

全然分からなかったので、
コンピュータで計算して予想は801個。でも最少性の証明はわからん。


583 名前:132人目の素数さん[sage] 投稿日:2006/02/16(木) 00:57:51
>>435

レスありがとう。12時間制の時計ということです。

1時間ごとに時報がなることを考えると、
たぶん1時間鳴らし続けられれば自動的に24時間なると思う。

345 名前:132人目の素数さん:2006/02/27(月) 10:12:32
時計二個あれば交互に時刻を合わせて鳴り続けさせられる

346 名前:132人目の素数さん:2006/02/27(月) 11:37:10
1個でええんじゃないの。12時の時報がなり始めたら、時計を12時の数秒前に
あわせ直す。これを延々続ける。

347 名前:132人目の素数さん:2006/02/27(月) 12:08:39
最初に自由にセットしてあとは放置ってことでそ?

348 名前:132人目の素数さん:2006/02/27(月) 12:15:52
問題作ってみた。

(連続とは限らない)関数f:R→R は、任意の実数x,yに対してf(x+y)=f(x)cosy+f(y)cosxを
満たしている。また、原点で微分可能とする。f(x)を求めよ。

349 名前:132人目の素数さん:2006/02/27(月) 12:27:35
>>348
とりあえずf(x)=sinxは条件を満たす

350 名前:132人目の素数さん:2006/02/27(月) 12:29:22
間違えた
とりあえずf(x)=Csinxは条件を満たす

351 名前:132人目の素数さん:2006/02/28(火) 12:04:43
f(x)=f(x-π/2+π/2)=f(x-π/2)cos(π/2)+f(π/2)cos(x-π/2)=f(π/2)sin x

352 名前:132人目の素数さん:2006/03/01(水) 18:29:28
ax^3+bx^2+cx+d=0をxについて解け

353 名前:132人目の素数さん:2006/03/01(水) 18:53:47
おもしろーい

354 名前:132人目の素数さん:2006/03/02(木) 11:42:06
スレ違い・・・もしくは板違いであればすいません。
どこに質問していいやら分からなかったので、一番趣旨に近いスレを選びました;

よくある問題なんですが、3人が1000円づつ払って、店員が200円抜いて300円返したら、100円合わない。
そんな面白問題を教えて欲しいです。

どうぞよろしくお願いします^^;

355 名前:132人目の素数さん:2006/03/02(木) 13:28:36
>>354
http://www.geocities.co.jp/CollegeLife-Club/7442/math/#30$

356 名前:132人目の素数さん:2006/03/04(土) 23:53:00
実数Rの部分集合系(It |t∈∧)は、各元が全て可算集合とする。
(1)∪[t∈∧]Itは可算集合になると言えるか。
(2)a,b∈∧ならばIa⊂IbまたはIa⊃Ibが成り立つとするとき、∪[t∈∧]Itは可算集合になると言えるか。

357 名前:132人目の素数さん:2006/03/05(日) 07:23:38
(1)は、Λが非可算で各Itが排反ならもうダメなんじゃないの?
例えば Λ=R、It={t}とか。

358 名前:132人目の素数さん:2006/03/05(日) 09:20:07
(2)がちょっと大変だよね

359 名前:132人目の素数さん:2006/03/05(日) 21:23:27
(2) 最小の非可算順序数の狭義の始切片はすべて可算集合だということを使えばよい。

360 名前:132人目の素数さん:2006/03/06(月) 16:11:08
age

361 名前:132人目の素数さん:2006/03/07(火) 00:33:19
1*2*2の直方体ブロック6つと1*1*1の立方体ブロック3つで
3*3*3の立方体ブロックを組み立てる方法を2通り示す。

ただし回転と鏡映で移りあう物は同じ方法とみなす。

362 名前:132人目の素数さん:2006/03/07(火) 16:36:48
C[m,n]を二項係数とする。
C[2m,m]*C[2n,n]/C[m+n,m] が整数であることを、組合せの数を考えることによって示せ!
できるかな〜?

363 名前:132人目の素数さん:2006/03/07(火) 17:16:39
>>362
追加 : a[m,n] = C[2m,m]*C[2n,n]/C[m+n,m] とおくとき、
a[m+1,n]+a[m,n+]=4a[m,n] を組合せの数を考えることによって示せ。

364 名前:132人目の素数さん:2006/03/08(水) 00:59:22
>>362-363
計算では示せるが、場合の数を考えると、どんな状況を考えたらよいかワカンネ!

365 名前:132人目の素数さん:2006/03/08(水) 09:52:47
>>356
(2)Rの可算部分集合全体をVとすると、包含関係⊂に関して(V,⊂)は半順序集合になる。
もし∪[t∈∧]Itが必ず可算集合になっているとすると、これは「Vの任意の全順序部分集合が
上界を持つ」ということを意味するから、Vには極大元αが存在することになる。αは可算集合だから、
αに含まれない実数xが存在する。このようなxを1つ選びβ=α∪{x}とおくと、βもまた可算集合で、
しかもαはβの真部分集合となるので、αが極大元であることに矛盾する。従って∪[t∈∧]Itが非可算と
なる場合が存在することになる。

366 名前:132人目の素数さん:2006/03/08(水) 10:33:01
>>365
>>359で回答済み

367 名前:132人目の素数さん:2006/03/08(水) 13:30:42
数オリから作った問題。

上下左右の各方向に無限に敷き詰められたマス目がある。これらのマス目のうちいくつかを
黒で塗っていく。ただし、次の条件1,2をともに満たすように塗る。
条件1:黒マスに隣接する黒マスの個数は奇数である。
条件2:ちょうどnマスを黒で塗る。
(1)条件1,2をともに満たす塗り方が存在するための必要十分条件は、nが偶数であることを示せ。
(2)n≧6のとき、隣接する黒マスの個数が1つである黒マスは4個以上存在することを示せ。
(3)n=2mとおく。隣接する黒マスの個数が1つである黒マスがちょうどk個である塗り方が、
k=m+1,m+2,…,2mに対して必ず存在することを示せ。

368 名前:132人目の素数さん:2006/03/09(木) 01:13:48
(1)互いにくっついた■のグループを「島」と呼ぶことにする。
nが偶数の時は2個ずつの島に分ければいいから可能。
nが奇数の時に不可能であることを示す。

まず、どんな島であれ、他の■と接している辺の数(0〜4)を、
各■ごとに数えて総計すると偶数になることに注意しておく。
共有辺はちょうど2度ずつカウントされるのだから当たり前。

さて、nが奇数の時、少なくとも1つは奇数個の■から成る島がある。
その島の全ての■が、奇数個の■と接しているとすると、
接している辺の総計は奇数。これは上で述べたことに反する。

369 名前:132人目の素数さん:2006/03/09(木) 04:10:58
A(先手),B(後手)の2人が次のようなゲームを行なう。
交互にサイコロを振り、出目に応じて点数を得て、
先に決められた得点(ゴール点と呼ぶことにする)
に達した方を勝ちとする。
サイコロを1回振るごとに、出目が6のときは0点、
その他の場合は出目の数をそのまま得点とする。
また、ゴール点以上の点数に達すれば、
ちょうどでなくとも勝ちとする。
2人のゴール点を同じとすると先手が有利となるので、
確率的に公平となるように、それぞれのゴール点を
定めたい。
そのようなゴール点の組合せは存在するか。

370 名前:132人目の素数さん:2006/03/09(木) 13:25:55
>>369
とりあえず……
さいころを一回振ったときの平均得点は2.5点だから、先手が2.5点分後手より余計に得点しなければならないとすると、先手と後手が入れ替わるのと同じ効果。
ということは、公平にするためのゴール点の差は0と2.5の間の値。かな?

371 名前:132人目の素数さん:2006/03/10(金) 12:26:29
age

372 名前:293の証明:2006/03/12(日) 13:56:45
z=exp(πi/11)とする。
z^11= exp(πi)=-1
(1+z)(1-z+z^2-z^3+z^4-z^5+z^6-z^7+z^8-z^9+z^10)=1+z^11 = 0
1-z+z^2-z^3+z^4-z^5+z^6-z^7+z^8-z^9+z^10 = 0
(1-z^5)(z-z^2+z^3-z^4+z^5)=1

オイラーの公式により、z^k=cos(kπ/11)+isin(kπ/11)

よって
sin(kπ/11)=Im(z^k) = i(z^-k - z^k)/2
cos(kπ/11)=Re(z^k) = (z^-k +z^k)/2
tan(kπ/11)=sin(kπ/11)/cos(kπ/11) = i(z^-k - z^k)/(z^-k+z^k)=i(1-z^(2k))/(1+z^(2k))
tan(3π/11)=i(1-z^6)/(1+z^6)=i(z^5-z^11)/(z^5+z^11)=-i(1+z^5)/(1-z^5)= -i(2/(1-z^5)-1)
=-i(2(z-z^2+z^3-z^4+z^5)-1)=i(1-2z+2z^2-2z^3+2z^4-2z^5)
4sin(2π/11)=2i(z^-2-z^2)=-i(2z^2+2z^9)
tan(3π/11)+4sin(2π/11)=i(1-2z-2z^3+2z^4-2z^5-2z^9)

373 名前:293の証明の続き:2006/03/12(日) 14:10:37
(tan(3π/11)+4sin(2π/11))^2=-4(1/2-z-z^3+z^4-z^5-z^9)^2
=-4((1/2)^2+z^2+z^6+z^8+z^10+z^18-z-z^3+z^4-z^5-z^9
+2zz^3-2zz^4+2zz^5+2zz^9-2z^3z^4+2z^3z^5+2z^3z^9-2z^4z^5-2z^4z^9+2z^5z^9)
=-4(1/4)+z^2+z^6+z^8+z^10+z^18-z-z^3+z^4-z^5-z^9
+2z^4-2z^5+2z^6+2z^10-2z^7+2z^8+2z^12-2z^9-2z^13+2z^14)
=-4(1/4-z+z^2-z^3+3z^4-3z^5+3z^6-2z^7+3z^8-3z^9+3z^10+2z^12-2z^13+2z^12+z^18)
=-4(1/4-z+z^2-z^3+3z^4-3z^5+3z^6-2z^7+3z^8-3z^9+3z^10-2z+2z^2-2z^3-z^7)
=-4(1/4-3+3(1-z+z^2-z^3+z^4-z^5+z^6-z^7+z^8-z^9+z^10))
=-4(1/4-3)
=11
qed

374 名前:132人目の素数さん:2006/03/12(日) 15:28:19
哲学者ホメロスが、パティーとセルマという2人の優秀な数学者に異なる整数を与え、2人に以下のことを伝えた。

(i) パティーに与えられた数は、2つのある自然数aとbの積abである。
(ii) セルマに与えられた数は、2つのある自然数aとbの和a+bである。
(iii) b>a>1である。

aとbがそれぞれいくつなのかは2人に教えられておらず、かつ、お互いに相手の数がいくつなのかも教えられていない。
その後、以下のような会話がなされた。

パティー:私にはaとbがいくつなのかわかりません。
セルマ:でしょうね。あなたにはわかるはずがないと思いましたよ。
パティー:なるほど。そうですか。ではaとbがいくつなのかわかりましたよ。
セルマ:そうですか。それなら私もaとbがいくつなのかわかりました。

さて、いま第三者であるあなただけが、「aとbはともに20以下である」ことを知っているとするとき、aとbを決定せよ。


375 名前:132人目の素数さん:2006/03/12(日) 15:55:36
はいはいオマンコオマンコ

376 名前:132人目の素数さん:2006/03/12(日) 15:56:48
正方形を作図するとき、コンパスは最低何回必要になるか。

※1 コンパスの軸をずらす、又は描く円の半径の大きさを変える等するまでを1回とする。
※2 点を打って「1辺0cmの正方形」というのは面白くない。

↑の問題、0回でできたました。

377 名前:132人目の素数さん:2006/03/12(日) 16:57:11
正方形作図でコンパスは何回必要?の問
0回ですね!
仮に10cmとした直線ABをとる。次はABから30°程開いた10cmの直線ACを
とる。これで丁度Vの字型となる、そして三角形ABCは二等辺三角形になる。
次BCの中点DをとりADを延長して通る直線AFを引く。DからA方向にBDと同じ長さを
とってEとし、DからF方向にBDと同じ長さをとってGとする。EBGCは正方形となったでしょ!

378 名前:132人目の素数さん:2006/03/12(日) 17:18:21
>10cmの直線ACをとる
>BCの中点Dをとり
>BDと同じ長さをとって
>DからF方向にBDと同じ長さをとって

379 名前:132人目の素数さん:2006/03/12(日) 17:35:46
関連記事
ttp://messages.yahoo.co.jp/bbs?.mm=GN&action=m&board=1835554&tid=bfta4nbba83x&sid=1835554&mid=1&type=date&first=1
の340あたりから
作図可能な図形はコンパス1回だけで作図可能
ということの証明が書かれているっぽい
んだけどわかりにくい

380 名前:132人目の素数さん:2006/03/12(日) 21:25:07
100を10分割した行列をuとする
すべてのnにいついてun+1->unにシフトさせる行列をSとする、u1->unにする。
uのmod2をとる操作をMとする
AをAu=.5(E+M)Su-.5(E+M)u とする
かならずA^mu=u0=(10)になるか? mはuによる



381 名前:132人目の素数さん:2006/03/12(日) 23:18:32
大学時代に教授に教えてもらった問題だが。

ネズミが一匹、無限の広さをもつ碁盤の上に居る。
プレーヤーは一ターンの間にネズミの居ない任意の座標にブロックを一つおくことができる。
ネズミは一ターンの間に上下左右のいずれかの方向に一歩動くことができる。
ただし、ブロックのある座標には移動できない。

ネズミの四方がブロックで囲まれ、一歩も動けない状態になればプレーヤーの勝ち。
ネズミがいつまでも逃げることができればネズミの勝ち。

このゲームはどちらが勝つか。


382 名前:132人目の素数さん:2006/03/13(月) 00:06:10
>>381
どっちが先攻?

383 名前:382:2006/03/13(月) 00:07:02
ああ、どっちでもいいか。ごめん。

384 名前:132人目の素数さん:2006/03/13(月) 00:09:41
碁でいうところのシチョウになって逃げられるんじゃないの?


385 名前:132人目の素数さん:2006/03/13(月) 00:13:40
>>381
面白そうな問題やね。
ちょっと試してみるとどうやってもプレイヤーが勝ちそう。
もしプレイヤーに必勝法があるなら、最初の何個かをどんなに適当に置いても
途中から必勝法を適用できるのは明らかだから、置き間違えるということが
けっして無いのね。

386 名前:132人目の素数さん:2006/03/13(月) 00:15:12
ん?ネズミの勝ちでないの?

387 名前:385:2006/03/13(月) 00:17:43
わからん。もうちょっと考えてみる。

388 名前:132人目の素数さん:2006/03/13(月) 00:22:57
>>387
碁は知ってる?
シチョウってのがあるんだけど、境界がなきゃその形でどんどん伸びて
いけそうな気がする。

389 名前:132人目の素数さん:2006/03/13(月) 00:24:40
四角を作る前に壁までねずみがこれるかどうか

390 名前:385:2006/03/13(月) 00:28:05
碁はだいたいわかるけど、詳しくはないよ。
シチョウで無限に伸びていけるというのが確かなら、
それがこの問題に適用できるかどうかやね。
ちょっと調べてみるか。

391 名前:132人目の素数さん:2006/03/13(月) 00:39:08
「シチョウ」と違って遠くで予め待ち伏せするなんてことも出来るが

392 名前:132人目の素数さん:2006/03/13(月) 00:44:39
問題はネズミが到着するのが先か、逃げ道防ぐのが先か。

393 名前:385:2006/03/13(月) 00:45:54
そうやね。
いろいろ試したけど、ネズミのマスの隣りじゃなくて
斜め隣りのマスか1個離して打っていけば囲めるみたい。
証明できないけど。

394 名前:Sean ◆t.0oQdlc8Y :2006/03/13(月) 00:46:15
・・・ちょっと考えてるとプレイヤーが普通に勝つ気がしてきた。
でもよくわかんない。よって、実際に勝負して決めよう
●=プレーヤーのブロック、○=ネズミ
 | | | | | | | | | | 
ー+ー+ー+ー+ー+ー+ー+ー+ー+ー+ー
 | | | | | | | | | | 
ー+ー+ー+ー+ー+ー+ー+ー+ー+ー+ー
 | | | | | | | | | | 
ー+ー+ー+ー+ー+ー+ー+ー+ー+ー+ー
 | | | | | | | | | | 
ー+ー+ー+ー+ー+ー+ー+ー+ー+ー+ー
 | | | | | | | | | | 
ー+ー+ー+ー+ー●ー○ー+ー+ー+ー+ー
 | | | | | | | | | | 
ー+ー+ー+ー+ー+ー+ー+ー+ー+ー+ー
 | | | | | | | | | | 
ー+ー+ー+ー+ー+ー+ー+ー+ー+ー+ー
 | | | | | | | | | | 
ー+ー+ー+ー+ー+ー+ー+ー+ー+ー+ー
 | | | | | | | | | | 
ー+ー+ー+ー+ー+ー+ー+ー+ー+ー+ー
 | | | | | | | | | | 
ー+ー+ー+ー+ー+ー+ー+ー+ー+ー+ー
 | | | | | | | | | | 
さぁ、かかってこい(どーん)

395 名前:Sean ◆t.0oQdlc8Y :2006/03/13(月) 00:48:31
あーあ、なれないことやるからずれた・・・
もう、試合放棄(早

396 名前:385:2006/03/13(月) 00:53:41
1000までいったらネズミの勝ちとか?

397 名前:132人目の素数さん:2006/03/13(月) 01:05:41
>>394
だから、それで普通にシチョウになるでしょ。待ち伏せの石打ったら、
その瞬間に方向転換されちゃうよ。

398 名前:Sean ◆t.0oQdlc8Y :2006/03/13(月) 01:06:26
とりあえず、俺が明日(マジで)
囲碁全国レベルの友達に聞いてみるから、
皆さんは安心して寝てください。


399 名前:Sean ◆t.0oQdlc8Y :2006/03/13(月) 01:08:20
あ、厳密には今日だった。
無駄レス失礼

400 名前:385:2006/03/13(月) 01:10:45
>>397
でもさらにどんどん待ち伏せしていけば
すぐに行き場が無くなってしまうと思う。
いろいろ試してみた実感。

401 名前:132人目の素数さん:2006/03/13(月) 01:33:31
囲碁碁会所3段の俺が答えよう。
ずばりプレイヤーの勝ち。

シチョウ云々より、はじめから遠巻きにネズミを追い込んでいけば勝てるはず。
ただ、最短何手で詰みになるのかはこりゃまったくわからん


402 名前:132人目の素数さん:2006/03/13(月) 01:38:17
上でシチョウ云々言ってる人はシチョウって本当にどういうのか知って書いてるのかな

囲碁よりも連珠やってる人のほうがまだ信用できるようなw

403 名前:132人目の素数さん:2006/03/13(月) 01:41:00
ネズミがシチョウに取られないように打っていけば
プレイヤーがシチョウに取るなんてことは神業だと思う。
というかムリだとw

404 名前:132人目の素数さん:2006/03/13(月) 01:44:44
こんな感じで捕まりそう。
碁はよく知らないんだが、斜めに待ち伏せるのが効果的っぽい。
☆鼠
●石

  1置く    2逃げる    3置く    4逃げる
┼┼┼┼┼/┼┼┼┼┼/┼┼┼┼┼/┼┼┼┼┼/
┼┼●┼┼/┼┼●┼┼/┼●┼┼┼/┼●┼┼┼/
┼┼┼☆┼/┼┼┼┼┼/┼┼┼┼┼/┼┼┼┼┼/
┼┼┼┼┼/┼┼┼☆┼/┼┼☆┼┼/┼☆┼┼┼/
┼┼┼┼┼/┼┼┼┼┼/┼┼┼●┼/┼┼┼●┼/
┼┼┼┼┼/┼┼┼┼┼/┼┼┼┼┼/┼┼┼┼┼/

   5置く      6逃げる    7置く
┼┼┼┼┼┼/┼┼┼┼┼┼/┼┼┼┼┼┼/
┼┼●┼┼┼/┼┼●┼┼┼/┼┼●┼┼┼/
┼┼┼┼┼┼/┼┼┼┼┼┼/┼┼┼┼┼┼/
┼┼☆┼┼┼/┼☆┼┼┼┼/●☆┼┼┼┼/
┼●┼┼●┼/┼●┼┼●┼/┼●┼┼●┼/
┼┼┼┼┼┼/┼┼┼┼┼┼/┼┼┼┼┼┼/

405 名前:385:2006/03/13(月) 01:53:19
┼●┼┼┼ こうなった時点でプレイヤーの勝ち
┼○┼┼┼ ってことがわかった。
┼┼●┼┼

┼●┼┼┼ 左に打って、
●○┼┼┼
┼┼●┼┼

┼●┼┼┼ どっちに逃げても同じだから
●┼○┼┼ 右に逃げるとして、
┼┼●┼┼

┼●┼●┼ 右上に打って、
●┼○┼┼
┼┼●┼┼

┼●┼●┼ 上に逃げても塞がれるだけだから
●┼┼○┼ 右に逃げるとして、
┼┼●┼┼

┼●┼●┼ 右に打って、
●┼┼○●
┼┼●┼┼

┼●┼●┼ って感じで結局こうなって、
●┼┼┼● もはや逃げられず。
┼┼●┼┼
●○┼┼●
┼●┼●┼

というわけで最初の図の状態にもっていければオッケーかな?


406 名前:132人目の素数さん:2006/03/13(月) 01:56:04
他にもそういう型がありそうだな。

407 名前:132人目の素数さん:2006/03/13(月) 02:09:12
最初の図はプレーヤーの手番だね

408 名前:385:2006/03/13(月) 02:18:28
●○┼┼ 左隣りに打って
┼┼┼┼ ネズミが右に逃げると、

●┼○● こう打つことができ、
┼┼┼┼ 上か下に逃げなければならない。

●┼┼┼ だから最初から
┼○┼┼ こう逃げるとしてもよい。

●┼┼┼ こう打つ。
┼○┼┼ ここで左に逃げると>>405の最初の図でダメ。
┼●┼┼ 上に逃げた場合はそのすぐ上に打って(省略)。

●┼┼┼ 右に逃げたらその右に打ち、
┼┼○● この後上に逃げ、そのすぐ上に打つ。
┼●┼┼

てな感じで、必ず>>405の最初の図になるね。
これで一応証明終わり。
思ったよりあっさりしてるかな。
えっと、合ってるよね?

>>407
うん。

409 名前:385:2006/03/13(月) 02:35:32
一応、補足。
>>405の必勝形は、図にあるブロック以外に
どこにいくらブロックがあってもけっして
プレイヤー側の邪魔にはならないということを
暗黙に用いてます。

410 名前:132人目の素数さん:2006/03/13(月) 02:36:52
完璧っぽい。

411 名前:385:2006/03/13(月) 02:43:54
>>410
どうもです。
最短手数は何手かな。
これは全手順を調べるしかないか…。
もう寝る。

412 名前:132人目の素数さん:2006/03/13(月) 02:56:52
ネズミが一回に2マス動けるとして、同じ問題やってみるか。

413 名前:132人目の素数さん:2006/03/13(月) 09:30:00
ゲタ。


414 名前:132人目の素数さん:2006/03/13(月) 11:45:14
>>404
それが最短だと思います

415 名前:132人目の素数さん:2006/03/13(月) 17:24:14
2マス進めるかどうかはともかく、面白い問題なんで拡張(一般化)してみたいなあ。

416 名前:132人目の素数さん:2006/03/13(月) 20:37:47
各辺の長さがnの正三角形を考える。(nは自然数)
この内部及び辺上に点をn^2+1個とる。
このとき、少なくとも一組、二点間の距離が1以下になるような点の組が存在することを示せ。


面白くも難しくもない・・・orz

417 名前:381:2006/03/13(月) 20:49:42
おおッ!たった一日で解けてる!
いつもながらこのスレのクオリティの高さには驚かされます。



418 名前:132人目の素数さん:2006/03/13(月) 21:04:50
囲碁の問題だったのか・・・定石読んでりゃできるんじゃなかったの?

419 名前:381:2006/03/13(月) 21:13:10
>>416
鳩ノ巣原理?


420 名前:Sean ◆t.0oQdlc8Y :2006/03/13(月) 21:22:01
なんだ、せっかく解いたのにもう解かれてたか・・・
じゃあ、新しい問題を。
1,2,3,5,7,11,15,22,30‥‥‥

さて、この数列は一体どんなものでしょう。
これ出来たら天才だ。間違いなく。
知ってる人もいるかもしれないが・・・
普通の数列として考えたら出来ません。死にます。
だからといって、なぞなぞではありません。
ちゃんと数学に関係する数列ではあるんだけど・・・
まぁ、思いついても確かめるのが面倒です(これはヒントになるかな?)。

421 名前:132人目の素数さん:2006/03/13(月) 21:31:29
素数の音楽
聴きたい…

422 名前:132人目の素数さん:2006/03/13(月) 21:35:52
>>420
この数列を[a_n]としたときに
一般項a_nがnの(9次以上の)多項式で表されるとかいうんじゃないだろうな

423 名前:132人目の素数さん:2006/03/13(月) 21:53:13
>>420
どこかで見たことあるような気がするんだが…
思い出せん。


424 名前:132人目の素数さん:2006/03/13(月) 22:32:05
10人が円卓を囲んでいる。各人の前に何枚かずつカードが置かれていて、合計は100枚である。
合図があると、各人は一斉に自分の前のカードの中から、もしそれが偶数枚だったらその半分の
カードを、奇数枚の時は1つ足してその半分の枚数のカードを、右側の人に渡す。
このカードの移動を繰り返すと、いつかみんなの前にはちょうど10枚ずつのカードが置かれている
状態になることを証明せよ。

425 名前:132人目の素数さん:2006/03/13(月) 22:48:38
>>420
ttp://www.research.att.com/~njas/sequences/

426 名前:Sean ◆t.0oQdlc8Y :2006/03/13(月) 22:52:24
>>421
そう、「素数の音楽」からの問題です。
素数は関係ないけどね。
>>424
どっかで・・てか多分この数学版のどこかで見た気がする・・
だが残念ながら、余白が足りないためここにその証明を記す事は出来ない。

427 名前:132人目の素数さん:2006/03/14(火) 01:30:53
>>424
10人のカードの枚数が全て同じではないとし、
その中の最大値をMとする。
もしMが偶数でM枚のカードを持つどの人についても
左隣の人がM枚またはM-1枚のカードを持っているとすると、
M-1枚のカードを持っている人の中に
その左隣の人のカードがM-2枚以下であるような人がいる。
この場合、カードの交換を行なうと、
M枚のカードを持つ人の人数は変わらず、
M-1枚のカードを持つ人の数が減る。
またそうでない場合、M枚のカードを持つ人の数が減る。
このことから、カードの交換を繰り返すと
全員の枚数が同じになるといえる。

428 名前:132人目の素数さん:2006/03/14(火) 02:55:29
今年の京大後期6番

tan1゚は有理数か?

429 名前:132人目の素数さん:2006/03/14(火) 02:59:43
>>381
ネズミがブロックを押せるとしたらどうかな。
1個しか押せない場合と、2個以上でも押せる場合が考えられるけど。

430 名前:132人目の素数さん:2006/03/14(火) 03:00:56
>>429
と思ったけど2個以上押せるとしたら囲まれても押せるじゃん。

431 名前:132人目の素数さん:2006/03/14(火) 06:20:58
ネズミを2匹にしたらどうなる?
さらに、1匹を踏み台にしてもう1匹がブロックを越えられるようにしたりとか
バリエーションいろいろ考えられるな…

432 名前:132人目の素数さん:2006/03/14(火) 06:27:23
ブロックの数が限られていて、手持ちが無くなったら盤上のどれかを動かすとすると、
6個あれば十分だ>>381
意外に少なくて済むな

433 名前:132人目の素数さん:2006/03/14(火) 10:18:44
>>427
>もしMが偶数でM枚のカードを持つどの人についても
>左隣の人がM枚またはM-1枚のカードを持っているとすると、
>M-1枚のカードを持っている人の中に
>その左隣の人のカードがM-2枚以下であるような人がいる。
>この場合、カードの交換を行なうと、
>M枚のカードを持つ人の人数は変わらず、
>M-1枚のカードを持つ人の数が減る。

例えば・・・,10,10,7,11,12,10,10・・・のときは?
>>428
帰納法使って有理数とするとtan60°が有理数になるから矛盾で終わりじゃん。
こんな簡単な問題出すなよ京大・・・OTL

434 名前:中川秀泰 ◆F.wl5K6LPU :2006/03/14(火) 11:21:56
>>433
途中で現れる tan の加法定理の分母が全部0でない事を言わないと正解にならない。

435 名前:132人目の素数さん:2006/03/14(火) 11:42:31
>>434
その必要はないだろw
tanの加法定理の証明をやってみれば分かるが、tan(a+b)°={tana°+tanb°}/{1−tana°tanb°}が
成り立つ必要十分条件は、a≠90(2k−1)かつb≠90(2k−1)かつa+b≠90(2k−1)…* となる。
これは、「tanx°」が有理数か否かとは関係なく、tanの中身の「x」のみによって定まる。だから、tan1°を
有理数と仮定しようがしまいが、それでtanの加法定理が使えなくなることは無い。で、じゃあ*の証明を書く
必要はあるかと言うと、そんなのは学校で習ったと見なしてよいはず。

436 名前:中川醜態:2006/03/14(火) 12:18:55
>>435
>tan(a+b)°={tana°+tanb°}/{1−tana°tanb°}が
>成り立つ必要十分条件は、a≠90(2k−1)かつb≠90(2k−1)かつa+b≠90(2k−1)

±∞とその「性質」さえ導入すれば、いいだけでしょ。
そうすれば、加法定理は常に成立する。

437 名前:132人目の素数さん:2006/03/14(火) 12:33:25
>>436
±∞の性質を加えても「∞−∞」の計算はできない。よって、∞を導入しても
0=tan(90−90)=(∞−∞)/(1+∞)=不定 となって駄目。

438 名前:132人目の素数さん:2006/03/14(火) 13:24:57
>>436
むちゃくちゃ言うな

439 名前:132人目の素数さん:2006/03/14(火) 14:08:20
(x-x)/(1+xx)=0.


440 名前:132人目の素数さん:2006/03/14(火) 21:39:30
tan60°じゃなくて、tan30°が無理数を使えばええんでないの?
数学的帰納法だから、aとbはそれぞれ 1とk (kは自然数)に置き換える
そしたら、k<30で、0<tan1°≦tan k°<tan30°=1/√3なんだから、
0<tan1°tank°<1/3で、分母の 1- tan1°tank°は0にはならん。

441 名前:427:2006/03/14(火) 21:44:38
>>433
全然証明できてなかった・・・or2
9,11,10,10,10,10,10,10,10,10から始めると8回までは11がずれていくだけで
枚数の組合せは変わらないことに気付いた。
むずいな・・・

442 名前:132人目の素数さん:2006/03/14(火) 21:45:55
60度のときと何が違うのかよくわからんが

443 名前:Sean ◆t.0oQdlc8Y :2006/03/14(火) 22:23:40
>>441
10人が持っているカードの枚数の最大値をM,最小値をmとする。
ここで、M≧10、m≦10であることが言える。
また、M枚のカードを持っている人が一人だとすると、
次の交換で必ずMは減少する。また、M枚のカードを持っている人が
N人(N≠10)で、その人たちがL人(2≦L≦N)連続している場合、
L回の交換でMは減少する。
同様にして、mは増加する。
よって、全員が10枚にならない限り、カードの枚数は変動し続けるので、
最終的に全員が10枚のカードを持つことになる。

・・・で、いいでしょうか。かなり不十分な気がしまくり・・・

444 名前:132人目の素数さん:2006/03/14(火) 22:30:38
> また、M枚のカードを持っている人が一人だとすると、
> 次の交換で必ずMは減少する。

これがそもそも間違いだとさんざん指摘されているのだが。

445 名前:132人目の素数さん:2006/03/14(火) 22:31:01
>>443
M=12で並びの一部分が11,12,12のときは
L回じゃ減少しなくない?

446 名前:132人目の素数さん:2006/03/14(火) 22:31:28
12枚持っている人が1人だけでも、左の人が 11 枚カードを持っていれば
(11+1)/2 = 6 枚やってくる。
自分が隣に渡す枚数も6枚だから、
必ずしも M は減少しないね。


447 名前:132人目の素数さん:2006/03/14(火) 22:40:08
>>424
てか、これってどっかの試験問題じゃなかった?
2chの頭脳をもってしても数日解けない問題って・・・


448 名前:Sean ◆t.0oQdlc8Y :2006/03/14(火) 22:48:18
あー、もう、やっぱりだめだったか・・・
ちょっと概念的に考えすぎていました。
でも、もし10枚ずつ全員が持つ以外に
その枚数の状態がずっと続くような組み合わせがなければ
背理法でいつかは全員10枚になると思うのだが・・・?
そしたら・・・だめ、こんがらかってきた。
てかもう考えるのがメンド(ry

449 名前:132人目の素数さん:2006/03/14(火) 23:36:26
Au=Eu-.5(E+M)u+.5S(M+E)u=(E-.5(S-E)(E+M))u
A^nu=.5^r(-1)^rnCr(E+M)^r(S-E)^r
=.5^r(-1)^r(2^rE+nCr(2^r-1)M)(S-E)^r
=(E+.5nCrM)(rCkS^k(-1)^2r-k)

450 名前:132人目の素数さん:2006/03/15(水) 00:08:32
>424
ある段階でi番目にある枚数を a_i とする。
Σ(a_i-10)^2
が一度の操作によって必ず減少することを示せばよい。
<x> で x の ceiling, [x] で x の floor を表すとする。

一度操作を実行すると、i 番目の枚数は
[a_i] + <a_{i-1}>
になる。なので、
Σ(a_i-10)^2 - Σ( [a_i]+<a_{i-1}>-10 )^2 > 0 (*)
を示せばよい。
a_i = [a_i] + <a_i>
を用いて計算すれば (*) は
2Σ([a_i]<a_i>-[a_i]<a_{i-1}>)
に簡略化できる。

これって全ての i で a_i=10 になってないときは >0 だと思うんだけど
あってるかな?


451 名前:132人目の素数さん:2006/03/15(水) 00:20:47
 2Σ([a_i]<a_i>-[a_i]<a_{i-1}>) > 0
は以下を使う。

【補題】
二つの数列
 x_1≦x_2≦...≦x_n
 y_1≦y_2≦...≦y_n
が与えられたときに、数列
 z_1,z_2,...z_n
を y_i を任意にに並べ替えた数列とする。このとき
 Σx_i y_i ≧Σx_i z_i
が成立。
等号は y_i = z_i のとき。

今の場合、
x_i = [a_i]
y_i = <a_i>
z_i = <a_{i-1}>
と置けばよい。
y_i = z_i となるのは 各 a_i が全て同じ値10をとるときのみ。



452 名前:132人目の素数さん:2006/03/15(水) 00:29:22
450=451です。
説明下手糞でごめん。

補題が言っているのは、二つの数列を掛け合わせるときは、
大きいもの同士を掛け合わせるのが一番良い、
ということで、数列
 [a_1], ..., [a_n]
 <a_1>, ..., <a_n>
の場合、
[a_i] が大きければ <a_i> も大きくなるので
 Σx_i y_i
が最大の組合せになる。ということです。

453 名前:132人目の素数さん:2006/03/15(水) 00:37:52
>>452
1/2 忘れてる
[a/2], <a/2> とか書かないと

b = floor((a-10)/2), c = ceil((a-10)/2)
とかを定義したほうがすっきりする

ほかはいいんじゃ?

454 名前:132人目の素数さん:2006/03/15(水) 00:43:29
あ、ほんとだ。。。
指摘ありがと

455 名前:132人目の素数さん:2006/03/15(水) 00:45:00
>>441


456 名前:132人目の素数さん:2006/03/15(水) 00:54:03
>>455
あ、a_i≠a_j でも [a_i]=[a_j] だったり <a_i>=<a_j> だったりするから
駄目なのか…

457 名前:132人目の素数さん:2006/03/15(水) 01:26:56
大学への数学の「11月の宿題(2001年)」に、これの簡単なバージョンがあったよ。
1人だけが石を100個持ってる状態から始めてる。渡す石の個数は[(手持ちの石)/2]に
なっていて、ちょっと違ってる。これの解説は2002年1月号に載ってた。

458 名前:132人目の素数さん:2006/03/15(水) 02:26:00
>>457
でかした。
君は英雄だ、最大の功績だ!

459 名前:132人目の素数さん:2006/03/15(水) 02:46:45
n人でジヤンケンをしてあいこになる確率をもとめよ

460 名前:UnitedNiceKindOptional of Tama Kinng  ◆gXS.bDGPDU :2006/03/15(水) 04:33:57
>>459その場合は2000人とじゃんけんしたら求められ。。。。。。。

461 名前:132人目の素数さん:2006/03/15(水) 05:30:00
隣り合った二つの数の片方の半分の切り捨てと
もう片方の半分の切り上げの和を作ってるから同じ。


462 名前:132人目の素数さん:2006/03/15(水) 08:57:01
>渡す石の個数は[(手持ちの石)/2]に
>なっていて、ちょっと違ってる。
それは反対回りにして一個ずらすだけだから
本質的な違いとは言えないのでは

初期状態が違うのも多分それほど本質的ではなかったと思う

463 名前:132人目の素数さん:2006/03/15(水) 11:23:19
>>462
大学への数学の解答見てみた。うん、反対回りになってるだけだったw
でも、初期状態の違いについてはどうかな。解答を見る限りでは、どの解答も
初期状態(初めは1人だけが100個持っている)の条件をフルに使ってたから、
>>424には使えない気ガス。

464 名前:132人目の素数さん:2006/03/15(水) 14:58:07
>>441 の例が出た時点で終ったと思っていたけど、まだやっていたんだ。

カードの数の最大値を M (>10) とし、M 枚持っている人の数を N とする。

持っているカードの枚数が、左から見て、
i) M が偶数のとき、
M-1,M-1,...,M-1 (0 人以上),M,M,...,M (1 人以上)
ii) M が奇数のとき、
M,M,...,M(1 人以上),M-1,M-1,...,M-1 (0 人以上)
であるような極大な弧の長さの総和を l とする。

このとき、カードを交換すると、M が減少するか、
M は変化せず N が減少するか、M, N は変化せず l が減少する。

465 名前:132人目の素数さん:2006/03/15(水) 17:35:53
>>464
おぉ、確かにそうなる。
M>10のときM-2枚以下の人がいることを使ってるよね。
その証明で人数やカードの枚数によらず
交換を繰り返せば必ずカードの枚数は全部同じか連続2整数になる
ということが示せるな。

466 名前:132人目の素数さん:2006/03/15(水) 19:36:44
>>369
A,Bのゴール点をa,bとしてAの勝率をP(a,b)で表すと、a,b≧1のとき
P(a,b)=(1/36)Σ[0≦i,j≦5]P(a-i,b-j)
即ち P(a,b)=(1/35)Σ[0≦i,j≦5, not(i=j=0)]P(a-i,b-j)
ただし a≦0のときP(a,b)=1, a≧1,b≦0のときP(a,b)=0
よってP(a,b)の分母は奇数になり、1/2にはならない。

467 名前:132人目の素数さん:2006/03/15(水) 22:03:00
少なくとも大数の宿題より難しいってことだよなw
これは面白くなってきたのか絶望的になったのか・・・

468 名前:132人目の素数さん:2006/03/15(水) 22:04:36
>>442
分母が0でないことが、より簡単に示せる。
>>435の説明でもいいが、こっちのほうが明快。

469 名前:132人目の素数さん:2006/03/15(水) 22:19:26
9-5+6=10,11-6+5=10,10...10,5+5=10

470 名前:132人目の素数さん:2006/03/15(水) 22:27:10
>>467
>>464で解けてるんじゃないの?

471 名前:132人目の素数さん:2006/03/15(水) 22:32:59
10,20->15,15
10,10,20->10,15,15->13,15,12->14,13,13->14,13,13
10,10,10,20->10,10,15,15->10,13,15,12->12,14,13,11->13,14,12,11->13,13,12,12->13,12,12,13


472 名前:132人目の素数さん:2006/03/15(水) 22:47:30
an=(an+an+1)/2
(e+e)/2=(an+an+1)/2
(e/2+(o+1)/2)=(an/2+((an+1)+1)/2)=(an+an+1)/2+1/2
(o-(o+1)/2+e/2)=an-(an+1)/2+(an+1)/2=an/2+(an+1)/2-1/2=(an+an+1)/2-1/2
(o-(o+1)/2+(o+1)/2)=an-(an+1)/2+(an+1+1)/2=an/2+(an+1)/2=(an+an+1)/2
移動平均で+/-1/2だから何回かやれば平均値の+/-1になる
100で10人だから移動平均で10になってしまうだけ

473 名前:132人目の素数さん:2006/03/15(水) 22:59:18
>>472
しかしそれは証明にはなってない

474 名前:132人目の素数さん:2006/03/15(水) 23:06:12
移動平均は山を低くするー>平均にちかづくー>平らになる でいいじゃないか
これは宇宙の摂理だ

475 名前:132人目の素数さん:2006/03/15(水) 23:08:07
>>474
誤差を集めると偏りが生じる可能性があるから駄目。宇宙の摂理はむしろ
「塵も積もれば山となる」だと思うぞ。

476 名前:132人目の素数さん:2006/03/15(水) 23:09:47
正確な平均ではないからなあ

477 名前:132人目の素数さん:2006/03/15(水) 23:16:37
10,9,10->10,9,10で9は移動しない
10,11,10->11,10,10で11は左に動く
10,9,11,10->10,10,10,10で9と10はアニヒレートする
だから平均+/-1の配列はすべて10になる。。。とか?

478 名前:132人目の素数さん:2006/03/15(水) 23:19:37
カードの総数は100をこえないから発散は出来ない
循環するか・・・収束するか・・・

479 名前:132人目の素数さん:2006/03/15(水) 23:25:07
手持ちカードの数を無限に続く自然数列とする
手持ちカードが偶数なら半分を捨てる
奇数なら場から2倍のカードをひろい、さらに1枚ひろう
この操作を無限回くりかえすと手持ちカードの数は1になる

480 名前:132人目の素数さん:2006/03/15(水) 23:29:10
>>478
>>464

481 名前:132人目の素数さん:2006/03/15(水) 23:58:35
一辺が1である正六角形に覆い尽くされる三角形の面積の最大値と、そのときの三角形の形を求めよ。

482 名前:132人目の素数さん:2006/03/16(木) 00:17:36
>>481
1辺が√3の正三角形。

三角形PQRの頂点Pが六角形の辺AB上にあれば、
三角形の面積を減少させずにPをAまたはBに移動できる。
よって三角形の頂点は六角形の頂点にあると考えてよい。

483 名前:132人目の素数さん:2006/03/16(木) 00:19:31
>三角形の面積を減少させずにPをAまたはBに移動できる。
QRとABが平行じゃないと無理じゃね?

484 名前:132人目の素数さん:2006/03/16(木) 00:21:41
あ…「減少させずに」か。確かに出来るな。

485 名前:132人目の素数さん:2006/03/16(木) 02:07:41
図形問題をいくつか。
・1辺が1の正三角形は、1辺がL<1の正三角形2つで覆い尽くすことが出来ないことを示せ。
・半径が1の円は、半径がr<1の円2つで覆い尽くすことが出来ないことを示せ。
・a≧3,b≧1とする。正ab角形に覆い尽くされるa角形の面積が最大になるような、a角形の形を求めよ。
・xy平面上の全ての格子点に、白と黒の色をつけていく。このとき、ある4つの格子点について、その4点は
全て同じ色であり、しかもその4点を頂点とする四角形が正方形になっているものが存在することを示せ。

最後の問題は’長方形’にすると簡単なんだけど、正方形に変えてみたら解けなかった(もしかしたら存在しないかも)。

486 名前:132人目の素数さん:2006/03/16(木) 03:52:53
>>485
1問目
1辺がL<1の正三角形は1辺が1の正三角形の頂点を1つしか覆えないので不可能。
2問目
半径がr<1の円と半径が1の円周の交わりは180°未満の円弧になるので不可能。
3問目
>>482と同様にa角形の頂点は正ab角形の頂点にあるとしてよい。
a角形の連続する3頂点P,Q,Rについて、P,QおよびQ,Rの間にある正ab角形の頂点の数が
等しいか差が1のときに三角形PQRは最大となることから、最大のa角形の形は正a角形。
4問目
縦3×横9の領域において、縦3つの白黒の組合せは8通りだから同じ組合せの縦列がある。
各縦列に白か黒のいずれかは2つあるから、それら2組が長方形を成す。
正方形については今のところ分からない。

487 名前:132人目の素数さん:2006/03/16(木) 14:52:45
結構オモシロかった。ちょっと問い方を変えて出題。

次の条件のもと、同色で塗られた2点で、距離が1であるものが必ず存在するのはどれか(答えは1つとは限らない)。
(0)xy平面上の全ての点を、1種類の色で塗る。
(1)xy平面上の全ての点を、2種類の色で塗る。
(2)xy平面上の全ての点を、3種類の色で塗る。

488 名前:132人目の素数さん:2006/03/16(木) 23:26:08
>>487
答え:(0),(1),(2)
3色で塗って同色の点の距離が1にならないように塗るとすれば、
PR=√3,QS=1のひし形PQRSにおいてPとRは同色になる。
P,RはPR=√3を満たす任意の点である。
よってPR=PT=√3,RT=1なる3点P,R,Tを考えればR,Tは同色。
したがって条件を満たすように塗るのは不可能。

489 名前:132人目の素数さん:2006/03/16(木) 23:58:36
>>487
4色の場合は難しいな…。
いわゆる4色問題と関係あるんだろうか。無いよな。

>>485の2問目の拡張で
n個の半径1の円で覆える最大の円の半径f(n)を求めよ
という問題を考えたんだが、n=5ぐらいになるとややこしすぎ。
f(1)=1
f(2)=1
f(3)=(2√3)/3
f(4)=√2
だと思ふ。

490 名前:132人目の素数さん:2006/03/18(土) 12:18:19
>n個の半径1の円で覆える最大の円の半径f(n)を求めよ
とりあえずf(n)≦√nだな。でも正確なf(n)の値は求められないと思う。

491 名前:132人目の素数さん:2006/03/18(土) 17:11:24
>>490
nが大きくなると、1つの円で1辺1の正六角形を覆うことになると思うので
f(n)≦√{(3√3)n/2π}が成り立つかも。

>>487の4色の場合
1辺の長さがaの正五角形の頂点のうち少なくとも2つは同色。
よって、距離がaまたは{(1+√5)/2}aである同色の2点が存在する。
a=1とすると、距離が1または(1+√5)/2である同色の2点が存在する。
a=(-1+√5)/2とすると、距離が(-1+√5)/2または1である同色の2点が存在する。
よって、距離が1である同色の2点が存在しないと仮定すると、
距離が(1+√5)/2および(-1+√5)/2である同色の2点がそれぞれ存在する。
なんか惜しいんだが、この考え方ではこれ以上無理かもしれない。

492 名前:愚問だったらスマソ:2006/03/20(月) 11:33:56
>>20が面白かったので触発された。


定規とコンパスのみが与えられている。

問(1)
正n角形(nは3以上の自然数)が、定規とコンパスのみで
作図可能であるための必要十分条件は何か。

問(2)
作図可能な正n角形について、その正n角形を作図するために
必要なコンパスの最低使用回数をgとする。この時、nを変数として
gを求めることができるような公式は存在するか。

493 名前:132人目の素数さん:2006/03/20(月) 13:04:22
>>492
問い2は答え出たけど証明できんw
のでここには書かない(メール欄に)。
面白いけどかなりの難問だと思う。

494 名前:132人目の素数さん:2006/03/20(月) 13:22:17
>>493
それが答えなら、基本路線は背理法で決定だな。

495 名前:Geek ◆8MQVxjnUkg :2006/03/20(月) 15:57:05
(x-a)(x-b)(x-c)(x-d)・・・・・・・・・・・(x-y)(x-z)
を展開せよ。

496 名前:132人目の素数さん:2006/03/20(月) 16:05:08
0
がいしゅつ

497 名前:Geek ◆8MQVxjnUkg :2006/03/20(月) 16:41:16
あ〜あ、そんな気がしたんだ。
じゃああの本のはほとんど出てしまってるのか・・・?
つまんね。

498 名前:132人目の素数さん:2006/03/20(月) 23:44:42
>>492
ググって手掛かりは掴めたんだが証明がさっぱり分からん(´・ω・`)

ttp://www.math.okayama-u.ac.jp/~yoshino/pdffiles/kumamoto04.pdf
ttp://www.nn.iij4u.or.jp/~hsat/misc/math/drawing/heptadecagon.html
ttp://www.hamaint.co.jp/math/math_th_compas.html

>>493は多分正解だがやはり証明が?

>因みに, G. Mohr と L. Mascheroni に拠れば, 定規とコンパスで作図できる図形は
>すべてコンパスだけで作図できる (但し実際に直線を引くことは断念する)。
>又 J.-V. Poncelet と Steiner に拠れば, 定規とコンパスで作図できる図形は,
>最初に一つの円とその中心とが与えられてあれば定規だけで作図できる
>(但し実際に円を描くことは断念する)。

>>20の答えが「1回」なのも納得。

499 名前:Geek ◆8MQVxjnUkg :2006/03/20(月) 23:52:00
これも既出だったらあきらめる。あるゲームの問題。

対戦する二人は、紙の上に、100より小さい数を交互に書いていく。
相手に、書かれた数の中に、和が等しい二組の部分集合を見つけられたら負けとなる。

例えば、六回目が終わって19,2,27,39,5,11と出ていたら、11を書いたほうでないプレイヤーに
抜かりがなかったら、19+27=2+39+5を見つけて勝者となるだろう。

実は、このゲームは10回より多くやる必要がない。というのは、
100より小さい10個の数字が与えられれば、その中に和が等しい
二つの部分集合があるからである。
このことを証明せよ。

500 名前:132人目の素数さん:2006/03/21(火) 00:18:37
>>499
100より小さい数というのは1以上99以下の整数という意味でいい?
同じ数字は書いちゃいけないんだよね?
そうだとすると、
10個の数があるとき、それらの部分集合の和は0以上945以下だが、
部分集合の取り方は1024通りだから、同じ値になるものがある。
ってことでOKかな。

501 名前:132人目の素数さん:2006/03/21(火) 00:23:53
>>500
ぐわっ,先を越された・・・
というわけで鳩の巣原理

502 名前:132人目の素数さん:2006/03/21(火) 00:49:22
>>498
作図の中で定規とコンパスの役割は何かと考えると、
定規・・・ある2つの点を結ぶ直線を引く
コンパス・・・ある点を中心としてある点を通る円を描く
ってことだよね。
で、円とその中心が最初に与えられているとして、
定規だけで作図すると円は描けないけど、円と直線の交点、円と円の交点となる点が
作図できたとしたら、定規とコンパスで作図できる図形(但し実際に円を描くことは断念)を
作図できたことになるわけだ。
具体的には、円と直線の交点というのは、直線lと点O,Pがあるとき、もしコンパスが使えれば
Oを中心としてOPを半径とする円を描いて直線lと交わってできる点なわけだけど、
その交点を、定規だけで作図できるってことだね。
円と円の交点は、点O,P,O',P'があるとき、中心O半径OPの円と中心O'半径O'P'の円の交点が
もしあればその交点が定規だけで作図できる。
これら2つのことが証明できればいいはず。
具体的な作図の手順も示せればいいんだけど。

503 名前:132人目の素数さん:2006/03/21(火) 21:36:43
思いつきで出題してみる。答えは知らないんで今から考える。

AB=√2,AC=BC=1の直角二等辺三角形ABCの頂点A,Bにそれぞれ動点P,Qがある。
P,Qはそれぞれ、辺上を速さ1で一定方向に進むが、
頂点に来るたびに(出発時も)1/2の確率で他のどちらの頂点に進むかを決める。
いま同時にP,Qが出発したとき、PとQが出会うまでの時間の期待値はいくらか。

504 名前:503:2006/03/24(金) 22:34:52
すごくシンプルな問題なのにうまい解法が見つからない。
1億回シミュレーションした結果、
P,Qの移動距離の合計=82202430+85429954√2
この1/2倍が所要時間で、それを1億で割れば平均時間。
よって求める値の近似値は0.41101215+0.42714977√2=1.0150931…
多分有効数字3桁ぐらいなので、1.01から1.02の間と予想。
これは数学的に解くのはかなり難しいと思うので、誰か面白い問題おねがい。

505 名前:132人目の素数さん:2006/03/25(土) 03:01:14
正三角形なら簡単なのにな。
出発の直後
1/4の確率でお互い相手のほうに向かう→1/2秒後に出会う
1/4の確率でお互い第三の点に向かう→1秒後に出会う
1/2の確率で行き違いになる→1秒後に初期状態になる

よって期待値x=1/8 + 1/4 + (1+x)/2
∴x=7/4

>503の設定だと行き違いになったときに初期状態にきれいに戻らないのが難しくなる原因だけど、
>504を信用するとそれによって期待値が0.7秒も短くなっている。
これを直感的に言うと「長さが不ぞろいな方が出会いやすい」という事になるわけだが
頭の中でいくらイメージしても全然ピンと来ない…。




506 名前:503:2006/03/25(土) 05:31:51
>>505のおかげで気付いたんだけど、プログラムにミスが…。
正しくは↓

1億回シミュレーションした結果、
P,Qの移動距離の合計=263702706+143253424√2
この1/2倍が所要時間で、それを1億で割れば平均時間。
よって求める値の近似値は1.31851353+0.71626712√2=2.3314682…
多分有効数字3桁ぐらいなので、2.33から2.34の間と予想。

もし最初1/2の確率で行き違いになった場合は最短で出会うように移動する
として計算すると、
時間の期待値は(6+3√2)/8=1.28…
となって、問題の答えはこの値を超えるはずだと分かる。
しかしこんな計算をしなくても1.015ってのは小さすぎると気付くべきだった…。


どうでもいいけどミスの内容を説明。
言語はC++で、
sum1+=p1+q1+((pp+qp==1)?0:1);
sum2+=p2+q2+((pp+qp==1)?1:0);
と書くべきところを
sum1+=p1+q1+(pp+qp==1)?0:1;
sum2+=p2+q2+(pp+qp==1)?1:0;
と書くという痛いミス。演算の優先順位を考えてなかった。
sum1+=p1+q1+(pp+qp!=1);
sum2+=p2+q2+(pp+qp==1);
でもよかったけど。
ちなみにsum1,sum2はP,Qの合計移動距離sum1+sum2*√2の係数、
p1,p2,q1,q2はP,Qそれぞれの移動距離p1+p2*√2, q1+q2*√2の係数、
pp,qpはP,Qがそれぞれ最後にいた頂点を0,1,2(=A,B,C)で表す。

507 名前:503:2006/03/25(土) 06:07:58
辺の長さ2,1,1の三角形(もはや三角形ではないけど)だと、期待値は2になる。
>>503はこれより辺が短いのに、実験では期待値が2より大きくなっている。
>>505の言い方を借りると、「長さが不ぞろいな方が出会いにくい」ということに。
これは直観的に納得…かな。

508 名前:132人目の素数さん:2006/03/25(土) 14:33:16
>>507
長さ、3,4,5の直角三角形の場合どうなるの?
長さ4の正三角形だと7秒になるけど、これが、
周長が12の三角形で最短時間ってことか?
じゃ、最長時間は(6,6,0)の極限?

509 名前:132人目の素数さん:2006/03/25(土) 14:34:51
天皇陛下万歳!大東亜戦争万歳!
劣等民族を日本列島から追放しよう!
大日本帝国万歳!


510 名前:132人目の素数さん:2006/03/25(土) 20:42:42
おk

511 名前:132人目の素数さん:2006/03/26(日) 01:17:50
>>508
>長さ、3,4,5の直角三角形の場合どうなるの?
シミュレーションの結果、
(AB,AC,BC) (平均時間) (平均時間/3辺の和)
 (3,4,5) → 9.82603,  0.818836
 (4,3,5) → 8.53254,  0.711045
 (5,3,4) → 9.96872,  0.830727
と出た(ACとBCは入れ替え可)。
辺の長さが整数で合計12の三角形全てについて調べると、
(5,3,4)が0.830727で最大、(6,3,3)が0.5で最小という結果。
>じゃ、最長時間は(6,6,0)の極限?
(1,10000,10000)→0.606198(平均時間/3辺の和)
(10000,1,10000)→0.749505
と出た。
様々な辺の長さの組合せについて、(平均時間/3辺の和)を調べてみると、
(n,n,n±1)でn→∞の極限が約0.834で、これが上限っぽい。
最小は(2,1,1)の1/2と思う。これは簡単に証明できそう。
ちなみに(14142,10000,10000)は平均時間23253.1と出た。
>>506の結果に近い。

512 名前:132人目の素数さん:2006/03/26(日) 01:30:07
>>511の補足。
試行回数は1億回もやると時間がかかりすぎて様々なパターンを調べられないので、
全て100万回ずつにした。
>>506より精度が低く、おそらく有効数字2桁強。3桁目は±2程度の誤差がある模様。

513 名前:132人目の素数さん:2006/03/26(日) 12:39:52
>>511
そっか、(2,1,1)が最短でしたね。こっちは面白くないから、
やっぱり最長時間のほうが面白そう。
しかし、どう理解したらいいんだろう。

514 名前:132人目の素数さん:2006/03/26(日) 13:56:45
Y=x*e^(−x)*sin(x)とx軸との間で囲まれた部分を
x軸の周りに一回転させる。
この時できる立体群の体積のうち、x=(n−1)πとx=nπ【n=1、2、・・・・・】
の間の立体の体積をV(n)とする。
(1)V(1)を求めよ。

(2)V(n)を求めよ。

(3)lim【n→∞】ΣV(k)[k=1→k=n]を求めよ。

高校生スレッドからコピペしてきた。
たった今香具師が最初の答えを出してきた所だ。
喪前らもやって見ろw

515 名前:132人目の素数さん:2006/03/26(日) 14:44:11
>>514
くだらない問題をいろんなところにコピペしすぎ。

516 名前:511:2006/03/26(日) 20:27:38
実験結果の追加報告。
(平均時間/3辺の和)の値の上限と思われる0.833…に近い値になるための条件は、
3辺の長さが十分近い値で、AC≠BCかつAB≠(AC+BC)/2
ということが分かった。
例えば
(AB,AC,BC) (平均時間/3辺の和)
(10000,9998,9999) 0.833172
(10000,9998,10001) 0.833103
(10000,9999,9999) 0.678253
(10000,9999,10000) 0.833093
(10000,9999,10001) 0.760502
(10000,9999,10002) 0.832595
(10000,10000,10001) 0.833893
(10000,10001,10001) 0.677136
(10000,10001,10002) 0.832306
というような結果が出ている。
もちろん他のいろいろな値についても検証を行なった。
AC≠BCが必要というのは分かりやすいが、他の条件はどう解釈すべきだろう。

517 名前:132人目の素数さん:2006/03/26(日) 20:31:48
あ、AB≠(AC+BC)/2もちょっと考えれば分かるね。

518 名前:132人目の素数さん:2006/03/27(月) 00:57:51
連続投稿で申し訳ない。
二角形バージョン、つまりCが無くてA,B間に2本の道がある場合も実験してみた。
(1,1)=3/4 が最大の可能性があり、(1,2)=25/42 が最小っぽい。
(10000,10001)などもほぼ0.75になったが、3/4より大きいか小さいかは不明。
(10000,19999)=0.743002
(10000,20001)=0.723660
のようになったので、(1,2)は少しずれると値が大きくなることが分かる。
(1,2)の値は中途半端な小数が出たので正確な計算をしてみたところ、
思ったよりは計算が面倒だと分かった。
三角形バージョンの(3,1,1)なんか計算してみると良い計算練習になるよ。

519 名前:132人目の素数さん:2006/03/27(月) 03:14:08
いずれか2辺が大きな双子素数の場合はどうだろうか?

520 名前:132人目の素数さん:2006/03/27(月) 05:26:18
互いに素なのと変わらんような希ガス

521 名前:132人目の素数さん:2006/03/27(月) 08:26:02
>>519
実験したところ、特に素数だからといって変わったことは起こらなかった。
例えば双子素数10007,10009と10000について
(10000,10007,10009)=0.833207
(10007,10000,10009)=0.833260
(10009,10000,10007)=0.833360
(1億回の試行の結果。有効数字はおそらく3桁)
のようになった。

522 名前:132人目の素数さん:2006/03/27(月) 10:12:15
たくさんのマッチ棒がある。これをn個の山に分ける。
k番目の山にはマッチ棒がAk個(1<=k<=n, Ak>0)ある。
ここで、2人で交互に、好きな山から好きな本数だけ
マッチ棒を取るゲームを行う。ただし、1回の手番で
複数の山から同時にマッチ棒を取ることはできない。

(1) 最後の1本を取った方が勝ちとする。
後手必勝となるようなA1, ..., Anの条件を求めよ。

(2) 最後の1本を取った方が負けとする。
後手必勝となるようなA1, ..., Anの条件を求めよ。


523 名前:522:2006/03/27(月) 10:15:57
ごめん、もう一つ条件を忘れてた。
パス(自分の手番で0本取る)は不可。
必ず1本以上取らないといけない。


524 名前:132人目の素数さん:2006/03/27(月) 11:42:42
関数列S[n](x)は
S[0](x)=x
S[n](x)=n∫[0,x]{S[n−1](t)+c}dt,c=1/n−∫[0,1]S[n−1](y)dy (n≧1)
で定義されているとする。xが非負整数のとき、S[n](x)=Σ[k=0〜x]k^nが成り立つことを示せ。

525 名前:132人目の素数さん:2006/03/27(月) 16:50:32
nを3以上の整数とする。正n角形の各頂点に1つずつ整数を割り当てそれらn個の
整数の和が正になるようにする。連続する3個の頂点に割り当てられた整数を
それぞれx,y,zとする。このときy<0ならば次の操作を行う:
 3つの数x,y,zをそれぞれx+y,-y,z+yで置き換える。
n個の整数のうち少なくとも1つが負である限り上述の操作を繰り返し実行する。
有限回の操作の後、この手続きが完了するか否かを決定せよ。

526 名前:132人目の素数さん:2006/03/28(火) 00:34:29
>>522
2つの自然数x,yを被演算子とする2項演算x#yを次のように定める:
x,yを2進数で表したときの2^kの位の数をそれぞれx_k,y_kとして
x#y=Σ[k=0,∞]|x_k-y_k|*2^k
この演算は交換法則x#y=y#xおよび結合法則(x#y)#z=x#(y#z)が成り立つ。
A1#A2#…#An≠0のときはA1#A2#…#An=0となるようにマッチ棒を取ることができるが、
A1#A2#…#An=0のときはどう取ってもA1#A2#…#An≠0となる。
これを用いて、
(1) A1#A2#…#An=0のとき後手必勝。
(2) 全ての山がマッチ棒1本でそれが奇数個あるとき、
またはマッチ棒が2本以上の山が1つ以上ありA1#A2#…#An=0のとき後手必勝。

527 名前:132人目の素数さん:2006/03/28(火) 00:41:38
>>525
>>424の問題に似てるな。
操作を行なうときに負の数が複数から選べるけど、どう選んでも同じ回数で終わるみたい。

528 名前:522:2006/03/28(火) 21:49:24
>>526
おみごと。
ってか簡単でしたね。


529 名前:132人目の素数さん:2006/03/30(木) 19:33:46
>>525
操作を行なう負の数の選び方を変えても同じ回数で終わるだけでなく、
最後のパターンが同じになるようだ。

(-1),1,-2,3
1,0,(-2),2
1,(-2),2,0
(-1),2,0,0
1,1,0,(-1)
0,1,(-1),1
0,0,1,0

-1,1,(-2),3
-1,(-1),2,1
(-2),1,1,1
2,(-1),1,-1
1,1,0,(-1)
0,1,(-1),1
0,0,1,0
(括弧を付けた所が操作を施した所)

さらに、操作を行なった負の数をその場所ごとに合計していくと、どちらの場合も
-2,-2,-3,-1 で同じになっている。
いろいろ実験してみると、どうもこのことは全てのパターンについて言えるらしい。
しかし証明方法は不明。

530 名前:132人目の素数さん:2006/03/31(金) 10:04:46
狭義単調増加する自然数列anがa1=1,lim[n→∞]an/n=c (1≦c<∞)を満たすとき、ある自然数mが
存在して、どんな自然数kも高々m個のAの元の和で表せることを示せ。ただしA={an|n∈N}とする。

531 名前:132人目の素数さん:2006/03/31(金) 18:25:27
y=x*e^−x*sinxとx軸との間で囲まれた部分の面積のうち、
x=(n-1)π x=nπ で区切られた部分の面積をS(n)とする。

(1)S(1)を求めよ

(2)S(n)を求めよ

(3)Σ【k=1→k=n】S(k)を求めよ。

(4)lim【n→∞】Σ【k=1→k=n】S(k)を求めよ。


532 名前:532:2006/03/32(土) 21:16:24
5-3=2


533 名前:132人目の素数さん:2006/04/03(月) 18:21:43
難しい問題ばっかだな…。
ヒントくれ。

>>525
n=3の場合は簡単なんだけどね。

a,b,c で a+b+c>0 のとき、
負の数が2つある場合は1回操作すると負の数が1つになるから
最初から負の数が1つ(a<0, b>0, c>0)として考える。
このとき1回操作すると
-a,a+b,a+c で a+b<0 または a+c<0
さらに a+b<0 の場合、もう1回操作すると
b,-a-b,2a+b+c
このとき b>0, -a-b>0, 2a+b+c>a(a+b+c>0より) であり、
2a+b+c≧0 であれば操作は終了だが、
2a+b+c<0 の場合、最初の a,b,c と比べて、負の数が1つのままで
その値が大きくなっている。
よって操作を繰り返すといずれは負の数が無くなる。
a+c<0 の場合も同様。

n=4の場合も同じように考えればいけそうだが、一般に拡張するのは難しそう。

534 名前:132人目の素数さん:2006/04/05(水) 01:58:45
n個の実数a_1,a_2,…,a_nがあるとき、1≦k≦nなる任意のkについて、
a_kを除くn-1個を2つの組に分けてそれぞれの和が等しくなるようにできるならば、
a_1,a_2,…,a_nは全て等しいといえるか。

535 名前:132人目の素数さん:2006/04/05(水) 04:10:00
いえない。


536 名前:132人目の素数さん:2006/04/05(水) 06:38:19
>>535
簡単杉だったので訂正します。

2n+1個の実数a_1,a_2,…,a_{2n+1}があるとき、1≦k≦2n+1なる任意のkについて、
a_kを除く2n個をn個ずつ2組に分けてそれぞれの和が等しくなるようにできるならば、
a_1,a_2,…,a_{2n+1}は全て等しいといえるか。

537 名前:132人目の素数さん:2006/04/05(水) 10:57:28
>>536
a_1,a_2,…,a_{2n+1} はすべて自然数と仮定できる。
(R を Q 線型空間と見て、勝手な基底をとって成分を考えればよい)

明らかに、a_1,a_2,…,a_{2n+1} はすべて奇数となるかすべて偶数である。

b_k=a_k/2 (a_k が偶数のとき), (a_k+1)/2 (a_k が奇数のとき) とおくと、
b_1,b_2,…,b_{2n+1} も同じ条件を満たし、b_1+b_2+…+b_{2n+1}≦a_1+a_2+…+a_{2n+1}
となっている。この操作を続けていくと、いつかは等号が成立する。

等号が成立するのは a_1=a_2=...=a_{2n+1}=0 or 1 のときなので、これから最初の数の組がすべて等しいことがわかる。


538 名前:132人目の素数さん:2006/04/05(水) 15:59:33
>>537
正解。
ちなみに、Rの基底の存在を用いずに証明することもできるね。

和を積に変えると、0を3個以上含む場合は全て等しいと言えないが、
0を含まない場合は全て等しくなる。

539 名前:132人目の素数さん:2006/04/05(水) 22:42:19
age

540 名前:132人目の素数さん:2006/04/06(木) 00:01:57
age

541 名前: ◆xeS.CIM.Jk :2006/04/06(木) 01:58:11
n^4-19n^3-10n^2-532n+784が素数値をとるようなnは存在するか?

542 名前:132人目の素数さん:2006/04/06(木) 02:47:50
xyz空間内の点Pは1秒ごとに点Pからの距離が3であるいずれかの格子点に移動する。
原点を出発した点Pがn秒後に存在可能な点の総数をf(n)とするとき、
lim[n→∞]f(n)/(n^3)を求めよ。

543 名前:132人目の素数さん:2006/04/06(木) 04:12:10
>>541
n^4-19n^3-10n^2-532n+784=(n^2+3n+28)(x^2-22n+28)

544 名前:132人目の素数さん:2006/04/06(木) 06:53:03
(n^2+3n+28)(n^2-22n+28)でどうじゃろ?

545 名前:132人目の素数さん:2006/04/06(木) 17:02:54
「任意の自然数nに対してgcd(s,2*3^n+4^n+12^n−1)=1」を満たす自然数sを全て求めよ。

546 名前:132人目の素数さん:2006/04/06(木) 21:17:08
>>545
f(n)=2*3^n+4^n+12^n-1とおくと
p≧5なる素数pについて
f(p-2)=2*3^(p-2)+4^(p-2)+12^(p-2)-1
≡2*3^(p-1)/3+4^(p-1)/4+12^(p-1)/12-1
≡2/3+1/4+1/12-1
≡0(mod p)
またf(n)はつねに奇数かつ3の倍数なので
条件を満たすのはs=2^k(k≧0)

547 名前:132人目の素数さん:2006/04/06(木) 21:19:16
その十分性を示さないと。

548 名前:132人目の素数さん:2006/04/06(木) 21:26:15
>>547
明らか、じゃダメ?

549 名前:132人目の素数さん:2006/04/06(木) 21:30:56
明らかなのかなぁ…その十分性の証明は俺にはできんけど…。

550 名前:132人目の素数さん:2006/04/06(木) 21:33:49
f(n)は奇数だから。
何か勘違いしてるかなあ

551 名前:132人目の素数さん:2006/04/07(金) 09:02:29
>>546
正解です。十分性は明らかですぞ。

次の問題。まあ数オリの問題なわけだが。
整数列{an}は、正の項も負の項も無限回現れるとする。また、任意の自然数nに対して、
a1,a2,…,anをnで割った余りは全て異なっているとする。このとき、数列{an}には
全ての整数が1回ずつ現れることを示せ。

552 名前:132人目の素数さん:2006/04/07(金) 10:40:00
a(i)(1≦i≦n)は連続したn個の整数。


553 名前:132人目の素数さん:2006/04/07(金) 10:45:28
>>552
0,1,-1,2,-2,3,-3,…

554 名前:132人目の素数さん:2006/04/07(金) 16:28:41
>>551
>>552でほとんど正解だと思うけど答えてみる。

i<jならば、a1,a2,…,ajをjで割った余りは全て異なるので、ai≠ajである。
a1,a2,…,anの中に|ai-aj|=m≧nなるi,j(i<j)があるとすれば、
a1,a2,…,amにおいてaiとajはmで割った余りが等しいので矛盾。
よってa1,a2,…,anの任意の2項の差は最大でn-1なので、
a1,a2,…,anはn個の連続整数を任意の順に並べたもの。
{an}は正の項も負の項も無限に含むから、任意の自然数Nについて
ai<-N,N<ajなるai,ajが存在し、a1からaiまたはajまでの項は連続する整数だから
aiとajの間の整数は全て{an}に含まれる。
よって{an}は全ての整数を1個ずつ含む。

555 名前:132人目の素数さん:2006/04/08(土) 20:53:04
>>545も数オリの改題やね

556 名前:132人目の素数さん:2006/04/09(日) 01:23:07
円を直線で二等分して同じ面積にするには、中心点を通る線で分割すればいい。
では残った半円を、中心点を通る線と平行な線でさらに二等分して同じ面積にする場合
どこで分割すればいいか?
半径をRとして、中心点と通る線とその線の距離は?

557 名前:132人目の素数さん:2006/04/09(日) 02:38:10
いや、そんなに面白くは無いか。ハム切ってる最中に考えたネタだったが
冷めてみるとつまらんな。

558 名前:132人目の素数さん:2006/04/10(月) 22:01:11
>>556

sin2θ + 4θ = π を、θについて解けばよい(θが中心角)というところまでは分かった。
でも、数式処理のソフトに解かせてみても無理だったので、代数解は無いんじゃね?w


559 名前:132人目の素数さん:2006/04/12(水) 01:57:02
「1階からa階までn秒かかるエレベータは1階からb階まで何秒かかるか?」
これを、1階からb階までは1階からa階までのb/a倍の長さとして解いてしまい、
整数の答えが得られたが、正しい答えも整数であった。
これを満たす整数a,b,nの組を全て求めよ。

560 名前:132人目の素数さん:2006/04/12(水) 02:05:02
>>559
ごめん、この問題、無かったことにして。

561 名前:Prince:2006/04/14(金) 07:47:18
>1階からa階までn秒かかる
a階で止まる時か?それとも通過する時か?
途中の各階は通過するのか止まるのか?


562 名前:132人目の素数さん:2006/04/14(金) 12:18:54
平面上の3つの円C_1,C_2,C_3がそれぞれ互いに2点で交わっている。
C_2とC_3の2交点を結ぶ線分をl_1、
C_3とC_1の2交点を結ぶ線分をl_2、
C_1とC_2の2交点を結ぶ線分をl_3
とおくとき、l_1,l_2,l_3は共通の点を通ることを証明せよ。
エレガントな解答があります。どうぞ。

563 名前:132人目の素数さん:2006/04/14(金) 18:54:26
>>562
問題の平面をαとし、αを含む空間を考える。
同じ大きさの球S_1,S_2,S_3を、αとの交線がC_1,C_2,C_3となるようにとり、
S_1,S_2,S_3の中心をそれぞれP,Q,R、これらを通る平面をβとする。
S_2とS_3の交線を通る平面をγ_1、
S_3とS_1の交線を通る平面をγ_2、
S_1とS_2の交線を通る平面をγ_3
とおくと、γ_1,γ_2,γ_3はそれぞれQR,RP,PQの垂直二等分面なので、
それらは△PQRの外心を通りβに垂直な直線mを共有する。
l_1,l_2,l_3はそれぞれγ_1,γ_2,γ_3とαの交線であるから、
これらはmとαの交点を通る。

564 名前:132人目の素数さん:2006/04/14(金) 19:16:54
>>563
アイデアはほぼ模範解答と同じです!
ですが模範解答はもう少し簡単で想像しやすいものです。

565 名前:132人目の素数さん:2006/04/15(土) 00:22:08
>>564
その模範解答おしえて〜な

566 名前:562:2006/04/15(土) 01:00:57
模範解答です。

問題の平面をαとし、αを含む空間を考える。
中心と半径がC_kと同じである球面をS_kとおくと、(k=1,2,3)
S_2とS_3の交線、
S_3とS_1の交線、
S_1とS_2の交線
をそれぞれαに直交射影したときの像がl_1,l_2,l_3である。
S_1,S_2,S_3は共有点(2点)をもつので、
l_1,l_2,l_3はいずれもその点をαに直交射影した点を通る。

567 名前:132人目の素数さん:2006/04/15(土) 01:31:44
>S_1,S_2,S_3は共有点(2点)をもつので、

もたない場合もあるよ。

568 名前:132人目の素数さん:2006/04/15(土) 01:43:51
>>567
どういう場合?

569 名前:562:2006/04/15(土) 01:50:21
あーそうか。もともとの問題に不備があったようだ。
C_1,C_2,C_3の各内部の共通部分は空でない。
ことを入れておかなければなかったですね。
(そもそもそうじゃなきゃ線分l_1,l_2,l_3は交わらない。直線ならおk)
考えてくださった方申し訳ありませんでしたorz

570 名前:132人目の素数さん:2006/04/15(土) 01:55:15
l_1などは直線とすれば>>563でOK

571 名前:132人目の素数さん:2006/04/15(土) 02:00:00
x^2+y^2+ax+by+c=x^2+y^2+dx+ey+f,
x^2+y^2+dx+ey+f=x^2+y^2+gx+hy+iならば
x^2+y^2+ax+by+c=x^2+y^2+gx+hy+i。


572 名前:132人目の素数さん:2006/04/15(土) 02:07:00
>>571
なるほど、こっちのほうが簡単かも。

573 名前:132人目の素数さん:2006/04/15(土) 02:10:00
(x+1)^2+y^2=2^2。
x^2+y^2=2^2。
(x−1)^2+y^2=2^2。


574 名前:132人目の素数さん:2006/04/15(土) 02:20:29
>>573
あ、それ反例だな。気付かなかった。

575 名前:562:2006/04/15(土) 02:28:48
ふーむ、3円の交わり方にちゃんと条件を与えないといけなかったようですね。

576 名前:132人目の素数さん:2006/04/15(土) 02:41:59


【下4ケタ目は2】
【080で始まる】



(080を除いた下8ケタで)
3で割り切れない →割って1余る数ではない
2、5、11のいずれでも割り切れない

(携帯番号の下4桁)
2 3 5 7 9 11 13 17 19 23のいずれでも割り切れない

携帯番号の下4桁 もしくは (携帯番号の下4桁-1)は34で割り切れる

携帯番号の下8桁目は3、5、7のどれか

080以降に0はない



問:できる範囲までこの者の携帯電話番号を特定せよ

577 名前:132人目の素数さん:2006/04/15(土) 02:46:39


080◎○○▲●●●●


◎は3or5or7
▲は1or3or7or9
●は
2177
2279
2483
2687
2789のいずれか


(080を除いた下8ケタで)
3で割ると2あまる
奇数
5or11のいずれでも割り切れない

080以降に0はない

578 名前:132人目の素数さん:2006/04/16(日) 00:19:01
1辺の長さが1の正方形を4つつなげた凸の形のタイルをいくつか用いて
ある長方形を過不足無く覆えるとき、この長方形の縦と横の長さの条件を求めよ。

579 名前:132人目の素数さん:2006/04/16(日) 21:37:06
おそらく縦、横ともに4の倍数なんだろうが…
証明できん。orz


580 名前:132人目の素数さん:2006/04/16(日) 22:27:32
縦横片方が、だろ

たしか1×nのタイルをいくつか用いてある長方形を過不足無く覆えるとき
⇔縦か横のいずれかがnの倍数

が面倒な議論で証明できたと思う
nが素数だと楽なんだけど。。

581 名前:132人目の素数さん:2006/04/16(日) 23:25:51
>>580
>1辺の長さが1の正方形を4つつなげた凸の形のタイル
ですよ。

ちなみに1×nのタイルの場合の証明
長方形を1×1の正方形で区切り、上からi番目、左からj番目の場所に対し、
i-jをnで割った余りをその場所の番号とする。
このとき、1×nのタイル1枚は0からn-1までの番号を1つずつ覆う。
よって、0からn-1までの番号は同数ずつある。
長方形の縦の長さをnで割った余りをa、横の長さをnで割った余りをbとすると、
左上のa×bの部分以外の部分は0からn-1までの番号を同数ずつ含む。
左上のa×bの部分について、0番はMin(a,b)個ある。
よってab=Min(a,b)×nなのでa=0またはb=0、即ち長方形の縦または横の長さはnの倍数。

582 名前:132人目の素数さん:2006/04/20(木) 15:57:15
宇宙にある量子の数は?

583 名前:132人目の素数さん:2006/04/20(木) 20:43:01
たくさん。

584 名前:132人目の素数さん:2006/04/24(月) 05:09:31
将棋の手順の数よりは少ない。

585 名前:Queen ◆xeS.CIM.Jk :2006/04/25(火) 17:48:33
コンピュータに合計Nバイトのデータ集合があり、各々のファイルはmバイト以下であることが分かっている。
これらのバックアップを取って保管したい。このとき、容量nバイトのメディアを最低いくつ用意すればよいか。
ただし、N、m、nは自然数であり、N>n>m。

586 名前:GiantLeaves ◆6fN.Sojv5w :2006/04/25(火) 18:38:25
talk:>>585 そこは磁気テープを使うところだろう。

587 名前:Geek ◆8MQVxjnUkg :2006/04/25(火) 18:44:12
>>585
[[n/m]/[N/m]]個か?
深く考えてないから間違ってるかもしれないが。

588 名前:GiantLeaves ◆6fN.Sojv5w :2006/04/25(火) 18:47:17
talk:>>585 ファイルを記録する方法が明らかになっていないため、解けない。

589 名前:Geek ◆8MQVxjnUkg :2006/04/25(火) 18:48:26
>>588
そんな事言ってたら、例えば物理の問題なんて
一切解けない気がするんだが。

590 名前:Queen ◆xeS.CIM.Jk :2006/04/25(火) 18:48:25
>King
そういうトンチはやめてくれ。

>geek
どういう方針?気になる。

591 名前:Geek ◆8MQVxjnUkg :2006/04/25(火) 18:53:36
まず、最悪の場合を想定して、すべてのファイルがmバイトだとして、
そしたらすべてのファイルの個数は少なくとも[N/m](ここでガウス記号がいるか
要らないか自信ない・・・)となり、
また、nバイトのメディアに入れられるファイルは、少なくとも
[n/m]個だから、合計[[n/m]/[N/m]]個のメディアがあれば・・・あれ、
分子のほうが「少なくとも」ってのはおかしいか。ちょっと考え直す。

592 名前:Queen ◆xeS.CIM.Jk :2006/04/25(火) 18:58:40
具体的な数値でやってみるとよいだろう。

593 名前:132人目の素数さん:2006/04/25(火) 20:44:00
N/n.


594 名前:132人目の素数さん:2006/04/26(水) 21:29:05
まぁ、あれだ。
ファイルを全部つなげて圧縮してから
nバイトごとに分割保存だ。

595 名前:132人目の素数さん:2006/04/30(日) 19:30:04
テトリスDSがめちゃ楽しいんだが、ふと>>578の問題を思い出した。
これって>>581みたいなうまい解法はあるんだろうか。
それ以前に、解決されている問題なんだろうか。

596 名前:Queen ◆xeS.CIM.Jk :2006/05/01(月) 01:04:20
数列{a_n}を次で定義する。ただし、aは正の実数とする。

a_1=a、
a_(n+1)=(a_n)! (n:奇数)
a_(n+1)=1/(a_n) (n:偶数)
lim[n→∞]a_nを求めよ。

597 名前:ゆんゆんママ:2006/05/01(月) 01:36:17
自然数以外の階乗はΓ関数と解釈するという断り書きがないと
よろしくないのでは?

598 名前:Queen ◆xeS.CIM.Jk :2006/05/01(月) 02:08:40
>ゆんゆんママ
あぁ、そうだな。ご親切にありがとう。

599 名前:ゆんゆんママ:2006/05/01(月) 02:16:09
いえいえ。これからもゆんゆんをよろしくお願いします。

600 名前:Queen ◆xeS.CIM.Jk :2006/05/01(月) 02:19:26
>ゆんゆんママ
煽りや中傷を受けたなら私が守ろう。安心していてください。

601 名前:ゆんゆんママ:2006/05/01(月) 02:25:24
>>600
こちらこそ。ご丁寧にありがとうございます。
では、ごきげんよう。

602 名前:132人目の素数さん:2006/05/01(月) 06:14:44
なにこの展開! Σ(゚Д゚)!

603 名前:132人目の素数さん:2006/05/01(月) 15:38:22
>597-601
バロスwwwww

604 名前:他スレ 未解決問題:2006/05/04(木) 02:42:10
705 名前:132人目の素数さん[] 投稿日:2006/04/20(木) 22:50:21
αを1の2^n+1乗根とする。{f(α)}^2+{g(α)}^2=-1を満たし、係数が全て整数である多項式f(x)、g(x)が存在することを示せ。

どうすればいいのか分かりませんorz


(引用者コメント)
2^n+1=3 なら
α^2 + (α^2)^2 = -1
2^n+1=5 なら
(α+α^4)^2 + (α^2-α^3)^2 = -1

605 名前:132人目の素数さん:2006/05/04(木) 10:00:00
1+x+x^2+x^3+x^4+x^5+x^6+x^7+x^8=0。
(x+x^4+x^6−x^7)^2+(x^2−x^3−x^5−x^8)^2=−1。


606 名前:132人目の素数さん:2006/05/07(日) 08:42:26
n桁以下の非負整数を無作為に1つ選ぶとき、その整数のある
連続した2桁が「12」となっている確率pnを求めよ。

607 名前:132人目の素数さん:2006/05/07(日) 18:03:13
>>606
何進数? 二進数で0の代わりに1、1の代わりに2って書く国だったら
結構簡単な問題になるっぽい。

608 名前:132人目の素数さん:2006/05/07(日) 19:26:43
>>606
n桁以下の非負整数のうち、12の並びを「含まない」ものの総数をf(n)とし、
f(n+1)を漸化式で表すことを考える。

f(n)として数えられている各数の左に、任意の数字を1つ書き加えると、
総数は10倍に増える。ただしそれらのうち、書き加えたことにより先頭2桁が
12となってしまうものがf(n-1)通りあり、それらを除外する必要がある。
つまり f(n+1)=10f(n)-f(n-1)、( f(0)=1、f(1)=10 )。

これを解くと、p=5+2√6、q=5-2√6 として
f(n)=(p^(n+1) - q^(n+1))/(p-q)。
これにより、求める確率は、1-(f(n)/10^n)。

609 名前:132人目の素数さん:2006/05/07(日) 20:52:55
476 名前:□7×7=4□□ 投稿日:2006/04/24(月) 10:14:19 7Zv8LePn
解けるかどうか自分でもわからないんだが・・・

1:相手と自分とで数字を一つ指定する
2:何らかの方法で別の数字に変換する
3:変換された数字同士で計算し、最初に指定した数字がどっちが大きいか判断する

という方法は無いかな?

要は、相手の数字を知らずに自分の数字を知っているだけの状態で
どっちが大きいかだけがわかる方法ってないか?ってこと。

610 名前:132人目の素数さん:2006/05/07(日) 21:20:34
>>607
ゴメン、10進法のつもりだった。まあ何進法にしても大差ないけど。
>>608
正解。俺は「12の並びを含まないもののうち、先頭が1であるものをan,2であるものをbn,
それ以外であるものをcn」とおいて、an,bn,cnのそれぞれについて漸化式を立て、それを
使ってzn=an+bn+cnについての漸化式を立てて(当然z(n+1)=10zn−z(n−1)になる)解いた。

…実は、IDスレを見て作った問題。「IDの中に2文字の良い文字列が入ってる」確率を求めている感じ。

611 名前:132人目の素数さん:2006/05/08(月) 01:24:11
>>609
選べる数は整数?実数?
どっちにしても、数を変換する関数の逆関数が計算できると思うから、
相手の数の変換後の数から元の数を求めることができてしまうと思う。
つまり相手の数を知らずにというのは無理じゃないかと。
多分ね。

612 名前:132人目の素数さん:2006/05/08(月) 01:57:12
609ではないが、元スレを探してきたよ。
http://hobby8.2ch.net/test/read.cgi/puzzle/1092999551/476-480n

613 名前: ◆BhMath2chk :2006/05/08(月) 12:00:00
x^(2^n+1)=1。
x^2≠1。
y=Π_{0≦k<n}(1+ix^(2^k))。
z=Π_{0≦k<n}(1−ix^(2^k))。
 ((y+z)/2)^2−((y−z)/2)^2
=yz
=Π_{0≦k<n}(1+x^(2^(k+1)))
=Π_{0≦k<n}((1−x^(2^(k+2)))/(1−x^(2^(k+1))))
=(1−x^(2^(n+1)))/(1−x^2)。
(x(y+z)/2)^2+(x(y−z)/2i)^2=−1。


614 名前:132人目の素数さん:2006/05/10(水) 20:05:12
他スレより誘導されてきますた。
次の問題は解けますでしょうか?私は証明を知りません。
でも証明できたという人はいます。

2以上の任意の自然数nに対して Σ_[k=1,n]√k は無理数であることを示せ。

615 名前:132人目の素数さん:2006/05/10(水) 20:15:41
Σ1/kのときはどうやって証明するんだったかな

616 名前:132人目の素数さん:2006/05/12(金) 02:29:19
>>604
(2^n) + 1 ? (2^n) + 1 ?
確認したのか?

617 名前:132人目の素数さん:2006/05/13(土) 20:48:36


618 名前:132人目の素数さん:2006/05/17(水) 02:30:00
ほしゅ

619 名前:132人目の素数さん:2006/05/17(水) 12:25:08
age

620 名前:132人目の素数さん:2006/05/19(金) 17:28:49
大手家電量販店にはポイントカードというものがあります。
経験上、ポイントカードのシステムは次のようである。

・各商品には価格の何%というようにポイント還元率が設定されている。
・商品を購入する際、ポイントを「貯める」か「使う」かを選択できる。
・「貯める」場合は(価格)*(ポイント還元率)だけポイントが加算される。
・「使う」場合は貯まっているポイントの分価格から割引し(1pt=\1)、
 ポイントが0になる場合は割り引いた価格に対してポイント還元率を乗じて新しいポイントを得る。
 (ポイントが余る場合はそのまま)

さて、「貯める」か「使う」かの選択をどのような基準で選べば最も得をするか数学的に論ぜよ。
ただしポイントを「使う」ことによって早めに現金を手にすることは損得の基準とは関係ないこととする。
また、簡単のため一回の購入につき商品の数は1つであるとする。

621 名前:132人目の素数さん:2006/05/19(金) 17:29:39


622 名前:β:2006/05/19(金) 17:50:30
高い商品を買ってポイントを貯める。
その商品を売る。
売ったお金で安い商品を大量に買う。
このときポイントをそれぞれの商品に適量用いて利益が最大になるようにし、
別の所で安い商品を売る。

連鎖。

623 名前:132人目の素数さん:2006/05/19(金) 17:50:56
βは無視で。

624 名前:β:2006/05/19(金) 17:56:38
>>623
前みたいに顔射されたいかい?

625 名前: ◆BhMath2chk :2006/05/21(日) 18:00:00
A(0)=Q。
A(n)={a+b√(p(n))|a∈A(n−1),b∈A(n−1)}。

(1)Q⊂A(n)。
(2)x∈A(n),y∈A(n)=>x+y∈A(n),x−y∈A(n),xy∈A(n)。
(3)x∈A(n),y∈A(n),y≠0=>x/y∈A(n)。
(4)x∈A(n)でx^2が平方因数をもたない正の整数のとき
   x^2の素因数はp(k)(1≦k≦n)。
(5)a∈A(n),b∈A(n),a+b√(p(n+1))=0=>a=0,b=0。

n=0のときの(1),(2),(3),(4),(5)は成り立つ。

n=m−1のとき成り立つとする。
n=m−1のときの(1)からn=mのときの(1)は成り立つ。
n=m−1のときの(2)からn=mのときの(2)は成り立つ。
n=m−1のときの(2),(3),(5)からn=mのときの(3)は成り立つ。
n=m−1のときの(1),(2),(4),(5)から
n=mのときの(4)は成り立つ。
n=mのときの(2),(3),(4)からn=mのときの(5)は成り立つ。
よってn=m−1のときの(1),(2),(3),(4),(5)が成り立つなら
n=mのときの(1),(2),(3),(4),(5)が成り立つ。


626 名前:132人目の素数さん:2006/05/22(月) 17:49:03
有限集合Xの要素の数を#Xで表す。
A⊂Nがa>0に対し#(A∩{1,2,…,n})/n→a (n→∞)を満たすとき、
Aには任意の長さの(交差が0でない)有限等差数列が存在することを証明せよ。

627 名前:132人目の素数さん:2006/05/22(月) 19:53:47
非線形関数は線形関数であらわせるのか。

628 名前:132人目の素数さん:2006/05/23(火) 22:02:48
n次方程式 f(x)= Σ(i=0 to n) a_i *x^i =0 のn個の解を複素平面にとりその重心をGとする。

すると、f '=0,f''=0,f'''=0 ,,,,となる点の重心もGとなることを示せ。

629 名前:132人目の素数さん:2006/05/25(木) 21:35:30
根と係数の関係より明らか

630 名前:132人目の素数さん:2006/05/27(土) 13:06:35
ABCDE、五人の女の子が、パジャマパーティーをしました
五人のうち四人がパジャマの上着を交換しあいました
また、五人のうち三人が、ズボンを交換しあいました
そして、その交換後、上下をそろいで着ている女の子はいませんでした
五人はそれぞれ誰の上着とズボンを着ていたのでしょうか?

・「Aの上着を着ている女の子」のズボンをはいているのはCです
・「Bの上着を着ている女の子」のズボンをはいているのはAです
・「Dのズボンをはいている女の子」のズボンをはいているのはEです
・Aのズボンをはいている女の子は、Bの上着を着ていません

631 名前:132人目の素数さん:2006/05/27(土) 13:59:50
>>628
あたりまえ

632 名前:132人目の素数さん:2006/05/27(土) 14:00:31
>>630
誰がどれを着てたかなんてどうでもいいから全部俺にくれ

633 名前:132人目の素数さん:2006/05/27(土) 14:21:05
>>630
私としてはC,A,Eはズボンのみを履いていておっぱい丸出し、
そしてBとDは全裸でいる、というのが理想です

634 名前:132人目の素数さん:2006/05/27(土) 20:36:09
>>630
パジャマは全部>>632にやるから、中身は全部俺にくれ!

635 名前:ああああ:2006/05/28(日) 01:47:49
>>628

3次方程式 f(x)= Σ(i=0 to 3) a_i *x^i =0 の3個の解を複素平面上に三角形ABC、f '=0  となる2点をD,Eとする。

D,Eは三角形ABCに内接する面積最大の楕円の焦点になることを示せ。

636 名前:132人目の素数さん:2006/05/30(火) 00:18:14
x^3-1=0 で、もうダメっぽいんだけど。面積最小の間違い?

637 名前:132人目の素数さん:2006/05/30(火) 00:49:20
>>636
???

638 名前:636:2006/05/30(火) 01:04:55
すまん。こちらが勝手に「外接」と勘違いしていた。

639 名前:132人目の素数さん:2006/06/03(土) 01:54:34
女の子だとどうしても解答に邪念が入るだろうから変更する

ABCDE、五人の男の子が、パジャマパーティーをしました
五人のうち四人がパジャマの上着を交換しあいました
また、五人のうち三人が、ズボンを交換しあいました
そして、その交換後、上下をそろいで着ている男の子はいませんでした
五人はそれぞれ誰の上着とズボンを着ていたのでしょうか?

・「Aの上着を着ている男の子」のズボンをはいているのはCです
・「Bの上着を着ている男の子」のズボンをはいているのはAです
・「Dのズボンをはいている男の子」のズボンをはいているのはEです
・Aのズボンをはいている男の子は、Bの上着を着ていません

640 名前:132人目の素数さん:2006/06/03(土) 01:57:58
このヒントだけで解けるのかい

641 名前:132人目の素数さん:2006/06/03(土) 03:52:31
>>639
女の子じゃないと解く気がしない

642 名前:132人目の素数さん:2006/06/03(土) 04:26:41
邪念が消えたらすんなり考える気になった。邪念恐るべし。
人 A B C D E
────────
上 C D A B E
下 D B C E A

おそらく、これしかないはず。
[人*上]、[人*下]という二つのマトリクスを作って、○×を書き込んで調べた。

・ [人*上] 表では、(A,B)のマスは常に×、なおかつ対角線上にはただ1つの○がくる。
・ [人*下] 表では、(D,D)のマスは常に×、なおかつ対角線上にはちょうど2つの○がくる。

という事実をフル活用したら、場合分けはそれほど爆発しなかった。

643 名前:132人目の素数さん:2006/06/04(日) 14:21:20
転載
http://science4.2ch.net/test/read.cgi/math/1134352879/645

645 名前:132人目の素数さん[] 投稿日:2006/06/04(日) 12:11:10
nを3以上の整数とする。正n角形の各頂点に1つずつ整数を割り当てそれらn個の
整数の和が正になるようにする。連続する3個の頂点に割り当てられた整数を
それぞれx,y,zとする。このときy<0ならば次の操作を行う:
 3つの数x,y,zをそれぞれx+y,-y,z+yで置き換える。
n個の整数のうち少なくとも1つが負である限り上述の操作を繰り返し実行する。
有限回の操作の後、この手続きが完了するか否かを決定せよ。

644 名前:132人目の素数さん:2006/06/04(日) 14:22:38
失敗、↓でした
http://science4.2ch.net/test/read.cgi/math/1149124263/645

645 名前:132人目の素数さん:2006/06/04(日) 18:00:01
正n角形の各頂点の数をx1〜xnとする
f_i(x1,x2,…,xn)=Σ|隣り合ったi個の数の和|とする
例えばf_1(x1,x2,x3,x4,x5)=|x1|+|x2|+|x3|+|x4|+|x5|
f_3(x1,x2,x3,x4,x5)=|x1+x2+x3|+|x2+x3+x4|+|x3+x4+x5|+|x4+x5+x1|+|x5+x1+x2|

f(x1,x2,…,xn)=Σ[i=1,2,…,n]f_iとすれば
f(x1,x2,…,x+y,-y,z+y,…,xn)-f(x1,x2,…,x,y,z,…,xn)=|y+Σxi|-|-y+Σxi|<0
よってx1〜xnに中に負の数があれば>>643の操作を繰り返すたびfは小さくなっていく
しかしfは絶対値の和だから0より小さくはならない
よって途中でx1〜xnの中に負の数がなくなって>>643の操作を出来なくなる事が分かる

646 名前:132人目の素数さん:2006/06/11(日) 16:54:41
実数xがx^[x] = 9/2を満たすとする。
このとき、xは有理数か、無理数か。

647 名前:132人目の素数さん:2006/06/11(日) 17:07:30
>>646 追加
[x]はxを超えない最大の整数。 念のために、x>0も追加しておく。

648 名前:132人目の素数さん:2006/06/11(日) 17:26:57
>>646
正の有理数x,非負整数mに対してx^m=9/2が成り立つのは(x,m)=(9/2,1)に限ることが簡単に
示せるので、これを使うと、x^[x]=9/2を満たす有理数xは(x,[x])=(9/2,1)を満たすことに
なるが、これを満たす有理数xは存在しない。よってx^[x]=9/2を満たすxは無理数。

649 名前:132人目の素数さん:2006/06/12(月) 00:58:52
>>646別解
関数x^[x]は単調増加でありx=2のとき2^2=4, x=3のとき3^3=27なので
x^[x]=9/2より[x]=2, さらにx=(9/2)^(1/2)とわかるからxは無理数

650 名前:132人目の素数さん:2006/06/12(月) 01:36:49
age

651 名前:132人目の素数さん:2006/06/13(火) 16:22:47
>>635 の答え
http://aozoragakuen.sakura.ne.jp/taiwaN/taiwaNch03/node10.html


652 名前:132人目の素数さん:2006/06/14(水) 00:33:13
意外に簡単な結果になるので出題してみる

sin(9°) + cos(9°)

を求めよ。

653 名前:132人目の素数さん:2006/06/14(水) 01:33:32
>>652
-( sin(666°) + cos(6*6*6°) ) を求めよ。

ってのも欧米人は好き。

654 名前:132人目の素数さん:2006/06/14(水) 03:00:43
>>639
俺はこっちで邪念が入る
誰がどれ着てたかなんてどうでもいいから服も中身も全部俺にくれ

655 名前:132人目の素数さん:2006/06/15(木) 16:25:59
任意の関数f:{1,2,...,n}^2→{-1,1}に対してΣ[x,x']Σ[y,y']f(x,y)f(x,y')f(x',y)f(x',y')≧n^3となる事を証明せよ

656 名前:132人目の素数さん:2006/06/15(木) 17:21:39
>>655
与式=Σ[x,x']{Σ[i=1〜n]f(x,i)f(x’,i)}^2
=Σ[x=x']{Σ[i=1〜n]f(x,i)f(x’,i)}^2+Σ[x≠x']{Σ[i=1〜n]f(x,i)f(x’,i)}^2
≧Σ[x=x']{Σ[i=1〜n]f(x,i)f(x’,i)}^2
=Σ[x=1〜n]{Σ[i=1〜n]f(x,i)f(x,i)}^2
=Σ[x=1〜n]{Σ[i=1〜n]1}^2
=n^3

657 名前:132人目の素数さん:2006/06/15(木) 18:10:52
1と9を二つずつ用いて、
四則計算で10を作りなさい

658 名前:132人目の素数さん:2006/06/15(木) 23:29:01
(1+(1/9))*9=10
(1+(1/9))*9=10
((1/9)+1)*9=10
((1/9)+1)*9=10
(1+(1/9))*9=10
(1+(1/9))*9=10
((1/9)+1)*9=10
((1/9)+1)*9=10
9*(1+(1/9))=10
9*((1/9)+1)=10
9*(1+(1/9))=10
9*((1/9)+1)=10
9*(1+(1/9))=10
9*((1/9)+1)=10
9*(1+(1/9))=10
9*((1/9)+1)=10

659 名前:132人目の素数さん:2006/06/16(金) 16:34:11
空間内に平面αがある。1辺の長さ1の正四面体Vのα上への正射影の面積をSとし、
Vがいろいろと位置を変えるときのSの最大値と最小値を求めよ。

660 名前:132人目の素数さん:2006/06/16(金) 17:20:41
>>659
東大過去問乙

661 名前:132人目の素数さん:2006/06/17(土) 05:12:04
>>1
「無意味なスレ立て厳禁」
って読めませんか?
そういうくだらない話は質問スレでやってください


 
                 終   了


そして>>1はすぐ死ね

662 名前:132人目の素数さん:2006/06/17(土) 09:27:07
tan(π/11)-tan(2π/11)+tan(3π/11)+tan(4π/11)-tan(6π/11)の値はどうすれば求めれますか?

663 名前:132人目の素数さん:2006/06/17(土) 13:06:56
今、ここにお金の入った封筒が2つあります
一つの封筒にはもう一つの封筒の2倍の金額が入っています
今あなたはどちらか一方の封筒を選ぶ事が出来ます
さて、今あなたは一つの封筒を選びましたが
もう一つの封筒に代えても良いと言われました
あなたは考えました
今の封筒にx円入っているとするともう一つに入っている
金額は2倍の確率(1/2)、半分の確率(1/2)だから
期待値は
2*x*(1/2)+(1/2)*A*(1/2)=1.25x
となるため、もう一つの方に代えた方が良いと考えました
さて、これは正しいでしょうか?

664 名前:132人目の素数さん:2006/06/17(土) 14:20:21
>>663
自然数の集合Nには確率測度が入らないからそういう確率の議論が出来ない、
とかじゃなかったっけ?

665 名前:132人目の素数さん:2006/06/17(土) 15:55:34
>>663
xのとりうる値に上限がない→xの期待値は無限大→xの半分も二倍も無限大

666 名前:132人目の素数さん:2006/06/17(土) 17:27:54
>>665
封筒を開けて金額を見ても良いという条件を加えたらどうなる?
例えば、1000円だったとしても、>>663の計算は
適用できるだろ?

667 名前:enkou@hot .co.jp:2006/06/17(土) 17:28:24
女子高生の簡単援交アルバイト。萌えっ娘くらぶ 投稿援交白書vol.2
http://ilikeenkou.com/

668 名前:132人目の素数さん:2006/06/17(土) 17:59:06
>>666
俺なら交換しない。なぜなら、
封筒に入っている金額が1000円以下である確率は0だから。
封筒に入っている金額が1億円以下である確率は0だから。
封筒に入っている金額が1京以下である確率は0だから。
封筒に入っている金額が10^1京円以下である確率は0だから。
封筒に入っている金額が10^(10^(10^(10^(10^(10^(10^10))))))円以下である確率は0だから。

一般に、どんな自然数nに対しても、封筒に入っている金額がn円以下である確率は0だから。
わざわざ交換しなくたって、封筒の中には我々が十分満足できる金額が入っている。そうだな、
一国家の数億年分の予算でもまだ足りないくらい入ってるだろう。

669 名前:668:2006/06/17(土) 18:00:08
>封筒に入っている金額が10^1京円以下である確率は0だから。
なんじゃこりゃwこれは削っておいてくれ。

670 名前:132人目の素数さん:2006/06/17(土) 18:08:14
>>668
なんか良く分からない
もう一つの封筒に入っている金額は
500円か2000円のはずなんだけど

671 名前:668:2006/06/17(土) 18:13:12
>>670
ああ、1000円の場合ね。だったら、封筒にたったの1000円しか入ってなかった
自分のあまりの運のなさに絶望して交換云々どころじゃないな。

672 名前:132人目の素数さん:2006/06/17(土) 18:20:11
>>671
ちゅうか数学の問題として考えろよ
じゃあ1000万円で

673 名前:132人目の素数さん:2006/06/17(土) 18:26:43
>>663-672
こんな確率もとめてみたい その1/3
http://science4.2ch.net/test/read.cgi/math/1109546954/


674 名前:668:2006/06/17(土) 18:34:59
>>672
まだ分からんのか?封筒に入っている金額に上限がないのに、一体どうやって期待値Eを算出するの?
直接計算するか?あるいは、封筒に入っている金額の上限がn円である場合の期待値Enを求め、これの
極限値を求めるか?それとも全く別の方法か?

前者の場合:
E=Σ[i=1〜∞]P(x)E(x)=0となる。
( P(x)は、初めの封筒に入っていた金額がx円である確率。つまりP(x)=0)
( E(x)は、初めの封筒に入っていた金額がx円であるときの、もう1つの封筒に入っている金額の期待値。)

後者の場合:
解釈1:問題文の設定より、Enは計算できない。というか、そもそも封筒Aに上限を設定することが出来ない。
なぜなら、初めの封筒Aに入っている金額の上限がn円である場合、残りの封筒Bに入っている金額の上限は2n円と
なるから。問題文の設定から、初めに封筒Bを開けてしまえば、1/2nの確率で封筒Bにはn円が入っていて、そのとき、
残りの封筒Aには1/2の確率でn*2=2n円入っていることになり、封筒Aに入ってる金額の上限がn円であることに矛盾する。

解釈2:「封筒をA,Bと区別し、必ず封筒Aから明け始める」と問題文を変更すれば上のような矛盾が起きないのでEnも
計算可能で、従ってlim[n→∞]En=Eも計算可能となる。En=Σ[i=1〜n](1/n)(n/2*1/2+2n*1/2)=Σ[i=1〜n]5/4=5n/4
よってE=∞

全く別の方法の場合:知らん。

675 名前:668:2006/06/17(土) 18:45:39
>今の封筒にx円入っているとするともう一つに入っている 金額は2倍の確率(1/2)、
半分の確率(1/2)だから 期待値は 2*x*(1/2)+(1/2)*A*(1/2)=1.25x となるため、
もう一つの方に代えた方が良いと考えました さて、これは正しいでしょうか?
↑この人は「封筒にx円入っていた場合の、もう片方の封筒に入っている金額の期待値」=E(x)を
求めたにすぎない。E(x)のみから判断してるのが意味不明。求める期待値Eは、このE(x)に「初めの
封筒に入っていた金額がx円である確率」=P(x)=0をかけた和だからE=Σ[i=1〜∞]P(x)E(x)=0となる。
つまり、この人は期待値Eを求める前に計算を止めて意味不明の判断を下しているだけ。もっとも、E自体も
0という意味不明な値になってるが。(これは問題文の状況設定「封筒に入ってる金額に上限が無い」が原因。)

676 名前:132人目の素数さん:2006/06/17(土) 18:50:57
>>675
だからx=1000円って事に問題変更
一つを選び、封筒を開けたら1000円入っていた
さて、あたなは代えるべきか否か
2000*(1/2)+500*(1/2)=1250
が期待値に思えるがどうなの?って問題


677 名前:668:2006/06/17(土) 18:59:33
>>676
初めから1000円に固定されているのなら、「変えるべき」だろ。期待値が1250円なのだから。
>2000*(1/2)+500*(1/2)=1250 が期待値に思えるがどうなの?って問題
明らかにそれは期待値。何を言ってるんだwちなみに、1000円に固定するのなら、
その問題は次の問題と同じ。

問題:自分の所持金は1000円である。目の前には1回1000円のくじ引き屋がある。
このくじ引きでは1/2の確率で500円が当たり、残り1/2の確率で2000円が当たる。
このくじ引きはするべきか否か。

678 名前:132人目の素数さん:2006/06/17(土) 20:26:02
期待値の高い戦略が常に有効かといわれるとそうでもない。
コインをn回投げて全部表なら4^n円もらえるとしよう。1枚でも裏なら0円。
nが大きくなれば期待値は発散するが、
この場合可算無限回投げるのが最適な戦術か。

679 名前:132人目の素数さん:2006/06/18(日) 10:39:53
>>677
じゃあ最初に戻って、1000円に固定しないようにしよう
封筒を取りました
開けても良いと言われました
開けたら1000円だった
上の議論により、「変えるべき」という結論
もう一回やりましょう
今度は10000円でした
やはり「変えるべき」
つまりこのゲームは常に「変えるべき」が答えなのか?

680 名前:132人目の素数さん:2006/06/18(日) 11:10:52
>>679
1回目では情報が得られていないので変えようが変えまいが一緒なのは自明。
2つのサイコロの目をふせたまま振って同じ目ならば勝ちというゲームで、
一方のサイコロの目をオープンにして、
このサイコロの出目を自由に変えてもいいよと言われて
勝つ確率を1/6より大きくも小さくもできないのと同じ。

一方だけを見てやめるを繰り返すと確率分布がわかってくる。
勝負をかけるならそこから。

681 名前:Geek ◆8MQVxjnUkg :2006/06/18(日) 11:13:24
この確率の問題もなかなか難しいな・・・・・・

xの上限値がないって言ってるけど、それって必要なの?
実際、上限値がある必要ないと思う。それに、これは
入っている金額をx円だと「仮定して」期待値を求めてるんだから、
「x円である確率」とか関係ないんじゃない?

そして、封筒を見て1000円だったときの期待値が1250円なら、
帰納法ですべての自然数についてそれがいえるんじゃないの?
まぁ、0.5円とか実質的に無理なのはあるけど、期待値には影響しないでしょ。
ドルとかセント使ってもいいんだし。

しかし、常に変えたほうがいいなんて事だったら、感覚的におかしいよな・・・・・
どっちも選んでない状態ならどっちかを選んで得する確率も損する確率も
1/2なのに選んだとたんに損する確率のほうがでかくなるって・・・・・・

三囚人問題とかもそうだけどさ、これって、「選ぶほう」じゃなくて
「選ばれるほう」を主観にしてみているからおかしな事になるんじゃない?

682 名前:132人目の素数さん:2006/06/18(日) 14:01:43
>>681
あなたは期待値だけで行動を決定するのですか。

歪みのないコインをn回投げて全部表ならa^n円もらえるとしたときどうしますか。
a=1.01、a=2、a=100のときで戦術は変わりますか。
その方法はコインを投げる回数nを最初から決めておかねばならないときと、
途中でnを変えてもよいときで変わりますか。
関数がa^nよりももっと速く増加する関数。
例えばn^n^…^n(指数がn重)、3↑…↑3(矢印がn個)のときはどうですか

683 名前:668:2006/06/18(日) 14:20:58
>>681
>それに、これは 入っている金額をx円だと「仮定して」期待値を求めてるんだから、
「x円である確率」とか関係ないんじゃない?
関係ある。封筒の金額に上限が無くて無限だから、有限個の場合(=封筒の金額に上限が
ある場合)と同じ考察を適用してはならない。

>「選ぶほう」じゃなくて「選ばれるほう」を主観にしてみているから
おかしな事になるんじゃない?
その言い分がもし正しいとすると、その言い分は他のあらゆる「期待値に関する問題」にも
適用できてしまうのだが。おかしなことになる、原因は封筒の中身に上限が無いことと、
問題文の「あなた」が期待値E(x)のみから「変えるべき」という支離滅裂な判断を下している
ところ。判断を下すなら期待値Eを求めなければならない。

684 名前:132人目の素数さん:2006/06/19(月) 09:22:45
二次関数 y=-x^2+4x (0≦x≦a)において、定数aの値が次のような範囲にあるとき、
この関数の最大値と最小値を求めよ。

(1) 0<a<2 (2) 2≦a<4 (3) a=4 (4) 4<a

すみません、おねがいします

685 名前:132人目の素数さん:2006/06/19(月) 09:31:50
>>684
コピペしすぎ

686 名前:132人目の素数さん:2006/06/19(月) 09:32:08
>>684
コピペしすぎ

687 名前:132人目の素数さん:2006/06/19(月) 12:09:28
(y/x) + (z/y) + (x/z) = 3
を満たす整数x,y,zを全て求めよ。

688 名前:132人目の素数さん:2006/06/19(月) 15:21:08
arctan(1/1)+arctan(1/2)+arctan(1/3)+arctan(1/4)+arctan(1/x)=π
を満たすxを求めよ。


689 名前:132人目の素数さん:2006/06/19(月) 18:29:29
仮に片方に1000円が入っていたとしても、
もう一方に2000円が入っている確率と500円が入っている確率は同じではないし
(同じであると考える理由が無い)

690 名前:132人目の素数さん:2006/06/19(月) 18:40:38
>>689
その確率が「同じである」というところまでが問題で設定されてるんじゃないの?

691 名前:132人目の素数さん:2006/06/19(月) 18:44:30
問題で仮定されてるのなら>>663は文句なく正しいわけだが

692 名前:668:2006/06/19(月) 18:54:52
>>679
>じゃあ最初に戻って、1000円に固定しないようにしよう
「最初に戻って」だと?ふざけるな。戻ってないじゃん。問題文変わってるよw

問題1:今、ここにお金の入った封筒が2つあります。一つの封筒にはもう一つの封筒の2倍の金額が入っています。
今あなたはどちらか一方の封筒を選ぶ事が出来ます。あなたは一つの封筒を選びました。が、そのとき(←封筒の
中身をまだ見てない)、「もう一つの封筒に代えてもいいよ」と言われました。あなたは封筒を変えるべきでしょうか?
解答:期待値Eを計算すると、(>>674の解釈1の場合)E=Σ[i=1〜∞]P(x)E(x)=0となり、(>>674の解釈2の場合)E=∞と
なるので意味不明。そもそもこの計算でちゃんと「期待値」になってるのかも怪しい。てか、そもそもこの問題では
どちらの封筒の中身もまだ分かってないので、変えても変えなくても一緒だろう。

問題2:今、ここにお金の入った封筒が2つあります。一つの封筒にはもう一つの封筒の2倍の金額が入っています。
今あなたはどちらか一方の封筒を選ぶ事が出来ます。あなたは一つの封筒を選んだら、x円入っていました。が、
そのとき、「もう一つの封筒に代えてもいいよ」と言われました。あなたは封筒を変えるべきでしょうか?
解答:変えるべき。期待値Eを計算するとE=1.25x になっているから。


当初の>>663は問題1の方。初めに選んだ封筒の中身がまだ分かってない状態で、変えるか否かの選択を求められて
いる。それに対し、おまいが>>679で言ってるのは問題2の方。初めに選んだ封筒の中身が分かってる状態で、変えるか
否かの選択を求められている。問題2の場合は明らかに「変えるべき」となる。

693 名前:132人目の素数さん:2006/06/19(月) 19:19:29
>>687
x=y=z

694 名前:132人目の素数さん:2006/06/19(月) 19:20:15
問題2なら変えるべき、ってそんなわけないじゃん。
見たら必ず変えることになるんだから。

実際には封筒に入った金額にはある確率分布が存在して、
その確率分布が分かっているなら当然変えるべきか
計算できる。(変えるべきかは見た金額による)
確率分布が分かっていないなら、当然変えても変えなくても同じ。

極端な話、封筒に入っているのは日本円の紙幣一枚と分かっていたとする。

1000円→もう一方は2000円なので変えるべき。
2000円→もう一方は1000円なので変えるべきでない。
5000円→もう一方は一万円なので変えるべき。
一万円→もう一方は5000円なので変えるべきでない。

となりますね。

別の例: 封筒に入っているのは1〜10000円のいずれかとする。

奇数→もう一方は必ず2倍なので変えるべき。
5000円以上→もう一方は必ず半額なので変えるべきでない。
それ以外→変えるべき。

という戦略になりますか。


695 名前:668:2006/06/19(月) 19:29:05
>問題2なら変えるべき、ってそんなわけないじゃん。
>見たら必ず変えることになるんだから。
分かってねぇな。たぶん おまいは「見たら必ず変える・見たら必ず変えない の2つの戦略は全く同じだろ!」と
言いたいのだろうが、それは違う。なぜなら、封筒の金額には上限が設定されてないから。おまいが言ってるのは

「(封筒の金額に上限が設定されているときは)見たら必ず変える・見たら必ず変えない の2つの戦略は全く同じだろ!」

ということにすぎない。
>極端な話、封筒に入っているのは日本円の紙幣一枚と分かっていたとする。
>別の例: 封筒に入っているのは1〜10000円のいずれかとする。
↑おまいのこれらの発言も、そのことを裏付けている。

696 名前:694:2006/06/19(月) 19:29:48
ちょっと抜け。後半は1〜10000円が全て等確率(1/10000)の場合ね。

確率を変えれば当然戦略は変わってくる。
1000円だったとき交換すべきかどうかを考えると、
全部等確率なら、500円と20000円が同じ確率なので
交換すべきだろう。
しかし、例えば2000円の確率が1/10000より低い場合を考えると、
(500, 1000)の組である確率が高く、(1000, 2000)の組である確率は
低いことになる。従って戦略は確率分布次第ということが分かる。


697 名前:694:2006/06/19(月) 19:40:06
>> 695

何を言われているのかよく理解できないのですが、
上限が無いなら無いで、確率分布を設定してみてくれませんか?
金額がxである確率p(x)を。
当然Σp(x) = 1な奴を。

それが確定すれば、見た金額に対して交換すべきかすべきでないかは
計算できるでしょう。

確率分布の設定が出来ないなら、問題が解けるはずがない。
サイコロの目のそれぞれが出る確率が分からずにサイコロに
関する確率の問題を解いているようなもんです。


698 名前:132人目の素数さん:2006/06/19(月) 23:50:51
大昔、ほんの少しだけ議論された六面体予想

ttp://web2.incl.ne.jp/yaoki/ahm6men.htm

解決とは言い難いけど、立方体が最大値っていうのは
ほぼ間違いなさそう。

699 名前:668:2006/06/20(火) 01:10:50
>>697
>金額がxである確率p(x)を。 当然Σp(x) = 1な奴を。
そうだな、無限小みたいな量εでp(x)=ε,lim[n→∞]Σ[i=1〜n]ε=1 を
満たすものを持ってくれば等確率になるから「見たら必ず変える」という戦略が
成り立つんじゃねーの。

700 名前:688:2006/06/20(火) 12:16:45
>>688 の解答例




arctan(1/1)=π/4 …@

(2+i)(3+i)=5+5i より
arctan(1/2)+arctan(1/3)=π/4 …A

直角三角形の内角の和より
arctan(1/4)+arctan(4/1)=π/2 …B

@、A、Bより
x=1/4


701 名前:132人目の素数さん:2006/06/20(火) 14:29:00
>>692
俺が元々聞きたかったのは問題2の方だな
問題の書き方がまずかったかな
で、問題2をこう変えたらどうなる?

問題:今、ここにお金の入った封筒が2つあります。
一つの封筒にはもう一つの封筒の2倍の金額が入っています。
今あなたはどちらか一方の封筒を選ぶ事が出来ます。
選んだ後、開けて金額を確かめ、別の方に撰択を変える事も出来ます。
さて、最適な戦略は?

常に別の方に変えるのが答え?

702 名前:132人目の素数さん:2006/06/20(火) 15:11:40
横から失礼。
すべての正の金額が等確率で出るという仮定の下、期待値だけで判断するなら>>701は正しいでしょ。
ところで
> 金額がxである確率p(x)を。 当然Σp(x) = 1な奴を。
が自然数全体に入れるのが難しいなら正の実数全体に入れればいかがか。

703 名前:694:2006/06/20(火) 15:25:54
自然数nに対してp(n)=2^(-n)とか、具体的に設定すれば
計算できますよ。

全ての自然数に対して確率が同じで、和が1ってのは無理だろうけど。

同様に正の実数に対して、∫p(x)dx=1 な分布はいくらでもあるだろうけど、
p(x)が全部のxに対して同じってのは無理でしょう。


704 名前:668:2006/06/20(火) 15:53:05
>>703
無限小みたいな量εでp(x)=ε,lim[n→∞]Σ[i=1〜n]ε=1 を
満たすものを持ってくれば等確率になるから、「見たら必ず変える」という
戦略が成り立つんじゃねーの。

705 名前: ◆BhMath2chk :2006/06/20(火) 18:00:00
>>687
x=y=z。

a+b+c=0。
x=ab^2d。
y=bc^2d。
z=ca^2d。


706 名前:132人目の素数さん:2006/06/20(火) 20:33:04
>>705
why?

707 名前:132人目の素数さん:2006/06/20(火) 21:38:40
以下の□の中に1から9までの整数をそれぞれ1個ずつ入れ、等式を完成させて下さい。
ただし、1と5はすでに入っているものとします。
(□□−□)÷(□□−□□)=15

708 名前:132人目の素数さん:2006/06/20(火) 22:34:54
(48−3)÷(72−69)=15
…あんまり、面白いとは言えないなあ。

709 名前:132人目の素数さん:2006/06/23(金) 00:51:02
平面上において(x,y)を(ax+c,ay+d)に移す変換T(ただしa<1)を行ったとき、
T(長方形ABCD)⊂長方形ABCDとなった。
この時T(X)=Xとなる不動点Xを点A,B,C,Dから定規だけで作図する方法は存在するか?


710 名前:132人目の素数さん:2006/06/23(金) 21:36:11
双子の兄弟がいます。
そのうちのどちらかはジョンです。
そのうちのどちらかはいつも嘘をつきそのうちのどちらかはいつも本当のことをいいます。

今どちらがジョンかを当てます。
それには英語で3語の質問を双子の兄弟のどちらかにすればよいです。
なんと質問すればいいでしょう?


711 名前:132人目の素数さん:2006/06/23(金) 21:43:17
>710
QBK

712 名前:Geek ◆8MQVxjnUkg :2006/06/23(金) 22:23:48
Is John liar?

713 名前:132人目の素数さん:2006/06/23(金) 23:18:29
>>712
liarの前に"a"がないから英語として間違ってるよ
Which is John?で良いんじゃね?

714 名前:Geek ◆8MQVxjnUkg :2006/06/23(金) 23:21:34
やっぱりそうか?
でもWhich is John?じゃだめじゃない?
johnとliarは質問文に含まれなければダメと思うんだが。

715 名前:710:2006/06/23(金) 23:29:54
>>714

そのとおり。Which is John?はダメです。



716 名前:Geek ◆8MQVxjnUkg :2006/06/23(金) 23:32:37
そのとおりって、
Is John liar? はあってるの?
あ、Does John lie? か?


717 名前:710:2006/06/23(金) 23:33:49
>>716
二つとも違います

718 名前:710:2006/06/23(金) 23:35:08
間違えました。
正解はDoes John lie?です。

719 名前:Geek ◆8MQVxjnUkg :2006/06/23(金) 23:35:39
ぎゃーん。。じゃあなんだ??
俺はさじ投げ。ほかの人もよければ
回答がほしい。

720 名前:Geek ◆8MQVxjnUkg :2006/06/23(金) 23:36:10
あ、あってたんだwよかった。

721 名前:132人目の素数さん:2006/06/23(金) 23:37:17
過去ログ置き場に、datファイルを追加しときました。
必要な方は落として使ってください。

ただ、Janeで動作確認済みですが、それ以外はわかりません。

722 名前:132人目の素数さん:2006/06/23(金) 23:37:57
「わかった!」とアリスの友達の一人がいいました。

723 名前:710:2006/06/23(金) 23:42:20
論理パズルの好きな方にはおすすめ傑作です。
レイモンド・M・スマリヤン著「パズルランドのアリス」

724 名前:132人目の素数さん:2006/06/24(土) 04:48:17
>>709
条件ぬけてないか?
ABCDだけじゃXは決まらんジャロ 

725 名前:132人目の素数さん:2006/06/24(土) 07:48:39
>>724
Xが一意的に決まらないって事か?Xを定規だけでは決められないって事か?

726 名前:132人目の素数さん:2006/06/24(土) 09:46:08
縮小写像だから不動点が一意に存在するのは明らかですね。

ABCDの移動先A'B'C'D'は作図に使えると思っていいのかな?


727 名前:132人目の素数さん:2006/06/24(土) 10:26:30
>>726
A,B,C,D,A',B',C',D'は作図に使えるという事でおねがいします

728 名前:132人目の素数さん:2006/06/25(日) 19:34:52
ふつうにAA’、BB’、CC’、'DD'をそれぞれ結んだ直線の交点じゃいかんの?

729 名前:132人目の素数さん:2006/06/25(日) 19:56:17
Is John alive?でもいいね。

730 名前:132人目の素数さん:2006/06/26(月) 03:48:13
よくない

731 名前:132人目の素数さん:2006/06/29(木) 22:34:49
正多面体が5種類しかないことを示しなさい。

732 名前:132人目の素数さん:2006/06/30(金) 02:53:09
「あるとしても5種類以下である」ことは簡単に示せるけど、「ちょうど5種類である」ことは
どうやって示すのだろう。つまり、立方体とか正20面体とかが隙間なく閉じた曲面を作ることを
どうやって示すのだろう。特に正12面体。

733 名前:132人目の素数さん:2006/06/30(金) 04:09:37
>隙間なく閉じた曲面を作る
こんなこと照明する必要ないような、

まあ厳密にやりたいなら位相幾何を勉強することになるんだろうけど

734 名前:132人目の素数さん:2006/06/30(金) 04:15:05
なんで位相幾何

735 名前:132人目の素数さん:2006/06/30(金) 16:16:44
その前にここでの正多面体を定義してくれ
定義次第で6種類目以降が示せてしまう

736 名前:132人目の素数さん:2006/06/30(金) 21:31:19
正多面体とは男のロマンである。

737 名前:132人目の素数さん:2006/06/30(金) 21:33:07
実際に構成すればお仕舞い。ユークリッドが既に原論でやっている。

738 名前:132人目の素数さん:2006/06/30(金) 23:34:49
星型正多面体の立場も考えて(ry

739 名前:132人目の素数さん:2006/06/30(金) 23:51:52
>>738
星型正多面体のこと、時々でイイから思い出してあげてくださいね。(AA略)

740 名前:132人目の素数さん:2006/07/01(土) 13:06:39
lim[ε↓0]Σ[n=1〜∞]1/(n^2+ε)を求めよ。

741 名前:132人目の素数さん:2006/07/01(土) 13:09:39
ふーん

742 名前:132人目の素数さん:2006/07/01(土) 21:18:44
数列
10001,100010001,1000100010001,10001000100010001,...
には素数が含まれないことを証明せよ。

743 名前:132人目の素数さん:2006/07/01(土) 21:20:21
ちなみに
100010001=3*7*13*37*9901
10001000100010001=41*271*3541*9091*27961

744 名前:132人目の素数さん:2006/07/01(土) 21:21:00
あと10001=73*137

745 名前:132人目の素数さん:2006/07/01(土) 22:18:05
>>742
奇素数pについて(10000^p-1)/9999が(10^p-1)/9で割り切れることを示せば十分。
{(10000^p-1)/9999}/{(10^p-1)/9}=(10000^p-1)/{1111(10^p-1)}
ここで10000^p-1は1111でも10^p-1でも割り切れ、かつ1111と10^p-1は互いに素であるから
10000^p-1は1111(10^p-1)で割りきれる。よって示せた。

746 名前:132人目の素数さん:2006/07/01(土) 23:43:50
>>745
正解

747 名前:132人目の素数さん:2006/07/02(日) 11:53:18
ある夫婦には子どもが二人います。上が男の子のとき、下も男の子である確率

748 名前:132人目の素数さん:2006/07/02(日) 12:00:49
>>747
下が男の子であることは、上が男であるか女であるかとは独立しているから、
下が男である確率は1/2となる。

749 名前:132人目の素数さん:2006/07/02(日) 13:05:41
それは子どもの男女比が1:1のとき。

750 名前:132人目の素数さん:2006/07/02(日) 13:07:15
各要素が1,2,3のどれかからなる数列a1,a2,a3,…を考える。
この数列は…,1,3,1,3,…や…,3,2,1,2,3,2,1,2,…や…,2,1,1,3,3,2,1,1,3,3,…などの
有限数列が2回繰り返すような部分を必ず含むと言えるか?

751 名前:132人目の素数さん:2006/07/02(日) 14:22:08
>>784
子供が80歳を超えると独立していない。

752 名前:132人目の素数さん:2006/07/04(火) 12:53:41
上位2桁で四捨五入する関数Round(x)の式教えて下さい

例:R(89340)=90000、R(5400)=5000、R(990)=1000

753 名前:132人目の素数さん:2006/07/04(火) 14:27:55
Round(x) = [ x/(10^[log_10(x)])+1/2]*(10^[log_10(x)])

754 名前:132人目の素数さん:2006/07/04(火) 22:07:11
aとbを互いに素な自然数とし、f(n)=(nをbで割った余り)とおく。
min[n∈N,n<b]f(an)/nを求めよ。

755 名前:132人目の素数さん:2006/07/05(水) 22:35:51
>>750
言えない。

この問題、数セミのエレ解で見た記憶があったんで
探してみたら1998年7月号出題(10月号解答)だった。
まさにエレガントな解答が載ってた。

756 名前:132人目の素数さん:2006/07/06(木) 00:55:16
どっかのホテルのメニューの話だったことだけは覚えてるw

757 名前:132人目の素数さん:2006/07/06(木) 01:01:34
>>756
ホテルじゃなくて、食堂だった希ガス

758 名前:132人目の素数さん:2006/07/06(木) 01:18:21
あーだったかもー

759 名前:132人目の素数さん:2006/07/06(木) 03:03:56
これがその問題文。

 とある年中無休の研修所のお昼ご飯は選択の余地がなく,カレーライスか牛丼か親子丼かスパゲティかのどれか一つ,利用者全員が同じ種類のものを食べます.
しかしながらこの研修所の食堂のおじさんのモットーは,利用者を飽きさせないように4品を出す順番を工夫して,どの日から何泊してる人にとっても,
お昼ご飯のパターンが連続して繰り返すこと(たとえば…親親…とか…カ牛カ牛…とか…牛ス親ス牛ス親ス…とか)がないようにすることです.
さて,このおじさんはどのような方法で毎日のメニューを決めていけばよいでしょうか.
 誤解を防ぐために,数学用語で問題を言い直しておきます.
数列{a_n}が繰り返し部分をもつとは,a_{k+i}=a_{k+l+i}(i=0,…,l-1)を満たす整数kと1以上の整数lが存在するときをいいます.
1,2,3,4だけを要素とする無限列で繰り返し部分をもたないもの(を生成するアルゴリズム)を見つけてください.
 たった4品のメニューで頑張っていたおじさんでしたが,不況の折,経費削減のためスパゲティを除く3品にメニューを減らすことになりました.
さて,このおじさんのモットーは依然として実現可能でしょうか.
余裕のある人はこの問題も考えてみてください.
 ヒント:繰返し部分をもたない短い数列から,その中の1,2,3,4をそれぞれ適当な有限列に置き換るというフラクタル的な操作で,繰返し部分をもたない長い数列が作れます.



食堂のおじさんには悪いが、どう頑張っても利用者は飽きると思うw

760 名前:132人目の素数さん:2006/07/06(木) 03:04:56
俺も思ったw

761 名前:132人目の素数さん:2006/07/06(木) 03:21:43
バカすぎるwwメニュー増やすことにアタマ使えよw

762 名前:132人目の素数さん:2006/07/06(木) 03:22:22
>>755
情報thx!

763 名前:132人目の素数さん:2006/07/06(木) 10:59:28
閉区間[0,1]上の有界関数からなる空間をB[0,1]、連続関数からなる空間をC[0,1]とする。
B[0,1]、C[0,1]にはそれぞれ||f||=sup_[0≦x≦1](f(x))でノルムを入れる。
この時、∀f∈C[0,1](Tf=f)となるような有界線型作用素T:B[0,1]→C[0,1]は存在するか?

764 名前:KingOfUniverse ◆667la1PjK2 :2006/07/08(土) 08:04:38
x軸上、1秒間だけ正の方向に秒速40で動点が移動し、その直後1秒間だけ正の方向に秒速60で動点が移動した。この2秒間の動点の平均速度を求めよ。

765 名前:132人目の素数さん:2006/07/08(土) 08:40:55
50だろ普通に

766 名前:KingOfUniverse ◆667la1PjK2 :2006/07/08(土) 08:50:06
talk:>>765 そうだな。

767 名前:132人目の素数さん:2006/07/08(土) 15:14:32
Rの自己同型でQを固定するもの(=Gal(R,Q))を総て求めよ。

768 名前:132人目の素数さん:2006/07/08(土) 21:46:34
767は、答えが面白いんだが誰かやってみない?

769 名前:KingOfUniverse ◆667lnVUqRY :2006/07/08(土) 22:15:41
人の脳お読む能力を悪用する奴を潰せ。

770 名前:KingOfUniverse ◆667la1PjK2 :2006/07/09(日) 06:17:57
人の脳を読む能力を悪用する奴を潰せ。

771 名前:132人目の素数さん:2006/07/09(日) 09:02:56
半径rの球を亀甲縛りにするとき、6角形のいっぺんの長さを求めよとか?

772 名前:中川泰秀 ◆5xTePd6LKM :2006/07/09(日) 09:05:37
ストリップの亀甲縛りの1辺は何度か  ?

773 名前:132人目の素数さん:2006/07/10(月) 01:07:45
素数が無限にあることを証明せよ。

774 名前:132人目の素数さん:2006/07/10(月) 01:51:24
素数が無限にあることを証明せよ。
↓up
素数に無限があることを証明せよ。

775 名前:132人目の素数さん:2006/07/10(月) 01:55:51
フェルマの素数が無限にあることを証明せよ。

776 名前:132人目の素数さん:2006/07/10(月) 02:46:22
フェラ魔の素人が無数にいることを証明せよ。

777 名前:132人目の素数さん:2006/07/10(月) 07:19:43
773数が無限にあるからじゃだめ?

778 名前:132人目の素数さん:2006/07/10(月) 11:08:13
>777 :132人目の素数さん :2006/07/10(月) 07:19:43
> 773数が無限にあるからじゃだめ?

n個、p1,…,pn とするとp1*…*pn + 1  はp1,…,pnのいずれでも割り切れないから矛盾

でいいんじゃ?

779 名前:132人目の素数さん:2006/07/10(月) 11:24:31
2*3*5*7+1、2*3*5*7*11+1、2*3*5*7*11*13*17*19*23*29*31+1は素数となる事を証明せよ

780 名前:132人目の素数さん:2006/07/10(月) 19:05:29
>>765
ちょっ、そんなわけないって。
1秒間の平均速度が100のとき、
2秒で平均速度を200にすることは不可能なのと同じ。


2/{1/40+1/60}=48

781 名前:132人目の素数さん:2006/07/10(月) 19:42:21
>>780
どゆこと?
単純に2秒間で100動いたじゃだめなわけ?


782 名前:132人目の素数さん:2006/07/10(月) 19:54:51
>>780
初めの1秒で40動き、次の1秒で60動いたから、この2秒間で100動いたことになる。
よってこの2秒間の平均速度は50。

783 名前:132人目の素数さん:2006/07/10(月) 21:17:17
>767
 f(x) = x       (x∈Q)
 f(x) = (ax+b)/(cx+d) (x∈R-Q)
 ただし a〜d∈Q, ad-bc≠0.

ぢゃあない罠....

784 名前:132人目の素数さん:2006/07/10(月) 21:49:42
>>781
スマソ。ただの勘違いだった。

785 名前:132人目の素数さん:2006/07/11(火) 02:30:52
自作問題。

実数列{an}は単調減少数列で、0に収束するとする。このとき
liminf[n→∞]n*{an−a(n+1)}=0となることを示せ。

786 名前:132人目の素数さん:2006/07/11(火) 02:47:36
>>785
なんか、直感に反する結果だな……
真面目に考えてみるか。

787 名前:785:2006/07/11(火) 02:53:01
おーっと、メチャクチャ簡単に出来てしまったぞ(゚A゚)無かったことにしといてくれ…

788 名前:132人目の素数さん:2006/07/11(火) 02:57:11
確かに……オレもできてしまった。。。>>786も無かったことに

789 名前:132人目の素数さん:2006/07/11(火) 03:01:14
lim で同じことが言えるような
まあいずれにせよこれだけじゃあまり面白くないが

790 名前:132人目の素数さん:2006/07/11(火) 04:20:00
いえない。


791 名前:132人目の素数さん:2006/07/11(火) 08:15:04
>>785
ニヤニヤ…

ヒント つ【数蝉過去問】

792 名前:132人目の素数さん:2006/07/11(火) 18:52:57
N=(A1∪A2∪…∪An)となる時、
A1〜Anのいずれかは2a=b+cとなる{a,b,c}を部分に含む事を示せ

793 名前:132人目の素数さん:2006/07/14(金) 21:23:31
>>792と一見同じようで違う問題が過去ログにあった。

面白い問題おしえてーな 九問目
http://www3.tokai.or.jp/meta/gokudo-/omoshi-log/1093676103.html
962 :132人目の素数さん :2005/05/06(金) 16:59:38
自然数の集合NをS1〜Smに分割したら、どれもx+y=zを満たすようなx,y,zのペアを含まなかった。
そんな風に分割出来る自然数mは存在するか?

これに対し同スレ971で解答が書かれていた。
>>792を同じような方法で解けないかさんざん考えたけど、難しい…。

794 名前:132人目の素数さん:2006/07/15(土) 08:50:55
Rの部分集合Aの各点aは孤立点であるとする。すなわち、任意のa∈Aに対して
あるδ>0が存在して(a−δ,a+δ)∩A={a}が成り立つとする。
(1)Aは高々可算集合であることを示せ。
(2)Aが有界集合ならば、Aは有限集合であることを示せ。

795 名前:132人目の素数さん:2006/07/15(土) 11:54:06
(1)x+1≦e^x (等号はx=0のとき)が成り立つことを示せ。
(2)x+1>0のときe^{x/(1+x)}≦x+1 (等号はx=0のとき)が成り立つことを示せ。
(3)自然数n,m (n>1)に対して(1−1/n)^m=e^(-r)−ε(n,m),r=m/n,
0<ε(n,m)<re^(-r)/(n−1)<1/(n−1) が成り立つことを示せ。

確率1/nで当たるくじをm回引いたとき、全部外れる確率をPとするとP≒e^(-m/n)になる、ということ。
しかもPとe^(-m/n)の誤差凵=bP−e^(-m/n)|は凵1/(n−1) となっていて、非常によい近似を
与えている(通常、nは相当大きな数字だから)。

796 名前:132人目の素数さん:2006/07/15(土) 14:09:29
>>794
(1)各a∈Aに対しq∈(a-δ/2,a+δ/2)∩Qを対応させれば
AとQの部分集合が1対1に対応するのでAは高々可算。
(2)成り立たない。反例{1/n|n∈N-{0}}

797 名前:132人目の素数さん:2006/07/15(土) 20:10:52
>(2)成り立たない。反例{1/n|n∈N-{0}}
あ…本当だ。ゴメン。「有界な閉集合」とでもしないと そうなっちゃうね。

798 名前:132人目の素数さん:2006/07/15(土) 20:34:56
{1/n ; n∈N-{0} } ∪ {0}
は閉集合だけど、、

Aが〜〜を満たすなら、Aの濃度は実数体の濃度と等しいか、
または高々可算であることを示せ、とかいう問題は結構面白いかもw
この場合は連続体仮説を使わなくても証明できるんだよ、みたいなw

799 名前:132人目の素数さん:2006/07/15(土) 23:00:00
コンパクトなら有限。


800 名前:132人目の素数さん:2006/07/15(土) 23:47:44
テクマクマヤコン。

801 名前:132人目の素数さん:2006/07/17(月) 10:20:04
複素数の集合Cの部分集合Aに対しP(θ)[A]={a*e^(iθ)|a∈A}とおく。
たとえばA={x+iy|x^2+y^2=1}=単位円周 のときP(θ)[A]=Aである。
次の条件を満たすA,θは存在するか?
・P(θ)[A]≠AかつP(θ)[A]⊂Aが成り立つ。(回転しただけなのに、元の集合の真部分集合となってしまう)

802 名前:132人目の素数さん:2006/07/17(月) 10:45:07
{e^(in) | n∈N}とか
なんかバナッハ・タルスキーっぽい

803 名前:132人目の素数さん:2006/07/17(月) 22:47:01
ならばP(x)[A]∩P(y)[A]=φでP(x)[A]∪P(y)[A]⊂Aとなる
x,y,Aは存在するか?

804 名前:132人目の素数さん:2006/07/17(月) 23:16:14
>>803
A=φのみ。
そのようなx,y,Aが存在すればP(x+y)[A]はP(x)[A]およびP(y)[A]の部分集合となるから。

805 名前:132人目の素数さん:2006/07/17(月) 23:25:00
>>796
>各a∈Aに対しq∈(a-δ/2,a+δ/2)∩Qを対応させればAとQの部分集合が1対1に対応するので
qをよほどうまく取らないと1対1対応しないんじゃまいか?a,a’∈Aに対して、変なδ,δ’を
取ると(a−δ,a+δ)∩A={a},(a’−δ’,a’+δ’)∩A={a’},(a−δ,a+δ)∩(a’−δ’,a’+δ’)≠φ
となってしまうから、qは(a−δ,a+δ)∩(a’−δ’,a’+δ’)から取っている可能性がある。
これを防ぐには、δとδ’を十分小さくとって(a−δ,a+δ)∩(a’−δ’,a’+δ’)=φとすれば
良いように見えるが、もしAが非可算ならば、これが出来ないこともあるかもしれない。

806 名前:132人目の素数さん:2006/07/17(月) 23:27:15
今日たまたま塾のお便りに3、4、7、8を使い、*したり、+したり、-したり、/したりして10を作れというおもしろ問題が載っていたのだが、誰も分からないままなんだよね。
数学が得意な友達も含め、10人以上にメルして見たが誰も分からないまま。ちなみに数字は一回ずつしか使いません。計算の順番はもちろん自由です。
数字と数字をくっつけたりはしません(3、4→34はできない) 誰か分かる人が現われることを祈ります。

807 名前:132人目の素数さん:2006/07/17(月) 23:37:19
>>806
(3-7/4)*8とか?

808 名前:132人目の素数さん:2006/07/17(月) 23:39:40
>>805
δは
sup {x | (a-x,a+x)∩a = {a} }/2とかを取ると
一意に定めてやることが出来るし、それぞれのaたちを含む開区間は交わらない。
qはNで番号付けして一番最初のを取ってやればいい。

よって全て問題ない。

809 名前:132人目の素数さん:2006/07/17(月) 23:41:41
>>805
「Rの部分集合Aの各点aは孤立点であるとする」という条件があるので無問題。
q∈(a-δ/2,a+δ/2)∩Qのδは(a-δ,a+δ)∩A={a}を満たすδだけど、
隣りのaと重複してqを取ってしまうことが無いように「δ/2」としている。

810 名前:132人目の素数さん:2006/07/17(月) 23:53:57
>sup {x | (a-x,a+x)∩a = {a} }/2とかを取ると
なるほど…これなら重複しないな。ありがとう。

811 名前:132人目の素数さん:2006/07/18(火) 21:49:14
6,8,6,9を使って10を作れ。数字はこのままの順で使用すること。

812 名前:132人目の素数さん:2006/07/18(火) 21:52:03
>>811
書き忘れたけど、数字をつなげて2桁にするのは禁止。

813 名前:132人目の素数さん:2006/07/18(火) 21:52:40
>>811
しまった、√も禁止!

814 名前:132人目の素数さん:2006/07/18(火) 22:00:04
>>811
忘れてた。四則演算も禁止!

815 名前:132人目の素数さん:2006/07/18(火) 22:01:40
ちょwww
>>811=>>812=>>813>>814です、念のため。

816 名前:132人目の素数さん:2006/07/18(火) 22:10:00
6+8^(6/9)=10。


817 名前:132人目の素数さん:2006/07/18(火) 22:11:17
>>816
うおっ、正解!

818 名前:132人目の素数さん:2006/07/18(火) 23:57:23
次の4つの数を使って四則演算のみで10を作れ。
ただし4つの数はそのままの順序で、それぞれ1桁の数として使用すること。
(1) 1,1,9,9 (2) 1,2,8,8 (3) 1,4,3,6 (4) 1,4,6,6 (5) 1,7,3,3 (6) 1,9,1,9
(7) 3,1,7,3 (8) 3,2,4,3 (9) 3,3,3,9 (10) 3,4,2,3 (11) 3,4,3,2 (12) 3,6,8,3
(13) 3,7,3,1 (14) 3,7,4,8 (15) 4,1,4,8 (16) 4,2,3,3 (17) 4,3,2,3 (18) 4,6,6,9
(19) 4,7,5,1 (20) 5,0,3,6 (21) 5,0,4,8 (22) 5,3,0,6 (23) 5,3,8,2 (24) 5,4,0,8
(25) 5,5,2,4 (26) 5,6,5,7 (27) 5,8,7,9 (28) 5,8,9,7 (29) 5,9,5,8 (30) 5,9,6,6
(31) 6,9,6,4 (32) 6,9,8,7 (33) 6,9,9,6 (34) 7,2,3,6 (35) 7,2,4,7 (36) 7,3,1,3
(37) 7,8,9,6 (38) 8,1,1,5 (39) 8,3,3,6 (40) 8,3,4,8 (41) 8,3,7,4 (42) 8,9,4,1
(43) 8,9,5,1 (44) 9,1,1,9 (45) 9,1,9,1 (46) 9,4,1,8 (47) 9,6,6,4 (48) 9,6,9,6

819 名前:132人目の素数さん:2006/07/19(水) 05:17:54
↓ここに逝け
【数学】メイク10で遊ぼうぜ!@数学板【関係】
http://science4.2ch.net/test/read.cgi/math/1017740389/l50

820 名前:132人目の素数さん:2006/07/19(水) 17:08:09
脳を鍛える2chトレーニングか

821 名前:132人目の素数さん:2006/07/20(木) 11:43:48
>>701
封筒に入っている金額がx,2xとすれば
はじめにxを取ったときは取り直せばx増える。
はじめに2xを取ったときは取り直せばx減る。
取り直しをしても期待値は1.5xのまま変わらない。


822 名前:132人目の素数さん:2006/07/20(木) 12:22:42
>>821
xの値が分かっているならば、そうなるけどね。

823 名前:132人目の素数さん:2006/07/21(金) 09:34:03
次の賭け事をやることは得か損か判定せよ。
・1回につき2k円で、サイコロを6個渡される。
・サイコロは6個いっぺんに全部投げる。(まあ1個ずつ投げても同じだが。)
・少なくとも1個は1の目が出ていれば3k円もらえて、そこでオシマイ。1の目が1個もなければ何ももらえず、そこでオシマイ。

824 名前:132人目の素数さん:2006/07/21(金) 11:22:53
k=0の時損でも得でもない。

825 名前:132人目の素数さん:2006/07/21(金) 11:33:32
一の目がひとつも出ない確率は(5/6)^6
少なくとも1個は1の目が出る確率は1-(5/6)^6
もらえる期待値3k{1-(5/6)^6}
3k{1-(5/6)^6}-2k>0⇔k>0のとき得
3k{1-(5/6)^6}-2k=0⇔k=0のとき損得なし
3k{1-(5/6)^6}-2k=0⇔k<0のとき得

ただk<0のときはどうなのかなと。

826 名前:132人目の素数さん:2006/07/21(金) 11:42:26
3*{1-(5/6)^6}-2=-0.00469・・・<0なんで、
k<0のとき得かと

827 名前:132人目の素数さん:2006/07/21(金) 11:54:09
じゃあk>0のとき損なのか。

828 名前:132人目の素数さん:2006/07/21(金) 12:49:17
四つの9と、たす、ひく、わる、かける、かっこ、いこーる、を三つ使って100をつくれ!
って、問題とける?

829 名前:132人目の素数さん:2006/07/21(金) 12:53:51
99+9/9=

830 名前:132人目の素数さん:2006/07/21(金) 12:55:52
>825
ギャンブルとしては寺銭がかなりやすい
お得かも

831 名前:132人目の素数さん:2006/07/21(金) 12:58:07
ごめん、四つ使ってだった

832 名前:132人目の素数さん:2006/07/21(金) 13:33:17
99+(9/9)

833 名前:132人目の素数さん:2006/07/21(金) 18:19:07
ああああ……kは自然数のつもりだったのだが、言葉足らずだったね(^ ^;
>>826
正解。

では次の問題。

次の賭け事をやることは得か損か判定せよ。ただしk,nは自然数とし、n≧2とする。
・1プレイにつき2k円で、1/nの確率で当たるルーレットにn回挑戦できる。
・n回連続でルーレットをやる。
・少なくとも1回当たっていれば3k円もらえて、そこでオシマイ。1回も当たらなければ何ももらえず、そこでオシマイ。

834 名前:132人目の素数さん:2006/07/21(金) 18:23:34
kはキロつまり1000のことかとオモタ

835 名前:132人目の素数さん:2006/07/21(金) 18:23:43
同じでしょ

836 名前:132人目の素数さん:2006/07/21(金) 18:27:17
>>835>>833へのレスね

837 名前:132人目の素数さん:2006/07/21(金) 18:29:47
>>835
理由は?

838 名前:132人目の素数さん:2006/07/21(金) 18:34:04
>>837
すまぬ、早とちりだった・・・。

839 名前:132人目の素数さん:2006/07/21(金) 18:57:33
さいころの問題の6をnに変えただけですね。最後の計算を誰か頼みます。

当たりがひとつも出ない確率は{(n-1)/n}^n
少なくとも1回当たる確率は1-{(n-1)/n}^n
もらえる期待値3k[1-{(n-1)/n}^n]
R=3k[1-{(n-1)/n}^n]-2k=k[1-3{(n-1)/n}^n]
R>0のとき得
R=0のとき損得なし
R<0のとき損
あとはnの範囲を求める。ただ計算が大変。

840 名前:132人目の素数さん:2006/07/21(金) 19:26:09
自分で計算機を使って解決できました。
{(n-1)/n}^nはnが大きくなるにつれ大きくなる。
n=5のとき3{(n-1)/n}^n=0.98304
n=6のとき3{(n-1)/n}^n=1.00469…
k>0を考えると
n=2,…,5のとき[1-3{(n-1)/n}^n]>0なのでR>0
n=6,…のとき[1-3{(n-1)/n}^n]<0なのでR<0

841 名前:132人目の素数さん:2006/07/21(金) 19:40:34
{(n-1)/n}^nの極限てなんだったっけ?

842 名前:132人目の素数さん:2006/07/21(金) 19:46:29
以下の□の中に1から9までの整数をそれぞれ1個ずつ入れ、等式を完成させて下さい。

(□/□□)+(□/□□)+(□/□□)=1

843 名前:132人目の素数さん:2006/07/21(金) 19:56:07
1

844 名前:132人目の素数さん:2006/07/21(金) 20:04:22
>>840
正解。でも、電卓を使ったのなら
>{(n-1)/n}^nはnが大きくなるにつれ大きくなる。
これは単なる「予想」にすぎないわけで、証明が要る。まあ
f(x)=(1−1/x)^xを微分して増減を調べればよいわけだが。
>>841
{(n-1)/n}^n=(1−1/n)^nだから1/e(≒0.368)に収束する。確率1/nのくじをn回連続で引けば、
かなり運が悪くない限り1回は当たりそうに感じるが、「1回も当たらない」確率は実際のところ
どうなっているのかと言うと(1−1/n)^n≒1/e≒0.368となっていて、実は結構な確率で「1回も
当たらない」ことになる。>>795の不等式を使うと1/e>(1−1/n)^n>(1/e)−1/{e(n−1)} になる
ことが分かる。

845 名前:132人目の素数さん:2006/07/22(土) 03:27:18
ここも夏厨スレになってしまったか
皆の衆、秋にまた会おう

846 名前:132人目の素数さん:2006/07/22(土) 08:48:12
夏厨と夏厨厨がウザい時期

847 名前:132人目の素数さん:2006/07/22(土) 09:33:09
長方形I⊂R^2が長方形I1,I2,…,Inに分割されているとする。つまりI=∪[k=1〜n]Ik,
(Ip∩Iq)^i=φ (p≠q)が成り立っているとする。ただしA⊂R^2に対してA^i=(Aの内点の集合)
である。さらに、「Ikの縦の辺と横の辺のうち少なくとも片方は自然数である」が各k=1〜nに
対して成り立っているとする。このとき、Iの縦の辺と横の辺のうち少なくとも片方は自然数で
あることを示せ。

848 名前:132人目の素数さん:2006/07/22(土) 10:13:46
夏厨と夏厨厨のやりとりに反応する厨がウザイ時期

849 名前:132人目の素数さん:2006/07/22(土) 11:28:45
1+2+3+…+4=10

850 名前:132人目の素数さん:2006/07/22(土) 19:05:30
oioi

851 名前:132人目の素数さん:2006/07/22(土) 22:57:05
>>847
Iの1つの頂点からその対角の点まで∂I_kに沿って進むことを考える。
分かれ道では自然数の道を取るようにする(設定上可能)。
どの道も自然数であれば題意は満たされている。
最後のT字路で自然数でない道を進ま無ければならなくなる場合もあるが、
そのときは、その自然数でない道に垂直なIの辺が自然数。


852 名前:851:2006/07/22(土) 23:01:03
隣り合う頂点に到達する場合もあるのでもう少し詳しく説明がありそうな気もするけど、
ま、いいや。

853 名前:851:2006/07/22(土) 23:04:29
対辺みたいなところまでいければいいのか。解決。
スレ無駄遣い失礼。

854 名前:132人目の素数さん:2006/07/22(土) 23:16:59
>>851
>分かれ道では自然数の道を取るようにする(設定上可能)。
可能とは限らないのでは?長方形Iの左下の頂点から右上の頂点まで点Pが移動すると
して、途中でこういうT字路に出会ったら、Pは左右どちらの方向に進んでも無理数に
なっている可能性がある。
←←P→→
   ↑
   ↑
   ↑

855 名前:132人目の素数さん:2006/07/22(土) 23:20:00

┌┬┬─┐
││├┬┤
├┴┤││
├┬┴┼┤
│├─┤│
└┴─┴┘
        B

┌─┐
└─┘
(1/2)×1。


856 名前:132人目の素数さん:2006/07/24(月) 21:33:00
マルチでお邪魔します

天才的なひらめき募集

0〜4と+-*/根号、累乗、階乗、括弧を利用して1000まで作ろうぜ
http://ex16.2ch.net/test/read.cgi/news4vip/1153548534/l50

857 名前:132人目の素数さん:2006/07/26(水) 08:30:29
(X,≦)を全順序集合とする。p∈Xは次の2条件を満たすとする。
・q<pを満たすq∈Xが存在する
・x<pならば、x<y<pを満たすy∈Xが存在する
このとき、次の2条件を満たすXの点列{pn}は存在するか。
・pn<p(n+1)<p (n∈N)
・x<pならば、x<pmを満たすm∈Nが存在する

858 名前:132人目の素数さん:2006/07/26(水) 12:56:23
反例 X=ω_1 + 1, p=ω_1

859 名前:132人目の素数さん :2006/07/27(木) 12:22:51
てすと

860 名前:132人目の素数さん:2006/07/28(金) 18:29:35
680

861 名前:たむ ◆Tamu.jIzZ. :2006/07/29(土) 12:39:25
無限に伸びるゴムひもの上を蟻が歩く。
いまゴムひもの長さは1mあり、蟻が端から毎秒1pの速さで歩き始めるのと同時に、ゴムひもを毎秒1m伸ばす。
蟻は反対側の端にたどり着くことが出来るだろうか?
たどり着けないなら理由を、たどり着けるとしたら約何秒後かを求めよ。

862 名前:132人目の素数さん:2006/07/29(土) 17:24:02
マルチ

863 名前:132人目の素数さん:2006/07/30(日) 09:16:39
4秒後

864 名前:132人目の素数さん:2006/07/30(日) 14:54:22
exp(100)-1秒後かな?ちがうと思うが

865 名前:132人目の素数さん:2006/07/30(日) 18:11:37
ゴムの伸ばし方で変わったりしないの?両端を同じ力で引っ張るのと、
片方の端を固定してもう片方の端だけ引っ張るのでは、伸び方が違う。

866 名前:132人目の素数さん:2006/07/30(日) 18:15:00
>>865
同じでしょ。

867 名前:132人目の素数さん:2006/07/30(日) 18:18:18
>>177-184で外出

868 名前:たむ ◆Tamu.jIzZ. :2006/07/30(日) 18:51:50
うは、ガイシュツでしたか・・
失礼しました

>>864
せ、正解です(´・д・`)

869 名前:132人目の素数さん:2006/07/30(日) 19:38:32
>>865
高校の物理を勉強してから来なさい、ぼうや…

870 名前:132人目の素数さん:2006/07/31(月) 03:44:02
ゴムひもの伸びとアリは等速運動をしていると仮定するのか?

871 名前:132人目の素数さん:2006/08/02(水) 12:21:36
ttp://d.hatena.ne.jp/hoshikuzu/20051010#P20051010IFF

「正直族とうそつき族」の類似問題。これらは数学じゃないっぽいけど。

872 名前:132人目の素数さん:2006/08/02(水) 16:07:44
まぁ3人のうち1人が真or偽である事は分かるな

873 名前:132人目の素数さん:2006/08/02(水) 17:03:25
ヤー( ´∀`)/   \(`∀´ )ダー

874 名前:132人目の素数さん:2006/08/02(水) 17:11:39
神様キモイwww

875 名前:132人目の素数さん:2006/08/04(金) 09:08:40
>>861
宇宙の膨張を思い浮かべた

876 名前: ◆KahomJjMRQ :2006/08/05(土) 22:55:36
sage

877 名前:132人目の素数さん:2006/08/06(日) 02:18:34
p,qを互いに素な自然数とし、p<qとする。rをqと互いに素な自然数とする。
T:N→NをT(n)=np/q ( q|nのとき),n+r (それ以外のとき)
と定義すると、任意の自然数kに対して、数列{T^n(k)}(n∈N)は
発散しないことを示せ。ただしT^nはTのn回合成関数とする。

878 名前:132人目の素数さん:2006/08/08(火) 23:17:15
赤と黒のカードそれぞれn枚ずつ、合計2n枚をよく切ったのち伏せて積み上げる。
この山の上から順に1枚ずつカードをめくっていくのだが、毎回めくる前に、
次のカードが赤か黒かを予想する。

ただし、山の中に赤(黒)が多く残っている時は必ず赤(黒)と予想し、
同数の時は1/2の確率で赤or黒と予想するものと決める。

このようにして2n枚のカード全てをめくるとき、
予想が当たる回数の期待値を求めよ。

879 名前:132人目の素数さん:2006/08/09(水) 18:12:59
(1)一辺の長さがaの正三角形の内部または周上に、次の条件を満たす
ように3つの点が配置できるときの、aの最小値を求め、理由も示せ。
・異なる2点間の距離は1以上である
(1)’一辺の長さがaの正三角形の内部または周上に、次の条件を満たす
ように4つの点が配置できるときの、aの最小値を求め、理由も示せ。
・異なる2点間の距離は1以上である
(2)一辺の長さがaの正方形の内部または周上に、次の条件を満たす
ように4つの点が配置できるときの、aの最小値を求め、理由も示せ。
・異なる2点間の距離は1以上である

880 名前:132人目の素数さん:2006/08/11(金) 19:36:12
>>878
前スレの320

881 名前:132人目の素数さん:2006/08/16(水) 11:58:11
(1)正方形を合同でない二つの相似な図形に分割せよ
(2)正三角形を合同でない二つの相似な図形に分割せよ
(3)円を合同でない二つの相似な図形に分割せよ

882 名前:132人目の素数さん:2006/08/16(水) 11:59:00
無限に大きい方眼紙の交点を頂点にする正三角形は存在するか?

883 名前:132人目の素数さん:2006/08/16(水) 12:10:37
すべては数理に支配されていることを示せ?

884 名前:132人目の素数さん:2006/08/16(水) 12:50:00
(0,1]=(∪(a^(2n+1),a^(2n)])∪(∪(a^(2n+2),a^(2n+1)])。


885 名前:king:2006/08/16(水) 16:01:59
すべてはkingに支配されている

886 名前:132人目の素数さん:2006/08/16(水) 16:29:07
オシムJAPAN2010年ワールド・カップ本戦予選リーグ通過の可能性

887 名前:132人目の素数さん:2006/08/17(木) 13:04:03
>>882
平面上で頂点の座標がすべて有理数にすることができる正多角形は正方形だけだったような。

888 名前:132人目の素数さん:2006/08/17(木) 17:23:41
>>887
正三角形の場合は禿ガイ

889 名前:132人目の素数さん:2006/08/17(木) 17:29:43
>>885 俺達elementsはsystemの一部なんじゃん。よってKingOfUniverseなわけよ!どうよ?

890 名前:132人目の素数さん:2006/08/17(木) 17:39:08
>>883宇宙上にある総原子数を数えられるかどうかよりも、地球上の総原子数は有限である。原子は互いに離散であって有限個数なので、地球上の総原子は自然数と対応付けられる。よって全て(の原子)は数理に支配されている。終。どうよ?

891 名前:132人目の素数さん:2006/08/17(木) 17:42:06
>>887
空間の場合は?
さらに、n次元空間の場合は?

892 名前:132人目の素数さん:2006/08/17(木) 17:45:10

真理はkingが持ってるじょ

ゆるして、きんぽー☆

893 名前:132人目の素数さん:2006/08/17(木) 18:14:51
>>891
空間なら正三角形が書けるがな
n次元なら正n角形が書けるがな

894 名前:132人目の素数さん:2006/08/17(木) 18:15:18
>>890
ボーズ・アインシュタイン凝縮のせいで自然数との対応も微妙に

895 名前:132人目の素数さん:2006/08/17(木) 18:16:13
>>893
正方形も仲間に入れてやってくれ

896 名前:132人目の素数さん:2006/08/17(木) 18:23:20
>>894
ボーズ・アインシュタインに任せずに、数学を志す者なら
自ら「原子」を定義し単位とする事がポイント。

897 名前:KingOfUniverse ◆667la1PjK2 :2006/08/17(木) 22:41:06
talk:>>885,>>892 I'm the King of kings.

898 名前:132人目の素数さん:2006/08/17(木) 23:09:24
>>893
それ本当?
3次元空間で(1,0,0),(0,1,0),(0,0,1)は正三角形になるけど、
4次元空間で(1,0,0,0),(0,1,0,0),(0,0,1,0),(0,0,0,1)は正方形ではなく正四面体だよね。
つまりこの方法は高次元の場合に適用できない。
じゃあどうすればいいんだろう?

899 名前:132人目の素数さん:2006/08/17(木) 23:28:52
ひょっとして>>891は多角形の話ではないのか?

900 名前:132人目の素数さん:2006/08/18(金) 00:20:25
kingて誰だよ

901 名前:132人目の素数さん:2006/08/18(金) 00:39:50
4次元空間で(1,0,0,0),(0,1,0,0),(0,0,1,0),(0,0,0,1)は正方形ではなく正四面体だよね。
4次元空間で(1,0,0,0),(0,1,0,0),(0,0,1,0),(0,0,0,1)は正方形ではなく正四面体だよね。
4次元空間で(1,0,0,0),(0,1,0,0),(0,0,1,0),(0,0,0,1)は正方形ではなく正四面体だよね。
4次元空間で(1,0,0,0),(0,1,0,0),(0,0,1,0),(0,0,0,1)は正方形ではなく正四面体だよね。
4次元空間で(1,0,0,0),(0,1,0,0),(0,0,1,0),(0,0,0,1)は正方形ではなく正四面体だよね。

902 名前:132人目の素数さん:2006/08/18(金) 00:48:47
>>901
何が言いたいの?

903 名前:132人目の素数さん:2006/08/18(金) 00:49:37
(・∀・)ニヤニヤ

904 名前:132人目の素数さん:2006/08/18(金) 00:49:39
ああ、>>899へのレスか。

905 名前:132人目の素数さん:2006/08/18(金) 00:58:52
同じものを何回も書く奴は馬鹿と決まっているので

906 名前:132人目の素数さん:2006/08/18(金) 02:26:41
馬鹿っていうやつが馬鹿ってのも昔から決まってるわけで

907 名前:132人目の素数さん:2006/08/18(金) 02:47:41
(・∀・)ニヤニヤ

908 名前:132人目の素数さん:2006/08/18(金) 04:14:28
何次元空間だろうと、cos(2π/n)が無理数ならば有理点の頂点からなる正n角形は存在しない。
もしそのような有理点の組が存在すれば、その重心も有理点であり、これを原点Oとしても一般性を失わない。
その正n角形の頂点のうち隣り合う2点A,BをとるとVec(OA)・Vec(OB)=OA*OB*cos(2π/n)だが、
Vec(OA)・Vec(OB)は有理数であり、またOA*OB=OA^2も有理数なので、cos(2π/n)も有理数となり矛盾。

cos(2π/5)は無理数なので有理点からなる正五角形は存在しないことがわかる。

ちなみに、OA=OBかつVec(OA)・Vec(OB)=OA*OB*cos(2π/n)を満たす2点A,Bが存在することは
有理点の頂点からなる正n角形が存在するための必要十分条件である。
必要性は明らか。次に、条件を満たすA,Bが存在したとして、十分性を示す。
直線OBの式はVec(OP)=t*Vec(OB)
この直線にAから下ろした垂線の足をHとし、Vec(OH)=k*Vec(OB)とすると
(k*Vec(OB)-Vec(OA))・Vec(OB)=0より
k=Vec(OA)・Vec(OB)/(OB^2)
これは有理数となるから、Hは有理点。
よって、Hに関してAと対称な点も有理点であり、この点は正n角形の第3の頂点となる。
同様に第4、第5…の頂点も有理点として得られ、有理点の頂点からなる正n角形が得られる。

909 名前:KingOfUniverse ◆667la1PjK2 :2006/08/18(金) 07:04:12
talk:>>900 何だよ?

910 名前:132人目の素数さん:2006/08/18(金) 17:34:00
3次元空間(あるいは一般にn次元空間)において、有理点の頂点からなる任意の三角形の外心は有理点であることを示せ。

911 名前:132人目の素数さん:2006/08/18(金) 18:36:50
>>910
明らか

912 名前:132人目の素数さん:2006/08/18(金) 19:31:57
>>911
俺には明らかじゃない。

913 名前:132人目の素数さん:2006/08/18(金) 19:38:36
>>910
3本の直線は全部係数が有理数だから、どの2点の垂直二等分線もそうであり、
従ってその交点も有理点になる。でおk?

914 名前:132人目の素数さん:2006/08/18(金) 20:11:09
>>913
3次元空間で垂直二等分線はひとつに定まらない。

915 名前:132人目の素数さん:2006/08/18(金) 20:32:45
無理点を中心に持つ円はその円周上に有理点を最大2個しか持たない、というのを見つけた
んだけど、自力では示せそうに無い。もしこれが示せたら周上に3点の有理点を持つ円の中心は必ず有理点と言うことが出来るんだけど。

916 名前:132人目の素数さん:2006/08/19(土) 17:30:28
>>910-915
3点を通る平面の(連立)方程式は有理的
(各方程式の係数が有理数に取れる)
からそこで考えればよい。垂直二等分線(面)の方程式も有理的。

917 名前:132人目の素数さん:2006/08/20(日) 17:38:05
正の実数列A1,A2,A3…について
どのnに対してもΣ[k<n]Ak^2 < K*An^2となるKが存在する時、
どのnに対してもΣ[k<n]Ak < K'*AnとなるK'が存在する事を証明せよ。

918 名前:132人目の素数さん:2006/08/21(月) 17:19:41
fn:R→R (∀n∈N)を連続関数とするとき、lim[n→∞]fn(x)an=0 (∀x∈R),an≠0 (∀n∈N)
を満たす(xによらない)実数列{an}が存在することを示せ。

919 名前:132人目の素数さん:2006/08/21(月) 20:53:04
あるゲームのゲーム木を考える。
このゲーム木は高さnの完全2分木であるとする。
このゲーム木をミニマックス法で探索するとこを考える。
ここでデータベースをつかって探索を高速化することを考える。
データベースにはあるひとつの局面につき結果(先手勝か後手勝ちか引き分けか)
をサイズO(1)で記憶することができ、計算量O(1)でその結果を得られるものとする。
データベースのサイズと各局面の探索の計算量の平均を
両方nの多項式サイズに収めることは可能か?
※データベースを作成するための時間は考えなくて良い。

答えは俺も知らん。

920 名前:132人目の素数さん:2006/08/22(火) 20:05:15
>>919
それができたらP=NPが示せそうな予感。
あくまで俺の直感だが。

921 名前:132人目の素数さん:2006/08/22(火) 23:31:24
1つのケーキを2人で分けて文句の出ないようにするには、どちらか一人に2分の1に分けさせて、
もう一人が好きな方をとる、という方法が有名ですよね。では

1つのケーキをナイフで切って3人で分け合います。3人とも
「自分は元のケーキの3分の1以上をもらった」
と思えるように分けなさい。

ではどうですか?これガイシュツ?
(つーか何年か前は2ちゃんのどこかにあったんだよね。でもまだ知りたい人いるかなあと
思ってさ。ちなみにステインハウスって人が意外な方法を見つけたんだってね。)

922 名前:132人目の素数さん:2006/08/23(水) 12:43:37
>>917
(Σ[i=1〜n−1]Ai^2)/An^2<K (∃K,2≦∀n∈N) → (Σ[i=1〜n−1]Ai)/An<L (L=K+√(K^2+K),2≦∀n∈N)
が成り立つことを数学的帰納法で示す。なお、K+2KL=L^2,√K<Lとなっていることに注意する。

n=2のとき…(Σ[i=1〜n−1]Ai)/An=A1/A2<√K<L となって、成立。2≦n≦tのとき成り立つとすると、
n=t+1のとき…{(Σ[i=1〜n−1]Ai)/An}^2={Σ[i=1〜t]Ai^2+2Σ[1≦i<j≦t]AiAj}/An^2
={Σ[i=1〜t]Ai^2+2Σ[j=2〜t](A1+A2+…+A(j−1))Aj}/An^2
={Σ[i=1〜t]Ai^2+2Σ[j=2〜t](Aj^2)(A1+A2+…+A(j−1))/Aj}/An^2
≦{Σ[i=1〜t]Ai^2+2Σ[j=2〜t](Aj^2)L}/An^2
<{Σ[i=1〜t]Ai^2+2Σ[j=1〜t](Aj^2)L}/An^2
<K+2LK=L^2
よって(Σ[i=1〜n−1]Ai)/An<Lとなり、n=t+1のときも成立。数学的帰納法から、
(Σ[i=1〜n−1]Ai)/An<L (L=K+√(K^2+K),2≦∀n∈N) は確かに成立する。

923 名前:132人目の素数さん:2006/08/25(金) 21:07:44
>921
最初の人が2:1にわけ、
二番目の人が2の方を等分し、
三番目から逆に取っていく、というのを考えたんですが、
二番と三番が結託する可能性が排除できません...

924 名前:132人目の素数さん:2006/08/25(金) 21:21:31
凸多角形Pの各辺bに対して、bを1つの辺とする三角形であってPに含まれるものの面積の最大値を割りあてる。
この凸多角形Pの各辺に割りあてられた面積の和はPの面積の2倍以上であることを示せ。 2006IMO

925 名前:921:2006/08/26(土) 16:55:39
さすが2ちゃんねる。あんまり食いつきよくないですね。
>>923
http://torito.jp/puzzles/308.shtml
の「ところで」から読んでみなされ。

926 名前:132人目の素数さん:2006/08/26(土) 17:27:10
>>921,925
全員が「自分は1/3以上」と思える分配ならそれほど難しくない。

(液体状の物を)全員が「自分は1位か1位タイ」と思えるように分配するのが困難で、
かつてスレが立って解決した。
http://makimo.to/2ch/science2_math/1060/1060272490.html

927 名前:132人目の素数さん:2006/08/26(土) 20:15:31
>>924
ムズイ

928 名前:132人目の素数さん:2006/08/26(土) 22:00:42
>>925
君はその問題を自力で解けたのかな?

929 名前:132人目の素数さん:2006/08/26(土) 22:04:30
うん

930 名前:132人目の素数さん:2006/08/26(土) 22:17:32
エライッ!

931 名前:132人目の素数さん:2006/08/27(日) 07:18:22
以下の等式を満たす整数の組(x,y)を全て求めよ。 2006IMO
1+2^x+2^(2x+1)=y^2


(x,y)=(0,2),(4,23)で合ってる?

932 名前:132人目の素数さん:2006/08/27(日) 08:55:00
>>931
ヒント:yは負でもよい

933 名前:132人目の素数さん:2006/08/27(日) 09:27:48
>>932
オオウ…やっちまった。これなら合ってる?

(x,y)=(0,±2),(4,±23)

934 名前:132人目の素数さん:2006/08/27(日) 22:02:44
その4組しかないことはどうやって証明するの?

935 名前:132人目の素数さん:2006/08/27(日) 23:18:53
こんな感じ。

y>0としてよい。
x>0のとき…x=1のときは存在しないから、x≧2としてよい。このときyは奇数となるから
y=2k+1 (k≧0)とおける。y=1のとき与式を満たすxは存在しないので、k≧1としてよい。
{1+2^(x+1)}2^x=(y−1)(y+1)=4k(k+1)となるので{1+2^(x+1)}2^(x−2)=k(k+1) …*
となる。k,k+1のうち片方は偶数である。
kが偶数のとき…k+1は奇数なので2^(x−2)|kとなり、k=L*2^(x−2)とおけて、これを
*に代入して変形すると(8−L^2)2^(x−2)=L−1≧0 よってL^2≦8となり、L=1,2となるが、
両方とも*を満たさないことが分かるので不適。
k+1が偶数のとき…kは奇数なので2^(x−2)|(k+1)となり、k+1=L*2^(x−2)とおけて、これを
*に代入して変形すると(L^2−8)2^(x−2)=L+1 となるが、L^2−9<(L^2−8)2^(x−2)=L+1<L+3
よりL−3<1つまりL≦3となるので、L=1,2,3となるが、*を満たすのはL=3のときのみで、このとき
x=4,y=23となる。よって、x>0のときは(x,y)=(4,23)のみ。

x=0のとき…y=±2となり、(x,y)=(0,2)

x<0のとき…z=−x>0とおけば、与式は2+2^z+2^(2z)=(y^2)2^(2z)と
変形でき、両辺2で割って1+2^(z−1)+2^(2z−1)=(y^2)2^(2z−1)となる。
右辺は偶数だから左辺もそうであるが、左辺が偶数となるには2^(z−1)+1が
偶数でなければならず、よってz=1となるしかない。このとき4=2y^2となって
y=±√2となり、不適。
以上より、(yが負のときも入れて) (x,y)=(0,±2),(4,±23)

936 名前:132人目の素数さん:2006/08/29(火) 05:26:40
>>935
なんでyは奇数としていいのですか?

937 名前:132人目の素数さん:2006/08/29(火) 05:55:15
左辺が奇数だから。

938 名前:132人目の素数さん:2006/08/29(火) 06:45:07
お〜なるほど すっきりした

939 名前:132人目の素数さん:2006/08/30(水) 17:51:11


940 名前:132人目の素数さん:2006/08/30(水) 18:11:58
age

941 名前:132人目の素数さん:2006/08/31(木) 03:48:37
解答が出てないようなので。>>847

Iの底辺に接している長方形のうち、縦の長さが自然数である長方形を任意に選ぶ。
その選んだ長方形の縦の長さをLとする。(そういう長方形が無い場合はIの底辺は自然数になっている。)

次にIを底辺から高さLのところで切り、残ったものをI'とする。
I'の底辺に接する長方形の中には切り取られて小さくなった長方形もあるが、どれも、1つの辺は自然数という性質はそのままになっている。

この操作をI'',I'''と繰り返していく。
途中で終わったら(底辺に接する長方形に縦が自然数のものが無くなったとき)Iの底辺が自然数。
最後まで切り取ることが出来たら(Iの高さは有限)Iの高さが自然数。



942 名前:132人目の素数さん:2006/08/31(木) 03:57:55
フィボナッチ数列の一般項ってどうやって求めたの?

943 名前:132人目の素数さん:2006/08/31(木) 04:07:20
>>I'の底辺に接する長方形の中には切り取られて小さくなった長方形もあるが、どれも、1つの辺は自然数という性質はそのままになっている。

やっぱまずいな。わかんね。


944 名前:847:2006/08/31(木) 04:38:18
実は、立ち読みした本に載ってた問題。信じられない解答ですた。

解答:Ik=[ak,bk]×[ck,dk] (k=1〜n)と表記しておく。I=[0,a]×[0,b]
としてよい。さて、I上でf(x,y)=exp(2πix+2πiy)を重積分する。このとき
∫∫(I)f=Σ[k=1〜n]∫∫(Ik)fが成り立つが、フビニの定理から
∫∫(Ik)f=∫[ak,bk]exp(2πix)dx∫[ck,dk]exp(2πiy)dy
={exp(2πi(ak+ck))}{exp(2πi(bk−ak))−1}{exp(2πi(dk−ck))−1}/(2πi)^2
=0 (k=1〜n)
が成り立つ。なぜなら、k=1〜nについて、bk−akかdk−ckのうち
どちらか一方は自然数だから。よって、∫∫(I)f=Σ[k=1〜n]0=0
となり、これと∫∫(I)f={exp(2πia)−1}{exp(2πib)−1}/(2πi)^2
から{exp(2πia)−1}{exp(2πib)−1}=0を得るので、aかbのうち
一方は自然数となる。


Ikについての条件を
・各Ikに対して「Ikの縦の辺と横の辺のうち少なくとも片方は有理数である」
に変えると、
・Iの縦の辺と横の辺のうち少なくとも片方は有理数である
ことが示せる(この解法を応用すればよい)。

945 名前:132人目の素数さん:2006/08/31(木) 07:09:46
自作問題。
0<p<1とする。実数列{an}(n=0,1,2,…)がα∈Rに収束するとき、
lim[n→∞]Σ[i=0〜n−1]ak*p^(n−k) を求めよ。また、なぜその値に
なるのかも証明せよ。一般に、実数列{bn}に対して級数{Σ[i=1〜n]bi}(n∈N)が
絶対収束するとき、lim[n→∞]Σ[k=0〜n−1]ak*b(n−k) を求めよ。
また、なぜその値になるのかも証明せよ。


もしかしたら俺の方が間違っていて、極限が存在しない例が存在する
可能性もあるので、挑戦しようと思う人は気をつけて(^^;

946 名前:132人目の素数さん:2006/08/31(木) 07:11:59
>>942
a[n+2]=a[n+1]+a[n]
特性方程式を使って
a[n]=Pα^n+Qβ^n…@
とおいて(勿論α,βは特性方程式の解)a[1],a[2]を代入、係数比較をしてP,Qを求めるのが早いです。隣接2項間漸化式の一般項が@の形になる事を証明しなければ記述には使えませんが。普通に等比数列作って引き算して出す時は突然3乗を計算するというテクニックが要ります。

947 名前:132人目の素数さん:2006/08/31(木) 10:13:43
>>944
もっと簡単な解法もあるよ。

Iを縦と横に1/2の間隔の平行線で区切り、黒と白の市松模様に塗り分ける。
各Ikにおいて、1辺が整数であることから黒と白の面積は等しい。
よってIも黒と白の面積が等しくなることから、Iの1辺は整数である。

「有理数」の場合、全体を整数倍してIkがそれぞれ整数の辺をもつようにすれば、上の方法が適用できる。

948 名前:132人目の素数さん:2006/08/31(木) 11:17:00
>>944
>>947
一つの本に両方出ているのを読んだ記憶がある。


949 名前:132人目の素数さん:2006/08/31(木) 11:25:05
どいつもこいつも…
何に載っているのか書けよ!
とくに >>944
自分でその本を読んでみたいんだよ! ヽ(`Д´)ノ ガロァ!

950 名前:132人目の素数さん:2006/08/31(木) 11:37:38
>>949
立ち読みしただけで、本のタイトルは覚えてない。ちなみに、その本には他にも
・正方形を、有限個の小さな正方形(ただし大きさは全て異なるようにする)に分割することは可能か?
という問題もあった。答えはYESなんだけど、その証明には なんと電気回路が出てきて、抵抗は全て1Ωで、
キフヒホッフの法則から云々とかやって証明してた。正方形の分割と電気回路が対応するらしい。
図形の三角形分割からホモロジーを求めるような感じか。

951 名前:947:2006/08/31(木) 11:51:13
>>949
何で読んだかは覚えてないけど、
>>944みたいな解答は「蟻を大砲で撃つようなもの」(表現は違ったかも)であって、
問題のレベルに相応しい解答として>>947が紹介されていたと思う。

952 名前:947:2006/08/31(木) 12:12:58
とはいえ、個人的には、>>944も面白い解答だと思う。
>>947と似てると思うんだけど、>>947を難しく言い換えると>>944になるってことなのかな?

953 名前:132人目の素数さん:2006/08/31(木) 15:10:34
>>946
サンクス!
んじゃ問題。
3個の箱A, B, Cがある。そして、いずれかの箱に100万円が入っていて、他の箱には何も入っていない。
あなたは箱を1個を選び、選んだ箱に入っているお金をもらえるとする。
例えばあなたが箱Aを選んだとする。(箱Aはまだ開けない)
どの箱にいくら入っているかを知っている私は、箱B、箱Cのうち空の箱を開ける。(2つとも空の時、箱B、Cを開ける確率は2分の1)
この時点でまだ開けていない2個の箱から、あなたが1個選べるとするとき、また箱Aを選ぶべきか、それとも選択を変えるべきだろうか。

954 名前:132人目の素数さん:2006/08/31(木) 15:31:47
>>953
箱B,Cのうち空の箱を開けたということは、B,Cをひとつの箱にまとめたのと同じ。
だからその時点でAでないほうの箱に2/3の確率で100万円が入っている。

955 名前:132人目の素数さん:2006/08/31(木) 15:58:51
>>953どういたしまして
>>954その考え方いいね!自分は単純に『Aに100万入ってる確率』が1/3だから。ってやったけど結局は同じ事言ってます。

956 名前:132人目の素数さん:2006/08/31(木) 16:25:42
>>954-955
正解!これは簡単だったかな。
次も簡単。
ある国では女子の出生率を増やすために『女子を産んだ夫婦はさらに子供を産んでもよいが、男子を産んだ夫婦はもう子供を産んではいけない』
という法律を作ったとする。この国の男女の比率はどうなるか。

957 名前:132人目の素数さん:2006/08/31(木) 16:28:52
lim[t→∞]のとき∞:0

958 名前:132人目の素数さん:2006/08/31(木) 16:33:55
男と女を生む確率がそれぞれ2分の1なら1:1だ。樹形図を書くとわかりやすい。

959 名前:957:2006/08/31(木) 16:36:51
>>958
はい、そうですね。
子供を生むには男女必要ですものね。

女子が一方的に増えるわけじゃないんだね。
一夫多妻だったりすると、違う答えになるのかな?

960 名前:132人目の素数さん:2006/08/31(木) 17:02:20
>>958
正解です!
1 2 3 4 5 6

男→×
女→男→×
 →女→男→×
   →女→男→×
     →女→男→×
これの繰り返しです。つまりは1:1
>>959
一夫多妻を一つの夫婦とするということですか?

961 名前:132人目の素数さん:2006/08/31(木) 17:34:21
>>960
一夫多妻を一つの夫婦とするということです。

962 名前:132人目の素数さん:2006/08/31(木) 17:37:04
一夫多妻制と言っても何人までという決まりがないと問題的に困ります…

963 名前:132人目の素数さん:2006/08/31(木) 17:43:35
しかし、限界があるんじゃありませんか?
「一夫多妻」女性10人同居 57歳男性
という事件もありましたし、
仮に10人ならどうなるでしょうか?

964 名前:132人目の素数さん:2006/09/01(金) 04:03:48
男女比は相変わらず1:1だがどんどん人口が減っていく。

965 名前:132人目の素数さん:2006/09/04(月) 18:41:47
簡単なやつ

連続する奇数の素数は17・19や41・43など多数存在するが
3連続する奇数で三つとも全て素数の並びは
 存在するか?しないか?
 存在するなら有限か?無限か?

966 名前:132人目の素数さん:2006/09/04(月) 21:33:20
P(x)=∀y,z(x=yz→(y=1 or z=1))として
∃x(x≠1,2 and P(x) and P(x+2) and P(x+4))の真偽を
自然数の公理から導けるプログラムを作ればいいんです!

967 名前:132人目の素数さん:2006/09/04(月) 23:44:34
>>965
する
3,5,7
n,n+2,n+4のうちどれか一つは3の倍数
3の倍数が素数になるには3しかない。
よってn=3の場合だけ。

968 名前:132人目の素数さん:2006/09/05(火) 09:46:00
ある日、王様は何個かのダイヤモンドを王子たちに分け与えることにした。
まず1人目の王子には1個と残りの7分の1を取らせ、
次に2人目の王子には2個と残りの7分の1を取らせ、
次に3人目の王子には3個と残りの7分の1を取らせ、
これを最後の王子まで続けたところ、全てのダイヤモンドがちょうど無くなった。

(1) ダイヤモンドは何個あったか?

(2) この問題において、じつは「7分の1」という数は必然的なものではない。
その理由をエレガントな方法で説明せよ。

969 名前:132人目の素数さん:2006/09/05(火) 10:06:39
>>968
重要な一文が抜けていたので、追加。

「王子たちがもらったダイヤモンドの個数は全員同じだった。」

970 名前:132人目の素数さん:2006/09/05(火) 11:52:51
n人目はn個貰う
n-1人目は(n-1)+(7n/6)*1/7もらう
n=n-1+n/6 n=6
王子は6人居てダイヤモンドは36個あった。

971 名前:132人目の素数さん:2006/09/05(火) 18:12:43
>>970
正解!

(2)の意味をもう少し補足。
問題文の「7分の1」の部分を「8分の1」や「9分の1」などに置き換えても、
つまりnを7以外の2以上の自然数として「n分の1」に置き換えても問題が成立する。
この理由を説明せよということ。
ただし>>969の条件が成り立つことも含めて説明する必要があるので注意。
直観的に納得のいく説明が望ましい。

972 名前:132人目の素数さん:2006/09/05(火) 18:16:46
さらに問題。
>>969の一文が無くても>>968は問題として成立するか?
(答えは知らないが、ひょっとして難問か?)

973 名前:132人目の素数さん:2006/09/05(火) 20:28:15
ダイヤモンドは1個あった。
1番目の王子が 1個と残りの1/7をもらった。
王子はひとりっこだった。


974 名前:132人目の素数さん:2006/09/05(火) 21:04:01
「王子たち」という表現から、王子は複数いると解釈できる。
ということで。

975 名前:132人目の素数さん:2006/09/06(水) 00:25:11
ダイヤモンドの総数をy個、王子のもらった数をx個とすると
1人目 x=1+(y-1)/n
2人目 x=2+(y-x-2)/n
連立方程式を解くと
x=n-1 , y=(n-1)^2

976 名前:972:2006/09/06(水) 03:00:22
>>974
おそらくそう返ってくるだろうと思っていたが
だとすると>>971
「つまりnを7以外の2以上の自然数として」は
3以上に修正しないと不味いのではないか?

977 名前:132人目の素数さん:2006/09/06(水) 15:43:43
>>975
うん、その通り。

>>976
なるほど。
では、>>968の4行目を削除。
ていうか>>972書いたの俺なんだけどw

しかし出題しといてなんだが>>968の(2)はあんまり面白くないかもね。
一応模範解答は用意してある。

978 名前:976:2006/09/06(水) 17:52:26
あら、すまん。間違えてたよ。
>>976は973が書いた。

>>972 は残りの取り分が一般に1/n (n:n>2)の時だとダメ
というか解がひとつに定まらないnがあるようだな。
n=7の場合は解がひとつになりそうな気はしてるんだが‥
もすこし考えてみる。

979 名前:132人目の素数さん:2006/09/06(水) 23:14:28
>>972はn=2の時は無限に解があるが、n≧3のならば解は1つに決まるのでは?


980 名前:132人目の素数さん:2006/09/07(木) 00:11:07
>>979
なんで?

981 名前:132人目の素数さん:2006/09/07(木) 02:01:29
IMO面白い

 ↓ ↓ ↓

IMO 1971の3
IMO 1990の6
IMO 1992の5
IMO 1993の3
IMO 1994の6
IMO 1995の6
IMO 1999の3
IMO 2000の3
IMO 2001の3
IMO 2002の6
IMO 2003の3
IMO 2004の3
IMO 2006の6

982 名前:132人目の素数さん:2006/09/07(木) 05:03:03



983 名前:132人目の素数さん:2006/09/07(木) 07:10:00
面白い問題おしえて〜な 十二問目
http://science4.2ch.net/test/read.cgi/math/1157580000/


984 名前:132人目の素数さん:2006/09/07(木) 11:49:02
http://www6.atwiki.jp/omoshiro2ch/pages/19.html
98さん御苦労

985 名前:132人目の素数さん:2006/09/07(木) 23:47:07
>>980
最後は必ずm番目の王子がm個のダイヤモンドをもらって終わる。
王子は2人以上の場合、最後の王子から数えてk人の王子のもらったダイヤモンドの合計をS_kとすると
S_{k+1}=(m-k+1)+nS_k/(n-1)
S_k=(n-1)(n-1-m+k+(m-n+1){(n/(n-1))^k})
ダイヤモンドの合計S_mは
S_m=(n-1)(n-1+(m-n+1){(n/(n-1))^m})
n=2ならば任意のmでS_mは整数になる。
n≧3の時は、m-n+1=0の時以外は整数にならない。


986 名前:132人目の素数さん:2006/09/08(金) 11:01:19
二百七十日。


987 名前:132人目の素数さん:2006/09/08(金) 11:45:21
>>985
適当な数字を代入して確認してみたけど、式が合ってないような気がす。

988 名前: ◆UJLOsclAdA :2006/09/09(土) 10:07:00
TEST

989 名前:132人目の素数さん:2006/09/09(土) 11:01:19
二百七十一日。


990 名前:132人目の素数さん:2006/09/10(日) 02:03:59
二百六十九日。

991 名前:132人目の素数さん:2006/09/10(日) 08:37:39
なんで減るの? スレが立ってからの日数じゃないのか?

992 名前:132人目の素数さん:2006/09/10(日) 11:01:19
二百七十二日。


993 名前:132人目の素数さん:2006/09/10(日) 11:19:25
>>990は偽者

994 名前:132人目の素数さん:2006/09/10(日) 12:40:45
人の噂も二百六十九日

995 名前:132人目の素数さん:2006/09/10(日) 23:17:43
>>968の王は絶倫

996 名前:132人目の素数さん:2006/09/11(月) 01:41:26
二百六十八日。

997 名前:132人目の素数さん:2006/09/11(月) 05:33:52


998 名前:132人目の素数さん:2006/09/11(月) 05:34:33


999 名前:132人目の素数さん:2006/09/11(月) 05:35:16


1000 名前:小西真奈美:2006/09/11(月) 05:35:57
1000ならジュースでも飲むか

1001 名前:1001:Over 1000 Thread
このスレッドは1000を超えました。
もう書けないので、新しいスレッドを立ててくださいです。。。


全部 最新50
DAT2HTML 0.34a Converted.